You are on page 1of 82

14

CALCULUS OF
VECTOR- VALUED
FUNCTIONS
14.1 Vector-Valued Functions (ET Section 13.1)
Preliminary Questions
1. Which one of the following does not parametrize a line?
(a) r
1
(t ) = 8 t, 2t, 3t
(b) r
2
(t ) = t
3
i 7t
3
j +t
3
k
(c) r
3
(t ) =
_
8 4t
3
, 2 +5t
2
, 9t
3
_
SOLUTION
(a) This is a parametrization of the line passing through the point (8, 0, 0) in the direction parallel to the vector 1, 2, 3,
since:
8 t, 2t, 3t = 8, 0, 0 +t 1, 2, 3
(b) Using the parameter s = t
3
we get:
_
t
3
i 7t
3
j +t
3
k
_
= s, 7s, s = s 1, 7, 1
This is a parametrization of the line through the origin, with the direction vector v = 1, 7, 1.
(c) The parametrization
_
8 4t
3
, 2 +5t
2
, 9t
3
_
does not parametrize a line. In particular, the points (8, 2, 0) (at t = 0),
(4, 7, 9) (at t = 1), and (24, 22, 72) (at t = 2) are not colinear.
2. What is the projection of r(t ) = t i +t
4
j +e
t
k onto the xz-plane?
SOLUTION The projection of the path onto the xz-plane is the curve traced by t i + e
t
k =
_
t, 0, e
t
_
. This is the curve
z = e
x
in the xz-plane.
3. Which projection of cos t, cos 2t, sin t is a circle?
SOLUTION The parametric equations are
x = cos t, y = cos 2t, z = sin t
The projection onto the xz-plane is cos t, 0, sin t . Since x
2
+ z
2
= cos
2
t +sin
2
t = 1, the projection is a circle in the
xz-plane. The projection onto the xy-plane is traced by the curve cos t, cos 2t, 0. Therefore, x = cos t and y = cos 2t .
We express y in terms of x:
y = cos 2t = 2 cos
2
t 1 = 2x
2
1
The projection onto the xy-plane is a parabola. The projection onto the yz-plane is the curve 0, cos 2t, sin t . Hence
y = cos 2t and z = sin t . We nd y as a function of z:
y = cos 2t = 1 2 sin
2
t = 1 2z
2
The projection onto the yz-plane is again a parabola.
4. What is the center of the circle with parametrization
r(t ) = (2 +cos t )i +2j +(3 sin t )k?
SOLUTION The parametric equations are
x = 2 +cos t, y = 2, z = 3 sin t
Therefore, the curve is contained in the plane y = 2, and the following holds:
(x +2)
2
+(z 3)
2
= cos
2
t +sin
2
t = 1
We conclude that the curve r(t ) is the circle of radius 1 in the plane y = 2 centered at the point (2, 2, 3).
242 C HA P T E R 14 CALCULUS OF VECTOR- VALUED FUNCTI ONS (ET CHAPTER 13)
5. How do the paths r
1
(t ) = cos t, sin t and r
2
(t ) = sin t, cos t around the unit circle differ?
SOLUTION The two paths describe the unit circle. However, as t increases from 0 to 2, the point on the path sin t i +
cos t j moves in a clockwise direction, whereas the point on the path cos t i +sin t j moves in a counterclockwise direction.
6. Which three of the following vector-valued functions parametrize the same space curve?
(a) (2 +cos t )i +9j +(3 sin t )k (b) (2 +cos t )i 9j +(3 sin t )k
(c) (2 +cos 3t )i +9j +(3 sin 3t )k (d) (2 cos t )i +9j +(3 +sin t )k
(e) (2 +cos t )i +9j +(3 +sin t )k
SOLUTION All the curves except for (b) lie in the vertical plane y = 9. We identify each one of the curves (a), (c), (d)
and (e).
(a) The parametric equations are:
x = 2 +cos t, y = 9, z = 3 sin t
Hence,
(x +2)
2
+(z 3)
2
= (cos t )
2
+(sin t )
2
= 1
This is the circle of radius 1 in the plane y = 9, centered at (2, 9, 3).
(c) The parametric equations are:
x = 2 +cos 3t, y = 9, z = 3 sin 3t
Hence,
(x +2)
2
+(z 3)
2
= (cos 3t )
2
+(sin 3t )
2
= 1
This is the circle of radius 1 in the plane y = 9, centered at (2, 9, 3).
(d) In this curve we have:
x = 2 cos t, y = 9, z = 3 +sin t
Hence,
(x +2)
2
+(z 3)
2
= (cos t )
2
+(sin t )
2
= 1
Again, the circle of radius 1 in the plane y = 9, centered at (2, 9, 3).
(e) In this parametrization we have:
x = 2 +cos t, y = 9, z = 3 +sin t
Hence,
(x 2)
2
+(z 3)
2
= (cos t )
2
+(sin t )
2
= 1
This is the circle of radius 1 in the plane y = 9, centered at (2, 9, 3).
We conclude that (a), (c) and (d) parametrize the same circle whereas (b) and (e) are different curves.
Exercises
1. What is the domain of r(t ) = e
t
i +
1
t
j +(t +1)
3
k?
SOLUTION r(t ) is dened for t = 0 and t = 1, hence the domain of r(t ) is:
D = {t R : t = 0, t = 1}
What is the domain of r(s) = e
s
i +

sj +cos sk?
3. Find a vector parametrization of the line through P = (3, 5, 7) in the direction v = 3, 0, 1.
SOLUTION We use the vector parametrization of the line to obtain:
r(t ) =

OP +t v = 3, 5, 7 +t 3, 0, 1 = 3 +3t, 5, 7 +t
or in the form:
r(t ) = (3 +3t )i 5j +(7 +t )k
Find a direction vector for the line with parametrization r(t ) = (4 t )i +(2 +5t )j +
1
2
t k.
S E C T I O N 14.1 Vector-Valued Functions (ET Section 13.1) 243
5. Match the space curves in Figure 8 with their projections onto the xy-plane in Figure 9.
(A) (B) (C)
y
x
z
y
x
z
y
x
z
FIGURE 8
(i)
x
y
(ii)
x
y
(iii)
x
y
FIGURE 9
SOLUTION The projection of curve (C) onto the xy-plane is neither a segment nor a periodic wave. Hence, the correct
projection is (iii), rather than the two other graphs. The projection of curve (A) onto the xy-plane is a vertical line, hence
the corresponding projection is (ii). The projection of curve (B) onto the xy-plane is a periodic wave as illustrated in (i).
Match the space curves in Figure 8 with the following vector-valued functions:
(a) r
1
(t ) = cos 2t, cos t, sin t
(b) r
2
(t ) = t, cos 2t, sin 2t
(c) r
3
(t ) = 1, t, t
7. Match the vector-valued functions (a)(f) with the space curves (i)(vi) in Figure 10.
(a) r(t ) =
_
t +15, e
0.08t
cos t, e
0.08t
sin t
_
(b) r(t ) =
_
cos t, sin t, sin 12t
_
(c) r(t ) =
_
t, t,
25t
1 +t
2
_
(d) r(t ) =
_
cos
3
t, sin
3
t, sin 2t
_
(e) r(t ) =
_
t, t
2
, 2t
_
(f) r(t ) =
_
cos t, sin t, cos t sin 12t
_
y
(i) (ii) (iii)
(iv) (v) (vi)
x
z
y
x
z
y
x
z
y
y
x
x
z
z
y
x
z
FIGURE 10
SOLUTION
(a) (v) (b) (i) (c) (ii)
(d) (vi) (e) (iv) (f) (iii)
Which of the following curves have the same projection onto the xy-plane?
(a) r
1
(t ) =
_
t, t
2
, e
t
_
(b) r
2
(t ) =
_
e
t
, t
2
, t
_
(c) r
3
(t ) =
_
t, t
2
, cos t
_
9. Match the space curves (A)(C) in Figure 11 with their projections (i)(iii) onto the xy-plane.
y
y
x
x
(A) (B) (C)
(i) (iii) (ii)
z
y
x
z
y
x
z
z
y
x
z
y
x
z
FIGURE 11
244 C HA P T E R 14 CALCULUS OF VECTOR- VALUED FUNCTI ONS (ET CHAPTER 13)
SOLUTION Observing the curves and the projections onto the xy-plane we conclude that: Projection (i) corresponds to
curve (C); Projection (ii) corresponds to curve (A); Projection (iii) corresponds to curve (B).
In Exercises 1013, the function r(t ) traces a circle. Determine the radius, center, and plane containing the circle.
r(t ) = (9 cos t )i +(9 sin t )j
11. r(t ) = 7i +(12 cos t )j +(12 sin t )k
SOLUTION We have:
x(t ) = 7, y(t ) = 12 cos t, z(t ) = 12 sin t
Hence,
y(t )
2
+ z(t )
2
= 144 cos
2
t +144 sin
2
t = 144
_
cos
2
t +sin
2
t
_
= 144
This is the equation of a circle in the vertical plane x = 7. The circle is centered at the point (7, 0, 0) and its radius is

144 = 12.
r(t ) = sin t, 0, 4 +cos t
13. r(t ) = 6 +3 sin t, 9, 4 +3 cos t
SOLUTION Since y(t ) = 9 the curve is contained in the vertical plane y = 9. By the given equations, x(t ) = 6 +3 sin t
and z = 4 +3 cos t , hence:
_
x 6
3
_
2
+
_
z 4
3
_
2
= sin
2
t +cos
2
t = 1
We conclude that the function traces a circle in the vertical plane y = 9, centered at the point (6, 9, 4) and with radius 3.
Describe the projections of the circle r(t ) = sin t, 0, 4 +cos t onto the coordinate planes.
15. Do either of P = (4, 11, 20) or Q = (1, 6, 16) lie on the curve r(t ) =
_
1 +t, 2 +t
2
, t
4
_
?
SOLUTION The point P = (4, 11, 20) lies on the curve r(t ) =
_
1 + t, 2 + t
2
, t
4
_
if there exists a value of t such that

OP = r(t ). That is,


_
4, 11, 20
_
=
_
1 +t, 2 +t
2
, t
4
_
Equating like components we get:
1 +t = 4
2 +t
2
= 11
t
4
= 20
The rst equation implies that t = 3, but this value does not satisfy the third equation. We conclude that P does not lie
on the curve. The point Q = (1, 6, 16) lies on the curve if there exists a value of t such that:
1, 6, 16 =
_
1 +t, 2 +t
2
, t
4
_
or equivalently:
1 +t = 1
2 +t
2
= 6
t
4
= 16
These equations have the solution t = 2, hence Q = (1, 6, 16) lies on the curve.
(a) Describe the curve r(t ) = t cos t, t sin t, t and its projections onto the xy- and xz-planes.
(b) Plot r(t ) with a computer algebra system if you have one.
17. Find the points where the path r(t ) = sin t, cos t, sin t cos 2t intersects the xy-plane.
SOLUTION The curve intersects the xy-plane at the points where z = 0. That is, sin t cos 2t = 0 and so either sin t = 0
or cos 2t = 0. The solutions are, thus:
t = k or t =

4
+
k
2
, k = 0, 1, 2, . . .
The values t = k yield the points: (sin k, cos k, 0) =
_
0, (1)
k
, 0
_
. The values t =

4
+
k
2
yield the points:
k = 0 :
_
sin

4
, cos

4
, 0
_
=
_
1

2
,
1

2
, 0
_
k = 1 :
_
sin
3
4
, cos
3
4
, 0
_
=
_
1

2
,
1

2
, 0
_
k = 2 :
_
sin
5
4
, cos
5
4
, 0
_
=
_

2
,
1

2
, 0
_
S E C T I O N 14.1 Vector-Valued Functions (ET Section 13.1) 245
k = 3 :
_
sin
7
4
, cos
7
4
, 0
_
=
_

2
,
1

2
, 0
_
(Other values of k do not provide new points). We conclude that the curve intersects the xy-plane at the following points:
(0, 1, 0), (0, 1, 0),
_
1

2
,
1

2
, 0
_
,
_
1

2
,
1

2
, 0
_
,
_

2
,
1

2
, 0
_
,
_

2
,
1

2
, 0
_
Parametrize the intersection of the surfaces
y
2
z
2
= x 2, y
2
+ z
2
= 9
using t = y as the parameter (two vector functions are needed as in Example 2).
19. Find a parametrization of the curve in Exercise 18 using trigonometric functions.
SOLUTION The curve in Exercise 18 is the intersection of the surfaces y
2
z
2
= x 2, y
2
+ z
2
= 9. The circle
y
2
+ z
2
= 9 is parametrized by y = 3 cos t , z = 3 sin t . Substituting in the rst equation and using the identity
cos
2
t sin
2
t = cos 2t , gives:
x = 2 + y
2
z
2
= 2 +(3 cos t )
2
(3 sin t )
2
= 2 +9
_
cos
2
t sin
2
t
_
= 2 +9 cos 2t
We obtain the following trigonometric parametrization:
r(t ) = 2 +9 cos 2t, 3 cos t, 3 sin t
Vivianis Curve C is the intersection of the surfaces x
2
+ y
2
= z
2
, y = z
2
(Figure 12).
(a) Parametrize each of the two parts of C corresponding to x 0 and x 0 taking t = z as parameter.
(b) Describe the projection of C onto the xy-plane.
(c) Show that C lies on the sphere of radius 1 with center (0, 1, 0). This curve looks like a gure eight lying on a
sphere [Figure 12(B)].
21. Show that any point on x
2
+ y
2
= z
2
can be written in the form (z cos , z sin , z) for some . Use this to nd a
parametrization of Vivianis curve (Exercise 20) with as parameter.
SOLUTION We rst verify that x = z cos , y = z sin , and z = z satisfy the equation of the surface:
x
2
+ y
2
= z
2
cos
2
+ z
2
sin
2
= z
2
_
cos
2
+sin
2

_
= z
2
We now show that if (x, y, z) satises x
2
+ y
2
= z
2
, then there exists a value of such that x = z cos , y = z sin .
Since x
2
+ y
2
= z
2
, we have |x| |z| and |y| |z|. If z = 0, then also x = y = 0 and any value of is adequate. If
z = 0 then
x
z
1 and
y
z
1, hence there exists
0
such that
x
z
= cos
0
. Hence,
y
z
=
_
z
2
x
2
z
2
=
_
1
_
x
z
_
2
=
_
1 cos
2

0
= sin
0
If
x
z
and
y
z
are both positive, we choose
0
such that 0 <
0
<

2
. If
x
z
> 0 and
y
z
< 0 we choose
0
such that
3
2
<
0
< 2. If
x
z
< 0 and
y
z
< 0 we choose
0
such that <
0
<
3
2
, and if
x
z
< 0 and
y
z
> 0 we choose
0
such
that

2
<
0
< . In either case we can represent the points on the surface as required. Vivianis curve is the intersection
of the surfaces x
2
+ y
2
= z
2
and x = z
2
. The points on these surfaces are of the form:
x
2
+ y
2
= z
2
: (z cos , z sin , z)
x = z
2
: (z
2
, y, z)
(1)
The points (x, y, z) on the intersection curve must satisfy the following equations:
_
z
2
= z cos
y = z sin
The rst equation implies that z = 0 or z = cos . The second equation implies that y = 0 or y = cos sin =
1
2
sin 2.
The x coordinate is obtained by substituting z = cos in x = z cos (or in x = z
2
). That is, x = cos
2
. We obtain the
following vector parametrization of the curve:
r(t ) =
_
cos
2
,
1
2
sin 2, cos
_
Use sine and cosine to parametrize the intersection of the cylinders x
2
+ y
2
= 1 and x
2
+ z
2
= 1 (use two
vector-valued functions). Then describe the projections of this curve on the three coordinate planes.
23. Use sine and cosine to parametrize the intersection of the surfaces x
2
+ y
2
= 1 and z = 4x
2
, and plot this
curve using a CAS (Figure 13).
x
y
z
FIGURE 13 Intersection of the surfaces x
2
+ y
2
= 1 and z = 4x
2
.
246 C HA P T E R 14 CALCULUS OF VECTOR- VALUED FUNCTI ONS (ET CHAPTER 13)
SOLUTION The points on the cylinder x
2
+ y
2
= 1 and on the parabolic cylinder z = 4x
2
can be written in the form:
x
2
+ y
2
= 1: (cos t, sin t, z)
z = 4x
2
: (x, y, 4x
2
)
The points (x, y, z) on the intersection curve must satisfy the following equations:
x = cos t
y = sin t
z = 4x
2
x = cos t, y = sin t, z = 4 cos
2
t
We obtain the vector parametrization:
r(t ) =
_
cos t, sin t, 4 cos
2
t
_
Using the CAS we obtain the following curve:
z
2
x
y
1 1
2
4
2
1
1
r(t ) =
_
cos t, sin t, 4 cos
2
t
_
Use hyperbolic functions to parametrize the intersection of the surfaces x
2
y
2
= 4, z = xy.
In Exercises 2534, nd a parametrization of the curve.
25. The vertical line passing through the point (3, 2, 0)
SOLUTION The points of the vertical line passing through the point (3, 2, 0) can be written as (3, 2, z). Using z = t as
parameter we get the following parametrization:
r(t ) = 3, 2, t , < t <
The line passing through (1, 0, 4) and (4, 1, 2)
27. The line through the origin whose projection on the xy-plane is a line of slope 3 and on the yz-plane is a line of
slope 5 (i.e., y/z = 5)
SOLUTION We denote by (x, y, z) the points on the line. The projection of the line on the xy-plane is the line through
the origin having slope 3, that is the line y = 3x in the xy-plane. The projection of the line on the yz-plane is the line
through the origin with slope 5, that is the line z = 5y. Thus, the points on the desired line satisfy the following equalities:
y = 3x
z = 5y
y = 3x, z = 5 3x = 15x
We conclude that the points on the line are all the points in the form (x, 3x, 15x). Using x = t as parameter we obtain
the following parametrization:
r(t ) = t, 3t, 15t .
The horizontal circle of radius 1 with center (2, 1, 4)
29. The circle of radius 2 with center (1, 2, 5) in a plane parallel to the yz-plane
SOLUTION The circle is parallel to the yz-plane and centered at (1, 2, 5), hence the x-coordinates of the points on
the circle are x = 1. The projection of the circle on the yz-plane is a circle of radius 2 centered at (2, 5). This circle is
parametrized by:
y = 2 +2 cos t, z = 5 +2 sin t
We conclude that the points on the required circle can be written as (1, 2 +2 cos t, 5 +2 sin t ). This gives the following
parametrization:
r(t ) = 1, 2 +2 cos t, 5 +2 sin t .
The ellipse
_
x
2
_
2
+
_
y
3
_
2
= 1 in the xy-plane, translated to have center (9, 4, 0)
31. The intersection of the plane y =
1
2
with the sphere x
2
+ y
2
+ z
2
= 1
S E C T I O N 14.1 Vector-Valued Functions (ET Section 13.1) 247
SOLUTION Substituting y =
1
2
in the equation of the sphere gives:
x
2
+
_
1
2
_
2
+ z
2
= 1 x
2
+ z
2
=
3
4
This circle in the horizontal plane y =
1
2
has the parametrization x =

3
2
cos t , z =

3
2
sin t . Therefore, the points on
the intersection of the plane y =
1
2
and the sphere x
2
+ y
2
+z
2
= 1, can be written in the form
_
3
2
cos t,
1
2
,

3
2
sin t
_
,
yielding the following parametrization:
r(t ) =
_

3
2
cos t,
1
2
,

3
2
sin t
_
.
The intersection of the surfaces
z = x
2
y
2
and z = x
2
+ xy 1
33. The ellipse
_
x
2
_
2
+
_
z
3
_
2
= 1 in the xz-plane, translated to have center (3, 1, 5) [Figure 14(A)]
(A)
3
1
(B)
y
x
z z
y
x
3
1
FIGURE 14 The ellipses described in Exercise 33 and 34.
SOLUTION The translated ellipse is in the vertical plane y = 1, hence the y-coordinate of the points on this ellipse is
y = 1. The x and z coordinates satisfy the equation of the ellipse:
_
x 3
2
_
2
+
_
z 5
3
_
2
= 1.
This ellipse is parametrized by the following equations:
x = 3 +2 cos t, z = 5 +3 sin t.
Therefore, the points on the translated ellipse can be written as (3 +2 cos t, 1, 5 +3 sin t ). This gives the following
parametrization:
r(t ) = 3 +2 cos t, 1, 5 +3 sin t .
The ellipse
_
y
2
_
2
+
_
z
3
_
2
= 1, translated to have center (3, 1, 5) [Figure 14(B)]
In Exercises 3537, assume that two paths r
1
(t ) and r
2
(t ) intersect if there is a point P lying on both curves. We say that
r
1
(t ) and r
2
(t ) collide if r
1
(t
0
) = r
2
(t
0
) at some time t
0
.
35. Which of the following are true?
(a) If r
1
and r
2
intersect, then they collide.
(b) If r
1
and r
2
collide, then they intersect.
(c) Intersection depends only on the underlying curves traced by r
1
and r
2
but collision depends on the actual
parametrizations.
SOLUTION
(a) This statement is wrong. r
1
(t ) and r
2
(t ) may intersect but the point of intersection may correspond to different
values of the parameters in the two curves, as illustrated in the following example:
r
1
(t ) = cos t, sin t (the unit circle)
r
2
(s) = s, 1 (the horizontal line y = 1)
x
(0, 1)
y
248 C HA P T E R 14 CALCULUS OF VECTOR- VALUED FUNCTI ONS (ET CHAPTER 13)
The point of intersection (0, 1) corresponds to t =

2
and s = 0.
(b) This statement is true. If r
1
(t
0
) = r
2
(t
0
), then the head of the vector r
1
(t
0
) (or r
2
(t
0
)) is a point of intersection of
the two curves.
(c) The statement is true. Intersection is a geometric property of the curves and it is independent of the parametrization
we choose for the curves. Collision depends on the actual parametrization. Notice that if we parametrize the line y = 1
in the example given in part (a) by r
3
(s) =
_
s

2
, 1
_
, then r
1
_

2
_
= r
3
_

2
_
hence the two paths collide.
Determine whether r
1
and r
2
collide or intersect:
r
1
(t ) =
_
t
2
+3, t +1, 6t
1
_
r
2
(t ) =
_
4t, 2t 2, t
2
7
_
37. Determine whether r
1
and r
2
collide or intersect:
r
1
(t ) =
_
t, t
2
, t
3
_
, r
2
(t ) =
_
4t +6, 4t
2
, 7 t
_
SOLUTION The two paths collide if there exists a value of t such that:
_
t, t
2
, t
3
_
=
_
4t +6, 4t
2
, 7 t
_
Equating corresponding components we obtain the following equations:
t = 4t +6
t
2
= 4t
2
t
3
= 7 t
The second equation implies that t = 0, but this value does not satisfy the other equations. Therefore, the equations have
no solution, which means that the paths do not collide. The two paths intersect if there exist values of t and s such that:
_
t, t
2
, t
3
_
=
_
4s +6, 4s
2
, 7 s
_
Or equivalently:
t = 4s +6
t
2
= 4s
2
(1)
t
3
= 7 s
The second equation implies that t
1
= 2s or t
2
= 2s. Substituting t
1
= 2s and t
2
= 2s in the rst equation gives:
t
1
= 2s : 2s = 4s +6 2s = 6 s
1
= 3
t
2
= 2s : 2s = 4s +6 6s = 6 s
2
= 1
The solutions of the rst two equations are thus
(t
1
, s
1
) = (6, 3); (t
2
, s
2
) = (2, 1)
(t
1
, s
1
) does not satisfy the third equation whereas (t
2
, s
2
) does. We conclude that the equations in (1) have a solution
t = 2, s = 1, hence the two paths intersect.
Further Insights and Challenges
Sketch the curve parametrized by r(t ) = |t | +t, |t | t .
39. Find the maximum height above the xy-plane of a point on r(t ) =
_
e
t
, sin t, t (4 t )
_
.
SOLUTION The height of a point is the value of the z-coordinate of the point. Therefore we need to maximize the
function z = t (4 t ). z(t ) is a quadratic function having the roots t = 0 and t = 4, hence the maximum value is
obtained at the midpoint of the interval 0 t 4, that is at t = 2. The corresponding value of z is:
z
max
= z(2) = 2 (4 2) = 4
The point of maximum height is, thus,
(e
2
, sin 2, 4) (7.39, 0.91, 4)
Let C be the curve obtained by intersecting a cylinder of radius r and a plane. Insert two spheres of radius r
into the cylinder above and below the plane, and let F
1
and F
2
be the points where the plane is tangent to the sphere
[Figure 15(A)]. Let K be the vertical distance between the equators of the two spheres. Rediscover Archimedess
proof that C is an ellipse by showing that every point P on C satises
PF
1
+ PF
2
= K
Hint: If two lines through a point P are tangent to a sphere and intersect the sphere at Q
1
and Q
2
as in Figure 15(B),
then the segments PQ
1
and PQ
2
have equal length. Use this to show that PF
1
= PR
1
and PF
2
= PR
2
.
41. Now reprove the result of Exercise 40 using vector geometry. Assume that the cylinder has equation x
2
+
y
2
= r
2
and the plane has equation z = ax +by.
(a) Show that the upper and lower spheres in Figure 15 have centers
C
1
=
_
0, 0, r
_
a
2
+b
2
+1
_
C
2
=
_
0, 0, r
_
a
2
+b
2
+1
_
S E C T I O N 14.2 Calculus of Vector-Valued Functions (ET Section 13.2) 249
(b) Show that the points where the plane is tangent to the sphere are
F
1
=
r
_
a
2
+b
2
+1
_
a, b, a
2
+b
2
_
F
2
=
r
_
a
2
+b
2
+1
_
a, b, a
2
+b
2
_
Hint: Show that C
1
F
1
and C
2
F
2
have length r and are orthogonal to the plane.
(c) Verify, with the aid of a computer algebra system, that Eq. (2) holds with K = 2r
_
a
2
+b
2
+1. To simplify the
algebra, observe that since a and b are arbitrary, it sufces to verify Eq. (2) for the point P = (r, 0, ar).
SOLUTION
(a) and (b) Since F
1
is the tangency point of the sphere and the plane, the radius to F
1
is orthogonal to the plane.
Therefore to show that the center of the sphere is at C
1
and the tangency point is the given point we must show that:

C
1
F
1
= r (1)

C
1
F
1
is orthogonal to the plane. (2)
We compute the vector

C
1
F
1
:

C
1
F
1
=
_
ra
_
a
2
+b
2
+1
,
rb
_
a
2
+b
2
+1
,
r(a
2
+b
2
)
_
a
2
+b
2
+1
r
_
a
2
+b
2
+1
_
=
r
_
a
2
+b
2
+1
a, b, 1
Hence,

C
1
F
1
=
r
_
a
2
+b
2
+1
a, b, 1 =
r
_
a
2
+b
2
+1
_
a
2
+b
2
+(1)
2
= r
We, thus, proved that (1) is satised. To show (2) we must show that

C
1
F
1
is parallel to the normal vector a, b, 1 to
the plane z = ax + by (i.e., ax + by z = 0). The two vectors are parallel since by (1)

C
1
F
1
is a constant multiple of
a, b, 1. In a similar manner one can show (1) and (2) for the vector

C
2
F
2
.
(c) This is an extremely challenging problem. As suggested in the book, we use P = (r, 0, ar), and we also use the
expressions for F
1
and F
2
as given above. This gives us:
PF
1
=
_
_
1 +2 a
2
+b
2
2 a
_
1 +a
2
+b
2
_
r
2
PF
2
=
_
_
1 +2 a
2
+b
2
+2 a
_
1 +a
2
+b
2
_
r
2
Their sum is not very inspiring:
PF
1
+ PF
2
=
_
_
1 +2 a
2
+b
2
2 a
_
1 +a
2
+b
2
_
r
2
+
_
_
1 +2 a
2
+b
2
+2 a
_
1 +a
2
+b
2
_
r
2
Let us look, instead, at (PF
1
+ PF
2
)
2
, and show that this is equal to K
2
. Since everything is positive, this will imply
that PF
1
+ PF
2
= K, as desired.
(PF
1
+ PF
2
)
2
= 2 r
2
+4 a
2
r
2
+2 b
2
r
2
+2
_
r
4
+2b
2
r
4
+b
4
r
4
= 2 r
2
+4 a
2
r
2
+2 b
2
r
2
+2 (1 +b
2
)r
2
= 4r
2
(1 +a
2
+b
2
) = K
2
14.2 Calculus of Vector-Valued Functions (ET Section 13.2)
Preliminary Questions
1. State the three forms of the Product Rule for vector-valued functions.
SOLUTION The Product Rule for scalar multiple f (t ) of a vector-valued function r(t ) states that:
d
dt
f (t )r(t ) = f (t )r

(t ) + f

(t )r(t )
The Product Rule for dot products states that:
d
dt
r
1
(t ) r
2
(t ) = r
1
(t ) r

2
(t ) +r

1
(t ) r
2
(t )
250 C HA P T E R 14 CALCULUS OF VECTOR- VALUED FUNCTI ONS (ET CHAPTER 13)
Finally, the Product Rule for cross product is
d
dt
r
1
(t ) r
2
(t ) = r
1
(t ) r

2
(t ) +r

1
(t ) r
2
(t ).
In Questions 26, indicate whether true or false and if false, provide a correct statement.
2. The derivative of a vector-valued function is dened as the limit of the difference quotient, just as in the scalar-valued
case.
SOLUTION The statement is true. The derivative of a vector-valued function r(t ) is dened a limit of the difference
quotient:
r

(t ) = lim
t 0
r (t +h) r(t )
h
in the same way as in the scalar-valued case.
3. There are two Chain Rules for vector-valued functions, one for the composite of two vector-valued functions and
one for the composite of a vector-valued and scalar-valued function.
SOLUTION This statement is false. A vector-valued function r(t ) is a function whose domain is a set of real numbers
and whose range consists of position vectors. Therefore, if r
1
(t ) and r
2
(t ) are vector-valued functions, the composition
(r
1
r
2
)(t ) = r
1
(r
2
(t )) has no meaning since r
2
(t ) is a vector and not a real number. However, for a scalar-valued
function f (t ), the composition r( f (t )) has a meaning, and there is a Chain Rule for differentiability of this vector-valued
function.
4. The terms velocity vector and tangent vector for a path r(t ) mean one and the same thing.
SOLUTION This statement is true.
5. The derivative of a vector-valued function is the slope of the tangent line, just as in the scalar case.
SOLUTION The statement is false. The derivative of a vector-valued function is again a vector-valued function, hence
it cannot be the slope of the tangent line (which is a scalar). However, the derivative, r

(t
0
) is the direction vector of the
tangent line to the curve traced by r(t ), at r(t
0
).
6. The derivative of the cross product is the cross product of the derivatives.
SOLUTION The statement is false, since usually,
d
dt
r
1
(t ) r
2
(t ) = r

1
(t ) r

2
(t )
The correct statement is the Product Rule for Cross Products. That is,
d
dt
r
1
(t ) r
2
(t ) = r
1
(t ) r

2
(t ) +r

1
(t ) r
2
(t )
7. State whether the following derivatives of vector-valued functions r
1
(t ) and r
2
(t ) are scalars or vectors:
(a)
d
dt
r
1
(t ) (b)
d
dt
_
r
1
(t ) r
2
(t )
_
(c)
d
dt
_
r
1
(t ) r
2
(t )
_
SOLUTION (a) vector, (b) scalar, (c) vector.
Exercises
In Exercises 14, evaluate the limit.
1. lim
t 3
_
t
2
, 4t,
1
t
_
SOLUTION By the theorem on vector-valued limits we have:
lim
t 3
_
t
2
, 4t,
1
t
_
=
_
lim
t 3
t
2
, lim
t 3
4t, lim
t 3
1
t
_
=
_
9, 12,
1
3
_
.
lim
t
sin 2t i +cos t j +tan 4t k
3. lim
t 0
e
2t
i +ln(t +1)j +4k
S E C T I O N 14.2 Calculus of Vector-Valued Functions (ET Section 13.2) 251
SOLUTION Computing the limit of each component, we obtain:
lim
t 0
_
e
2t
i +ln (t +1) j +4k
_
=
_
lim
t 0
e
2t
_
i +
_
lim
t 0
ln(t +1)
_
j +
_
lim
t 0
4
_
k = e
0
i +(ln 1)j +4k = i +4k
lim
t 0
_
1
t +1
,
e
t
1
t
, 4t
_
5. Evaluate lim
h0
r(t +h) r(t )
h
for r(t ) =
_
t
1
, sin t, 4
_
.
SOLUTION This limit is the derivative
dr
dt
. Using componentwise differentiation yields:
lim
h0
r (t +h) r(t )
h
=
dr
dt
=
_
d
dt
_
t
1
_
,
d
dt
(sin t ) ,
d
dt
(4)
_
=
_

1
t
2
, cos t, 0
_
.
Evaluate lim
t 0
r(t )
t
for r(t ) = sin t, 1 cos t, 2t .
In Exercises 714, compute the derivative.
7. r(t ) =
_
t, t
2
, t
3
_
SOLUTION Using componentwise differentiation we get:
dr
dt
=
_
d
dt
(t ),
d
dt
(t
2
),
d
dt
(t
3
)
_
=
_
1, 2t, 3t
2
_
v(t ) =
_
sin 3t, cos 3t
_
9. w(s) =
_
e
s
, e
2s
_
SOLUTION Componentwise differentiation gives:
w

(s) =
__
e
s
_

,
_
e
2s
_

_
=
_
e
s
, 2e
2s
_
r() =
_
tan , 4 2, sin
_
11. r(t ) =
_
t t
1
, 4t
2
, 8
_
SOLUTION We compute the derivative of each component to obtain:
r

(t ) =
_
(t t
1
)

, (4t
2
)

, (8)

_
=
_
1 +t
2
, 8t, 0
_
c(t ) = t
1
i e
2t
k
13. a() = (cos 2)i +(sin 2)j +(sin 4)k
SOLUTION Using componentwise differentiation yields:
a

() = (cos 2)

i +(sin 2)

j +(sin 4)

k = (2 sin 2) i +(2 cos 2) j +(4 cos 4) k


b(t ) =
_
e
4t 3
, sin(t
2
), (4t +3)
1
_ 15. Calculate r

(t ) and r

(t ) for r(t ) =
_
t, t
2
, t
3
_
.
SOLUTION We perform the differentiation componentwise to obtain:
r

(t ) =
_
(t )

, (t
2
)

, (t
3
)

_
=
_
1, 2t, 3t
2
_
We now differentiate the derivative vector to nd the second derivative:
r

(t ) =
d
dt
_
1, 2t, 3t
2
_
= 0, 2, 6t .
Sketch the curve r(t ) =
_
1 t
2
, t
_
for 1 t 1. Compute the tangent vector at t = 1 and add it to the sketch.
17. Sketch the curve r
1
(t ) =
_
t, t
2
_
together with its tangent vector at t = 1. Then do the same for r
2
(t ) =
_
t
3
, t
6
_
.
SOLUTION Note that r
1

(t ) = 1, 2t and so r
1

(1) = 1, 2. The graph of r


1
(t ) satises y = x
2
. Likewise, r
2

(t ) =
_
3t
2
, 6t
5
_
and so r
2

(1) = 3, 6. The graph of r


2
(t ) also satises y = x
2
. Both graphs and tangent vectors are given
here.
2
r
2
(t )
1
r
1
(t )
Sketch the cycloid r(t ) =
_
t sin t, 1 cos t
_
together with its tangent vectors at t =

3
and
3
4
.
In Exercises 1922, use the appropriate Product Rule to evaluate the derivative, where
r
1
(t ) =
_
8t, 4, t
3
_
, r
2
(t ) =
_
0, e
t
, 6
_
252 C HA P T E R 14 CALCULUS OF VECTOR- VALUED FUNCTI ONS (ET CHAPTER 13)
19.
d
dt
_
r
1
(t ) r
2
(t )
_
SOLUTION By the Product Rule for dot products we have:
d
dt
r
1
r
2
= r
1
r
2

+r
1

r
2
We compute the derivatives of r
1
and r
2
:
r

1
=
d
dt
_
8t, 4, t
3
_
=
_
8, 0, 3t
2
_
r

2
=
d
dt
_
0, e
t
, 6
_
=
_
0, e
t
, 0
_
By (1) we have:
d
dt
r
1
(t ) r
2
(t ) =
_
8t, 4, t
3
_

_
0, e
t
, 0
_
+
_
8, 0, 3t
2
_

_
0, e
t
, 6
_
= 4e
t
+18t
2
d
dt
_
t
4
r
1
(t )
_ 21.
d
dt
_
r
1
(t ) r
2
(t )
_
SOLUTION We use the Product Rule for cross products:
d
dt
r
1
r
2
= r
1
r

2
+r

1
r
2
=
_
8t, 4, t
3
_

_
0, e
t
, 0
_
+
_
8, 0, 3t
2
_

_
0, e
t
, 6
_
=

i j k
8t 4 t
3
0 e
t
0

i j k
8 0 3t
2
0 e
t
6

= t
3
e
t
i +8t e
t
k +3t
2
e
t
i +48j +8e
t
k
= t
2
e
t
(t +3)i +48j +8e
t
(t +1)k =
_
t
2
e
t
(t +3), 48, 8e
t
(t +1)
_
d
dt
_
r
1
(t ) r
3
(t )
_

t =5
, assuming that r
3
(5) = 3, 1, 2 and r

3
(5) = 1, 2, 7.
In Exercises 2325, let
r
1
(t ) =
_
t
2
, t
3
, 4t
_
, r
2
(t ) =
_
t
1
, 1 +t, 2
_
23. Let F(t ) = r
1
(t ) r
2
(t ).
(a) Calculate F

(t ) using the Product Rule.


(b) Expand the product r
1
(t ) r
2
(t ) and differentiate. Compare with part (a).
SOLUTION
(a) By the Product Rule for dot products we have:
F

(t ) = r
1
(t ) r

2
(t ) +r

1
(t ) r
2
(t )
We compute the derivatives of r
1
(t ) and r
2
(t ):
r

1
(t ) =
d
dt
_
t
2
, t
3
, 4t
_
=
_
2t, 3t
2
, 4
_
r

2
(t ) =
d
dt
_
t
1
, 1 +t, 2
_
=
_
t
2
, 1, 0
_
Thus,
F

(t ) =
_
t
2
, t
3
, 4t
_

_
t
2
, 1, 0
_
+
_
2t, 3t
2
, 4
_

_
t
1
, 1 +t, 2
_
=
_
1 +t
3
+0
_
+
_
2 +3t
2
+3t
3
+8
_
= 4t
3
+3t
2
+9
That is,
F

(t ) = 4t
3
+3t
2
+9 (1)
(b) We now rst compute the product r
1
(t ) r
2
(t ) and then differentiate the resulting function. This gives:
F(t ) = r
1
(t ) r
2
(t ) =
_
t
2
, t
3
, 4t
_

_
t
1
, 1 +t, 2
_
= t +t
3
(1 +t ) +8t = t
4
+t
3
+9t
Differentiating F(t ) gives:
F

(t ) = 4t
3
+3t
2
+9 (2)
The derivatives in (1) and (2) are the same, as expected.
S E C T I O N 14.2 Calculus of Vector-Valued Functions (ET Section 13.2) 253
Let G(t ) = r
1
(t ) r
2
(t ).
(a) Calculate G

(t ) using the Product Rule.


(b) Expand the cross product r
1
(t ) r
2
(t ) and differentiate. Compare with part (a).
25. Find the rate of change of the angle between r
1
(t ) and r
2
(t ) at t = 2, assuming that t is measured in seconds.
SOLUTION Recall the formula for the dot product:
r
1
(t ) r
2
(t ) = r
1
(t )r
2
(t ) cos
Thus,
cos =
r
1
(t ) r
2
(t )
r
1
(t )r
2
(t )
=
t +t
3
+t
4
+8t
_
t
4
+t
6
+16t
2
_
t
2
+5 +2t +t
2
=
t
4
+t
3
+9t
_
t
2
+t
4
+16
_
1 +5t
2
+2t
3
+t
4
= (t
4
+t
3
+9t )
_
(t
2
+t
4
+16)(1 +5t
2
+2t
3
+t
4
)
_
1/2
Taking the derivative, we nd that (after a lot of work)
d
dt
cos =
144 +48 t
2
80 t
3
36 t
4
+264 t
5
6 t
6
24 t
8
+5 t
9
__
16 +t
2
+t
4
_ _
1 +5 t
2
+2 t
3
+t
4
__ 3
2
So, at t = 2, we get (after a lot of work)
d
dt
cos

t =2
=
50
159

53
On the other hand, using the chain rule,
d
dt
cos = sin
d
dt

So, we have that
d
dt
=
1
sin
d
dt
cos
Thus, at t = 2,
d
dt

t =2
=
1
sin
d
dt
cos

t =2
We need only calculate sin at t = 2. From above, we know that
cos = (t
4
+t
3
+9t )
_
(t
2
+t
4
+16)(1 +5t
2
+2t
3
+t
4
)
_
1/2
so at t = 2, cos =
7

53
. Since sin
2
+cos
2
= 1, we get that sin =
2

53
. We can now conclude that
d
dt

t =2
=

53
2

50
159

53
=
25
159
In Exercises 2629, evaluate
d
dt
r(g(t )) using the Chain Rule.
r(t ) =
_
t
2
, 2t, 4
_
, g(t ) = e
t
27. r(t ) =
_
e
t
, e
2t
, 4
_
, g(t ) = 4t +9
SOLUTION We rst differentiate the two functions:
r

(t ) =
d
dt
_
e
t
, e
2t
, 4
_
=
_
e
t
, 2e
2t
, 0
_
g

(t ) =
d
dt
(4t +9) = 4
Using the Chain Rule we get:
d
dt
r (g(t )) = g

(t )r

(g(t )) = 4
_
e
4t +9
, 2e
2(4t +9)
, 0
_
=
_
4e
4t +9
, 8e
8t +18
, 0
_
r(t ) = 4 sin 2t, 6 cos 2t , g(t ) = t
2
29. r(t ) =
_
3
t
, tan
1
t
_
, g(t ) = sin t
254 C HA P T E R 14 CALCULUS OF VECTOR- VALUED FUNCTI ONS (ET CHAPTER 13)
SOLUTION We rst compute the derivatives of the two functions:
r

(t ) =
d
dt
_
3
t
, tan
1
t
_
=
_
3
t
ln 3,
1
1 +t
2
_
g

(t ) = cos t
We now differentiate the composition function r(g(t )) using the Chain Rule:
d
dt
r(g(t )) = g

(t )r

(g(t )) = cos t
_
3
sin t
ln 3,
1
1 +sin
2
t
_
=
_
3
sin t
cos t ln 3,
cos t
1 +sin
2
t
_
Let v(s) = s
2
i +2sj +9s
2
k. Evaluate
d
ds
v(g(s)) at s = 4, assuming that g(4) = 3 and g

(4) = 9.
31. Let r(t ) =
_
t
2
, 1 t, 4t
_
. Calculate the derivative of r(t ) a(t ) at t = 2, assuming that a(2) = 1, 3, 3 and
a

(2) = 1, 4, 1.
SOLUTION By the Product Rule for dot products we have
d
dt
r(t ) a(t ) = r(t ) a

(t ) +r

(t ) a(t )
At t = 2 we have
d
dt
r(t ) a(t )

t =2
= r(2) a

(2) +r

(2) a(2) (1)


We compute the derivative r

(2):
r

(t ) =
d
dt
_
t
2
, 1 t, 4t
_
= 2t, 1, 4 r

(2) = 4, 1, 4 (2)
Also, r(2) =
_
2
2
, 1 2, 4 2
_
= 4, 1, 8. Substituting the vectors in the equation above, we obtain:
d
dt
r(t ) a(t )

t =2
= 4, 1, 8 1, 4, 1 +4, 1, 4 1, 3, 3 = (4 4 +8) +(4 3 +12) = 13
The derivative of r(t ) a(t ) at t = 2 is 13.
Let r(t ) =
_
t
2
, t
3
, e
t
_
. Use Example 4 to calculate
d
dt
(r r

).
In Exercises 3337, nd a parametrization of the tangent line at the point indicated.
33. r(t ) =
_
1 t
2
, 5t, 2t
3
_
, t = 2
SOLUTION The tangent line is parametrized by:
(t ) = r(2) +t r

(2) (1)
We compute the vectors in the above parametrization:
r(2) =
_
1 2
2
, 5 2, 2 2
3
_
= 3, 10, 16
r

(t ) =
d
dt
_
1 t
2
, 5t, 2t
3
_
=
_
2t, 5, 6t
2
_
r

(2) = 4, 5, 24
Substituting the vectors in (1) we obtain the following parametrization:
(t ) = 3, 10, 16 +t 4, 5, 24 = 3 4t, 10 +5t, 16 +24t
r(t ) =
_
cos 2t, sin 3t, sin 4t
_
, t =

4
35. r(s) = 4s
1
i 8s
3
k, s = 2
SOLUTION The tangent line has the following parametrization:
(s) = r(2) +sr

(2) (1)
We compute the vectors r(2) and r

(2):
r(2) = 4 2
1
i 8 2
3
k = 2i k
r

(s) =
d
ds
_
4s
1
i 8s
3
k
_
= 4s
2
i +24s
4
k
r

(2) = 4 2
2
i +24 2
4
k = i +
3
2
k
S E C T I O N 14.2 Calculus of Vector-Valued Functions (ET Section 13.2) 255
Substituting in (1) gives the following parametrization:
(t ) = 2i k +s
_
i +
3
2
k
_
= (2 s) i +
_
3
2
s 1
_
k
or in scalar form:
x = 2 s, y = 0, z =
3
2
s 1.
r(t ) =
_
t
2
, t
4
_
, t = 1
37. r(s) = ln si +s
1
j +9sk, s = 1
SOLUTION The tangent line has the following parametrization:
(s) = r(1) +sr

(1) (1)
We compute the vectors r(1) and r

(1):
r(1) = ln 1i +1
1
j +9 1k = j +9k
r

(s) =
d
ds
(ln si +s
1
j +9sk) =
1
s
i s
2
j +9k r

(1) = i j +9k
We substitute the vectors in (1) to obtain the following parametrization:
(s) = j +9k +s(i j +9k) = si +(1 s)j +(9 +9s)k
or in scalar form:
x = s, y = 1 s, z = 9 +9s.
Let r(t ) = sin 2t cos t, sin 2t sin t, cos 2t . Show that r(t ) is constant and conclude using Example 6 that r(t )
and r

(t ) are orthogonal. Then compute r

(t ) and verify directly that it is orthogonal to r(t ).


39. Show, by nding a counterexample, that in general r

(t ) need not equal r(t )

.
SOLUTION Let r(t ) = 1, 1, t . Then r(t ) =
_
1
2
+1
2
+t
2
=
_
2 +t
2
, hence:
r(t )

=
d
dt
__
2 +t
2
_
=
2t
2
_
2 +t
2
=
t
_
2 +t
2
On the other hand, we have r

(t ) = 0, 0, 1, hence:
r

(t ) =
_
0
2
+0
2
+1
2
= 1.
We see that r

(t ) = r(t )

.
In Exercises 4045, evaluate the integrals.
_
1
0
2t, 4t, cos 3t dt
41.
_
4
1
_
t
1
i +4

t j 8t
3/2
k
_
dt
SOLUTION We perform the integration componentwise. Computing the integral of each component we get:
_
4
1
t
1
dt = ln t

4
1
= ln 4 ln 1 = ln 4
_
4
1
4

t dt = 4
2
3
t
3/2

4
1
=
8
3
_
4
3/2
1
_
=
56
3
_
4
1
8t
3/2
dt =
16
5
t
5/2

4
1
=
16
5
_
4
5/2
1
_
=
496
5
Hence,
_
4
1
_
t
1
i +4

t j 8t
3/2
k
_
dt = (ln 4) i +
56
3
j
496
5
k
_
1
0
_
t e
t
2
, t ln(t
2
+1)
_
dt
43.
_
2
2
_
u
3
i +u
5
j +u
7
k
_
du
SOLUTION We perform componentwise integration, but before doing so we notice that u
3
, u
5
and u
7
are all odd
functions, so their integrals over this symmetric region will all be zero! Thus, the answer is 0i +0j +0k.
_
1
0
_
1
1 +s
2
,
s
1 +s
2
_
ds
256 C HA P T E R 14 CALCULUS OF VECTOR- VALUED FUNCTI ONS (ET CHAPTER 13)
45.
_
t
0
_
3si +6s
2
j +9k
_
ds
SOLUTION We rst compute the integral of each component:
_
t
0
3s ds =
3
2
s
2

t
0
=
3
2
t
2
_
t
0
6s
2
ds =
6
3
s
3

t
0
= 2t
3
_
t
0
9 ds = 9s

t
0
= 9t
Hence,
_
t
0
_
3si +6s
2
j +9k
_
dt =
__
t
0
3s ds
_
i +
__
t
0
6s
2
ds
_
j +
__
t
0
9 ds
_
k =
_
3
2
t
2
_
i +
_
2t
3
_
j +(9t )k
In Exercises 4653, nd the general solution r(t ) of the differential equation and the solution with the given initial
condition.
dr
dt
= 1 2t, 4t , r(0) = 3, 1
47. r

(t ) = i j, r(0) = 2i +3k
SOLUTION The general solution is obtained by integrating r

(t ):
r(t ) =
_
(i j) dt =
__
1 dt
_
i
__
1 dt
_
j = t i t j +c (1)
Hence,
r(0) = 0i 0j +c = c
The solution with the initial condition r(0) = 2i +3k must satisfy:
r(0) = c = 2i +3k
Substituting in (1) yields the solution:
r(t ) = t i t j +2i +3k = (t +2) i t j +3k
r

(t ) = t
2
i +5t j +k, r(0) = j +2k
49. r

(t ) = sin 3t, sin 3t, t , r(0) = 0, 1, 8


SOLUTION We rst integrate the vector r

(t ) to nd the general solution:


r(t ) =
_
sin 3t, sin 3t, t dt =
__
sin 3t dt,
_
sin 3t dt,
_
t dt
_
=
_

1
3
cos 3t,
1
3
cos 3t,
1
2
t
2
_
+c (1)
Substituting the initial condition we obtain:
r(0) =
_

1
3
cos 0,
1
3
cos 0,
1
2
0
2
_
+c = 0, 1, 8 =
_

1
3
,
1
3
, 0
_
+c = 0, 1, 8
Hence,
c = 0, 1, 8
_

1
3
,
1
3
, 0
_
=
_
1
3
,
4
3
, 8
_
Substituting in (1) we obtain the solution:
r(t ) =
_

1
3
cos 3t,
1
3
cos 3t,
1
2
t
2
_
+
_
1
3
,
4
3
, 8
_
=
_
1
3
(1 cos 3t ) ,
1
3
(4 cos 3t ) , 8 +
1
2
t
2
_
dr
dt
=
_
e
2t
, e
t
, e
2t
_
, r(0) = 4, 2, 3
51. r

(t ) = 16k, r(0) = 1, 0, 0, r

(0) = 0, 1, 0
SOLUTION To nd the general solution we rst nd r

(t ) by integrating r

(t ):
r

(t ) =
_
r

(t ) dt =
_
16kdt = (16t ) k +c
1
(1)
S E C T I O N 14.2 Calculus of Vector-Valued Functions (ET Section 13.2) 257
We now integrate r

(t ) to nd the general solution r(t ):


r(t ) =
_
r

(t ) dt =
_
((16t ) k +c
1
) dt =
__
16(t ) dt
_
k +c
1
t +c
2
= (8t
2
)k +c
1
t +c
2
(2)
We substitute the initial conditions in (1) and (2). This gives:
r

(0) = c
1
= 0, 1, 0 = j
r(0) = 0k +c
1
0 +c
2
= 1, 0, 0 c
2
= 1, 0, 0 = i
Combining with (2) we obtain the following solution:
r(t ) = (8t
2
)k +t j +i = i +t j +(8t
2
)k
r

(t ) = 0, 0, 1, r(0) = 2, 1, 1, r

(0) = 3, 1, 1
53. r

(t ) =
_
e
t
, sin t, cos t
_
, r(0) = 1, 0, 1, r

(0) = 0, 2, 2
SOLUTION We perform integration componentwise on r

(t ) to obtain:
r

(t ) =
_
_
e
t
, sin t, cos t
_
dt =
_
e
t
, cos t, sin t
_
+c
1
(1)
We now integrate r

(t ) to obtain the general solution:


r(t ) =
_
__
e
t
, cos t, sin t
_
+c
1
_
dt =
_
e
t
, sin t, cos t
_
+c
1
t +c
2
(2)
Now, we substitute the initial conditions r(0) = 1, 0, 1 and r

(0) = 0, 2, 2 into (1) and (2) and solve for the vectors
c
1
and c
2
. We obtain:
r

(0) = 1, 1, 0 +c
1
= 0, 2, 2 c
1
= 1, 3, 2
r(0) = 1, 0, 1 +c
2
= 1, 0, 1 c
2
= 0, 0, 2
Finally we combine the above to obtain the solution:
r(t ) =
_
e
t
, sin t, cos t
_
+1, 3, 2 t +0, 0, 2 =
_
e
t
t, sin t +3t, cos t +2t +2
_
Show that w(t ) = sin(3t +4), sin(3t 2), cos 3t satises the differential equation w

(t ) = 9w(t ).
55. The path r(t ) of a particle satises
dr
dt
=
_
8, 5 3t, 4t
2
_
. Where is the particle located at t = 4 if r(0) = 1, 6, 0?
SOLUTION We rst nd the general solution by integrating the vector
dr
dt
componentwise. This gives:
r(t ) =
_
_
8, 5 3t, 4t
2
_
dt =
_
8t, 5t
3
2
t
2
,
4
3
t
3
_
+c (1)
Substituting t = 0 we get:
r(0) =
_
8 0, 5 0
3
2
0
2
,
4
3
0
3
_
+c = c
The initial condition r(0) = 1, 6, 0 gives c = 1, 6, 0. Combining with (1) we obtain the following solution:
r(t ) =
_
8t, 5t
3
2
t
2
,
4
3
t
3
_
+1, 6, 0 =
_
1 +8t, 6 +5t
3
2
t
2
,
4
3
t
3
_
To nd the particles position at t = 4, we substitute t = 4 in r(t ) obtaining:
r(4) =
_
1 +8 4, 6 +5 4
3
2
4
2
,
4
3
4
3
_
=
_
33, 2,
256
3
_
A ghter plane, which can only shoot bullets straight ahead, travels along the path r(t ) =
_
5 t, 21 t
2
, 3
t
3
/27
_
. Show that there is precisely one time t at which the pilot can hit a target located at the origin.
57. Find all solutions to r

(t ) = v, where v is a constant vector in R


3
.
SOLUTION We denote the components of the constant vector v by v = v
1
, v
2
, v
3
and integrate to nd the general
solution. This gives:
r(t ) =
_
v dt =
_
v
1
, v
2
, v
3
dt =
__
v
1
dt,
_
v
2
dt,
_
v
3
dt
_
= v
1
t +c
1
, v
2
t +c
2
, v
3
t +c
3
= t v
1
, v
2
, v
3
+c
1
, c
2
, c
3

We let c = c
1
, c
2
, c
3
and obtain:
r(t ) = t v +c = c +t v
Notice that the solutions are the vector parametrizations of all the lines with direction vector v.
258 C HA P T E R 14 CALCULUS OF VECTOR- VALUED FUNCTI ONS (ET CHAPTER 13)
Let u be a constant vector in R
3
. Find the solution of r

(t ) = (sin t )u satisfying r

(0) = 0.
59. Find all solutions to r

(t ) = 2r(t ) where r(t ) is a vector-valued function in three-space.


SOLUTION We denote the components of r(t ) by r(t ) = x(t ), y(t ), z(t ). Then, r

(t ) =
_
x

(t ), y

(t ), z

(t )
_
. Substi-
tuting in the differential equation we get:
_
x

(t ), y

(t ), z

(t )
_
= 2 x(t ), y(t ), z(t )
Equating corresponding components gives:
x

(t ) = 2x(t )
y

(t ) = 2y(t )
z

(t ) = 2z(t )

x(t ) = c
1
e
2t
y(t ) = c
2
e
2t
z(t ) = c
3
e
2t
We denote the constant vector by c = c
1
, c
2
, c
3
and obtain the following solutions:
r(t ) =
_
c
1
e
2t
, c
2
e
2t
, c
3
e
2t
_
= e
2t
c
1
, c
2
, c
3
= e
2t
c
Show that
d
dt
(a r) = a r

for any constant vector a.


61. Prove that r(t
0
) and r

(t
0
) are orthogonal at values t = t
0
where r(t ) takes on a local minimum or
maximum value. Explain how this result is related to Figure 7. Hint: In the gure, r(t
0
) is a minimum and the path
r(t ) intersects the sphere of radius r(t
0
) in a single point (and hence is tangent at that point).
z
y
x
r(t
0
)
r(t
0
)
r(t)
FIGURE 7
SOLUTION Suppose that r(t ) takes on a minimum or maximum value at t = t
0
. Hence, r(t )
2
also takes on a
minimum or maximum value at t = t
0
, therefore
d
dt
r(t )
2

t =t
0
= 0. Using the Product Rule for dot products we get
d
dt
r(t )
2

t =t
0
=
d
dt
r(t ) r(t )

t =t
0
= r(t
0
) r

(t
0
) +r

(t
0
) r(t
0
) = 2r(t
0
) r

(t
0
) = 0
Thus r(t
0
) r

(t
0
) = 0, which implies the orthogonality of r(t
0
) and r

(t
0
). In Figure 7, r(t
0
) is a minimum and the
path intersects the sphere of radius r(t
0
) at a single point. Therefore, the point of intersection is a tangency point which
implies that r

(t
0
) is tangent to the sphere at t
0
. We conclude that r(t
0
) and r

(t
0
) are orthogonal.
Newtons Second Law of Motion in vector form states that F =
dp
dt
where F is the force acting on an object of
mass m and p = mr

(t ) is the objects momentum. The analogs of force and momentum for rotational motion are
the torque = r F and angular momentum J = r(t ) p(t ). Use the Second Law to prove that =
dJ
dt
. This is
a rotational version of the Second Law.
Further Insights and Challenges
63. In this exercise, we verify that the denition of the tangent line using vector-valued functions agrees with the usual
denition in terms of the scalar derivative in the case of a plane curve. Suppose that r(t ) = x(t ), y(t ) traces a plane
curve C.
(a) Show that
dy
dx
=
y

(t )
x

(t )
at any point such that x

(t ) = 0. Hint: By the Chain Rule,


dy
dt
=
dy
dx
dx
dt
.
(b) Show that if x

(t
0
) = 0, then the line L(t ) = r(t
0
) +t r

(t
0
) passes through r(t
0
) and has slope
dy
dx

t =t
0
.
SOLUTION
(a) By the Chain Rule we have
dy
dt
=
dy
dx

dx
dt
Hence, at the points where
dx
dt
= 0 we have:
dy
dx
=
dy
dt
dx
dt
=
y

(t )
x

(t )
(b) The line (t ) = a, b +t r

(t
0
) passes through (a, b) at t = 0. It holds that:
(0) = a, b +0r

(t
0
) = a, b
That is, (a, b) is the terminal point of the vector (0), hence the line passes through (a, b). The line has the direction
vector r

(t
0
) =
_
x

(t
0
), y

(t
0
)
_
, therefore the slope of the line is
y

(t
0
)
x

(t
0
)
which is equal to
dy
dx

t =t
0
by part (a).
S E C T I O N 14.2 Calculus of Vector-Valued Functions (ET Section 13.2) 259
Verify the Sum and Product Rules for derivatives of vector-valued functions.
65. Verify the Chain Rule for vector-valued functions.
SOLUTION Let g(t ) and r(t ) = x(t ), y(t ), z(t ) be differentiable scalar and vector valued functions respectively. We
must show that:
d
dt
r (g(t )) = g

(t )r

(g(t )) .
We have
r (g(t )) = x (g(t )) , y (g(t )) , z (g(t ))
We differentiate the vector componentwise, using the Chain Rule for scalar functions. This gives:
d
dt
r (g(t )) =
_
d
dt
(x (g(t ))) ,
d
dt
(y (g(t ))) ,
d
dt
(z (g(t )))
_
=
_
g

(t )x

(g(t )) , g

(t )y

(g(t )) , g

(t )z

(g(t ))
_
= g

(t )
_
x

(g(t )) , y

(g(t )) , z

(g(t ))
_
= g

(t )r

(g(t ))
Verify the Product Rule for cross products [Eq. (6)]. 67. Prove that
d
dt
(r (r

)) = r (r

)
SOLUTION We use the Product Rule for dot products to obtain:
d
dt
_
r
_
r

__
= r
d
dt
_
r

_
+r


_
r

_
(1)
By the Product Rule for cross products and properties of cross products, we have:
d
dt
_
r

_
= r

+r

= r

+0 = r

(2)
Substituting (2) into (1) yields:
d
dt
_
r
_
r

__
= r
_
r

_
+r


_
r

_
(3)
Since r

is orthogonal to r

, the dot product r


_
r

_
= 0. So (3) gives:
d
dt
_
r
_
r

__
= r
_
r

_
+0 = r
_
r

_
Exercises 6871 establish additional properties of vector-valued integrals. Assume that all functions are integrable.
Prove the linearity properties
_
cr(t ) dt = c
_
r(t ) dt (c any constant)
_
_
r
1
(t ) +r
2
(t )
_
dt =
_
r
1
(t ) dt +
_
r
2
(t ) dt
69. Prove the Substitution Rule [where g(t ) is a differentiable scalar function]:
_
b
a
r(g(t ))g

(t ) dt =
_
g
1
(b)
g
1
(a)
r(u) du
SOLUTION (Note that an early edition of the textbook had the integral limits as g(a) and g(b); they should actually
be g
1
(a) and g
1
(b).) We denote the components of the vector-valued function by r(t ) dt = x(t ), y(t ), z(t ). Using
componentwise integration we have:
_
b
a
r(t ) dt =
_
_
b
a
x(t ) dt,
_
b
a
y(t ) dt,
_
b
a
z(t ) dt
_
Write
_
b
a
x(t ) dt as
_
b
a
x(s) ds. Let s = g(t ), so ds = g

(t ) dt . The substitution gives us


_
g
1
(b)
g
1
(a)
x(g(t ))g

(t ) dt . A
similar procedure for the other two integrals gives us:
_
b
a
r(t ) dt =
_
_
g
1
(b)
g
1
(a)
x (g(t )) g

(t ) dt,
_
g
1
(b)
g
1
(a)
y (g(t )) g

(t ) dt,
_
g
1
(b)
g
1
(a)
z (g(t )) g

(t ) dt
_
=
_
g
1
(b)
g
1
(a)
_
x (g(t )) g

(t ), y (g(t )) g

(t ), z (g(t )) g

(t )
_
dt
=
_
g
1
(b)
g
1
(a)
x (g(t )) , y (g(t )) , z (g(t )) g

(t ) dt =
_
g
1
(b)
g
1
(a)
r (g(t )) g

(t ) dt
Formulate and verify a version of Integration by Parts for vector-valued integrals.
260 C HA P T E R 14 CALCULUS OF VECTOR- VALUED FUNCTI ONS (ET CHAPTER 13)
71. Show that if r(t ) K for t [a, b], then
_
_
_
_
_
_
b
a
r(t ) dt
_
_
_
_
_
K(b a)
SOLUTION Think of r(t ) as a velocity vector. Then,
_
b
a
r(t ) dt gives the displacement vector from the location at time
t = a to the time t = b, and so
_
_
_
_
_
_
b
a
r(t ) dt
_
_
_
_
_
gives the length of this displacement vector. But, since speed is r(t )
which is less than or equal to K, then in the interval a t b, the object can move a total distance not more than
K(b a). Thus, the length of the displacement vector is K(b a), which gives us
_
_
_
_
_
_
b
a
r(t ) dt
_
_
_
_
_
K(b a), as
desired.
14.3 Arc Length and Speed (ET Section 13.3)
Preliminary Questions
1. At a given instant, a car on a roller coaster has velocity vector r

= 25, 35, 10 (in miles per hour). What would


the velocity vector be if the speed were doubled? What would it be if the cars direction were reversed but its speed
remained unchanged?
SOLUTION The speed is doubled but the direction is unchanged, hence the new velocity vector has the form:
r

= 25, 35, 10 for > 0


We use = 2, and so the new velocity vector is 50, 70, 20. If the direction is reversed but the speed is unchanged,
the new velocity vector is:
r

= 25, 35, 10 .
2. Two cars travel in the same direction along the same roller coaster (at different times). Which of the following
statements about their velocity vectors at a given point P on the roller coaster are true?
(a) The velocity vectors are identical.
(b) The velocity vectors point in the same direction but may have different lengths.
(c) The velocity vectors may point in opposite directions.
SOLUTION
(a) The length of the velocity vector is the speed of the particle. Therefore, if the speeds of the cars are different the
velocities are not identical. The statement is false.
(b) The velocity vector is tangent to the curve. Since the cars travel in the same direction, their velocity vectors point in
the same direction. The statement is true.
(c) Since the cars travel in the same direction, the velocity vectors point in the same direction. The statement is false.
3. A mosquito ies along a parabola with speed v(t ) = t
2
. Let L(t ) be the total distance traveled at time t .
(a) How fast is L(t ) changing at t = 2?
(b) Is L(t ) equal to the mosquitos distance from the origin?
SOLUTION
(a) By the Arc Length Formula, we have:
L(t ) =
_
t
t
0
r

(t ) dt =
_
t
t
0
v(t ) dt
Therefore,
L

(t ) = v(t )
To nd the rate of change of L(t ) at t = 2 we compute the derivative of L(t ) at t = 2, that is,
L

(2) = v(2) = 2
2
= 4
(b) L(t ) is the distance along the path traveled by the mosquito. This distance is usually different from the mosquitos
distance from the origin, which is the length of r(t ).
S E C T I O N 14.3 Arc Length and Speed (ET Section 13.3) 261
r(t )
Distance L(t)
Distance from
the origin
t
0
t
4. What is the length of the path traced by r(t ) for 4 t 10 if r(t ) is an arc length parametrization?
SOLUTION Since r(t ) is an arc length parametrization, the length of the path for 4 t 10 is equal to the length of
the time interval 4 t 10, which is 6.
Exercises
In Exercises 16, compute the length of the curve over the given interval.
1. r(t ) = 3t, 4t 3, 6t +1, 0 t 3
SOLUTION We have x(t ) = 3t , y(t ) = 4t 3, z(t ) = 6t +1 hence
x

(t ) = 3, y

(t ) = 4, z

(t ) = 6.
We use the Arc Length Formula to obtain:
L =
_
3
0
r

(t ) dt =
_
3
0
_
x

(t )
2
+ y

(t )
2
+ z

(t )
2
dt =
_
3
0
_
3
2
+4
2
+6
2
dt = 3

61
r(t ) = 2t i 3t k, 11 t 15
3. r(t ) =
_
2t, ln t, t
2
_
, 1 t 4
SOLUTION The derivative of r(t ) is r

(t ) =
_
2,
1
t
, 2t
_
. We use the Arc Length Formula to obtain:
L =
_
4
1
r

(t ) dt =
_
4
1
_
2
2
+
_
1
t
_
2
+(2t )
2
dt =
_
4
1
_
4t
2
+4 +
1
t
2
dt =
_
4
1
_
_
2t +
1
t
_
2
dt
=
_
4
1
_
2t +
1
t
_
dt = t
2
+ln t

4
1
= (16 +ln 4) (1 +ln 1) = 15 +ln 4
r(t ) =
_
2t
2
+1, 2t
2
1, t
3
_
, 0 t 4
5. r(t ) = t i +2t j +(t
2
3)k, 0 t 2. Hint:
_
_
t
2
+a
2
dt =
1
2
t
_
t
2
+a
2
+
1
2
a
2
ln
_
t +
_
t
2
+a
2
_
SOLUTION The derivative of r(t ) is r

(t ) = i +2j +2t k. Using the Arc Length Formula we get:


L =
_
2
0
r

(t ) dt =
_
2
0
_
1
2
+(2)
2
+(2t )
2
dt =
_
2
0
_
4t
2
+5 dt
We substitute u = 2t , du = 2 dt and use the given integration formula. This gives:
L =
1
2
_
4
0
_
u
2
+5 du =
1
4
u
_
u
2
+5 +
1
4
5 ln
_
u +
_
u
2
+5
_

4
0
=
1
4
4
_
4
2
+5 +
5
4
ln
_
4 +
_
4
2
+5
_

5
4
ln

5 =

21 +
5
4
ln
_
4 +

21
_

5
4
ln

5
=

21 +
5
4
ln
4 +

21

5
6.26
r(t ) = t cos t, t sin t, 3t , 0 t 2 7. Compute s(t ) =
_
t
0
r

(u) du for r(t ) =


_
t
2
, 2t
2
, t
3
_
.
SOLUTION The derivative of r(t ) is r

(t ) =
_
2t, 4t, 3t
2
_
. Hence,
r

(t ) =
_
(2t )
2
+(4t )
2
+(3t
2
)
2
=
_
4t
2
+16t
2
+9t
4
=
_
20 +9t
2
t
262 C HA P T E R 14 CALCULUS OF VECTOR- VALUED FUNCTI ONS (ET CHAPTER 13)
Hence,
s(t ) =
_
t
0
r

(u) du =
_
t
0
_
20 +9u
2
u du
We compute the integral using the substitution v = 20 +9u
2
, dv = 18u du. This gives:
s(t ) =
1
18
_
20+9t
2
20
v
1/2
dv =
1
18

2
3
v
3/2

20+9t
2
20
=
1
27
_
(20 +9t
2
)
3/2
20
3/2
_
.
In Exercises 811, nd the speed at the given value of t .
r(t ) = 2t +3, 4t 3, 5 t , t = 4
9. r(t ) =
_
e
t 3
, 12, 3t
1
_
, t = 3
SOLUTION The velocity vector is r

(t ) =
_
e
t 3
, 0, 3t
2
_
and at t = 3, r

(3) =
_
e
33
, 0, 3 3
2
_
=
_
1, 0,
1
3
_
.
The speed is the magnitude of the velocity vector, that is,
v(3) = r

(3) =
_
1
2
+0
2
+
_

1
3
_
2
=
_
10
9
1.05
r(t ) = sin 3t, cos 4t, cos 5t , t =

2
11. r(t ) = cosh t, sinh t, t , t = 0
SOLUTION The velocity vector is r

(t ) = sinh t, cosh t, 1. At t = 0 the velocity is r

(0) = sinh(0), cosh(0), 1 =


0, 1, 1, hence the speed is
v(0) = r

(0) =
_
0
2
+1
2
+1
2
=

2.
What is the velocity vector of a particle traveling to the right along the hyperbola y = x
1
with constant speed
5 cm/s when the particles location is (2,
1
2
)?
13. A bee with velocity vector r

(t ) starts out at the origin at t = 0 and ies around for T seconds. Where is the bee
located at time T if
_
T
0
r

(u) du = 0? What does the quantity


_
T
0
r

(u) du represent?
SOLUTION By the Fundamental Theorem for vector-valued functions,
_
T
0
r

(u) du = r(T) r(0), hence by the


given information r(T) = r(0). It follows that at time T the bee is located at the starting point which is at the origin. The
integral
_
T
0
r

(u) du is the length of the path traveled by the bee in the time interval 0 t T. Notice that there is a
difference between the displacement and the actual length traveled.
Which of the following is an arc length parametrization of a circle of radius 4 centered at the origin?
(a) r
1
(t ) = 4 sin t, 4 cos t
(b) r
2
(t ) = 4 sin 4t, 4 cos 4t
(c) r
3
(t ) =
_
4 sin
t
4
, 4 cos
t
4
_
15. Let r(t ) = 3t +1, 4t 5, 2t .
(a) Calculate s(t ) =
_
t
0
r

(u) du as a function of t .
(b) Find the inverse (s) = t (s) and show that r
1
(s) = r((s)) is an arc length parametrization.
SOLUTION
(a) We differentiate r(t ) componentwise and then compute the norm of the derivative vector. This gives:
r

(t ) = 3, 4, 2
r

(t ) =
_
3
2
+4
2
+2
2
=

29
We compute s(t ):
s(t ) =
_
t
0
r

(u) du =
_
t
0

29 du =

29 u

t
0
=

29t
(b) We nd the inverse (s) = t (s) by solving s =

29t for t . We obtain:
s =

29t t = (s) =
s

29
We obtain the following arc length parametrization:
r
1
(s) = r
_
s

29
_
=
_
3s

29
+1,
4s

29
5,
2s

29
_
To verify that r
1
(s) is an arc length parametrization we must show that r

1
(s) = 1. We compute r

1
(s):
r

1
(s) =
d
ds
_
3s

29
+1,
4s

29
5,
2s

29
_
=
_
3

29
,
4

29
,
2

29
_
=
1

29
3, 4, 2
Thus,
r

1
(s) =
1

29
3, 4, 2 =
1

29
_
3
2
+4
2
+2
2
=
1

29

29 = 1
S E C T I O N 14.3 Arc Length and Speed (ET Section 13.3) 263
Find an arc length parametrization of the circle in the plane z = 9 with radius 4 and center (1, 4, 9).
17. Find a path that traces the circle in the plane y = 10 with radius 4 and center (2, 10, 3) with constant speed 8.
SOLUTION We start with the following parametrization of the circle:
r(t ) = 2, 10, 3 +4 cos t, 0, sin t = 2 +4 cos t, 10, 3 +4 sin t
We need to reparametrize the curve by making a substitution t = (s), so that the new parametrization r
1
(s) = r
_
(s)
_
satises r

1
(s) = 8 for all s. We nd r

1
(s) using the Chain Rule:
r

1
(s) =
d
ds
r
_
(s)
_
=

(s)r

_
(s)
_
(1)
Next, we differentiate r(t ) and then replace t by (s):
r

(t ) = 4 sin t, 0, 4 cos t
r

_
(s)
_
=
_
4 sin (s), 0, 4 cos (s)
_
Substituting in (1) we get:
r

1
(s) =

(s)
_
4 sin (s), 0, 4 cos (s)
_
= 4

(s)
_
sin (s), 0, cos (s)
_
Hence,
r

1
(s) = 4|

(s)|
_
_
sin (s)
_
2
+
_
cos (s)
_
2
= 4|

(s)|
To satisfy r

1
(s) = 8 for all s, we choose

(s) = 2. We may take the antiderivative (s) = 2 s, and obtain the


following parametrization:
r
1
(s) = r
_
(s)
_
= r(2s) = 2 +4 cos(2s), 10, 3 +4 sin(2s) .
This is a parametrization of the given circle, with constant speed 8.
Show that one arch of the cycloid r(t ) = t sin t, 1 cos t has length 8. Find the value of t in [0, 2] where
the speed is at a maximum.
19. Find an arc length parametrization of r(t ) =
_
e
t
sin t, e
t
cos t, e
t
_
.
SOLUTION An arc length parametrization is r
1
(s) = r
_
(s)
_
where t = (s) is the inverse of the arc length function.
We compute the arc length function:
s(t ) =
_
t
0
r

(u) du (1)
Differentiating r(t ) and computing the norm of r

(t ) gives:
r

(t ) =
_
e
t
sin t +e
t
cos t, e
t
cos t e
t
sin t, e
t
_
= e
t
sin t +cos t, cos t sin t, 1
r

(t ) = e
t
_
(sin t +cos t )
2
+(cos t sin t )
2
+1
2
= e
t
(sin
2
t +2 sin t cos t +cos
2
t +cos
2
t 2 sin t cos t +sin
2
t +1)
1/2
= e
t
_
2(sin
2
t +cos
2
t ) +1 = e
t

2 1 +1 =

3 e
t
(2)
Substituting (2) into (1) gives:
s(t ) =
_
t
0

3 e
u
du =

3 e
u

t
0
=

3(e
t
e
0
) =

3(e
t
1)
We nd the inverse function of s(t ) by solving s =

3
_
e
t
1
_
for t . We obtain:
s =

3(e
t
1)
s

3
= e
t
1
e
t
= 1 +
s

3
t = (s) = ln
_
1 +
s

3
_
An arc length parametrization for r
1
(s) = r
_
(s)
_
is:
_
e
ln(1+s/(

3))
sin
_
ln
_
1 +
s

3
__
, e
ln(1+s/(

3))
cos
_
ln
_
1 +
s

3
__
, e
ln(1+s/(

3))
_
=
_
1 +
s

3
__
sin
_
ln
_
1 +
s

3
__
, cos
_
ln
_
1 +
s

3
__
, 1
_
264 C HA P T E R 14 CALCULUS OF VECTOR- VALUED FUNCTI ONS (ET CHAPTER 13)
Find an arc length parametrization of r(t ) =
_
t
2
, t
3
_
.
21. Express the arc length L of y = x
3
for 0 x 8 as an integral in two ways, using the parametrizations r
1
(t ) =
_
t, t
3
_
and r
2
(t ) =
_
t
3
, t
9
_
. Do not evaluate the integrals, but use substitution to show that they yield the same result.
SOLUTION For r
1
(t ) =
_
t, t
3
_
we have r

1
(t ) =
_
1, 3t
2
_
hence r

1
(t ) =
_
1 +9t
4
. For r
2
(t ) =
_
t
3
, t
9
_
we have
r

2
(t ) =
_
3t
2
, 9t
8
_
hence r

2
(t ) =
_
9t
4
+81t
16
. The length L may be computed using the two parametrizations by
the following integrals (notice that in the second parametrization 0 t
3
8 hence 0 t 2).
L =
_
8
0
r

1
(t ) dt =
_
8
0
_
1 +9t
4
dt (1)
L =
_
2
0
r

2
(t ) dt =
_
2
0
_
1 +9t
12
3t
2
dt (2)
We use the substitution u = t
3
, du = 3t
2
dt in the second integral to obtain:
_
2
0
_
1 +9t
12
3t
2
dt =
_
8
0
_
1 +9u
4
du
This integral is the same as the integral in (1), in accordance with the well known property: the arc length is independent
of the parametrization we choose for the curve.
Show that a helix of radius R and height h making N complete turns has the parametrization
_
R cos
_
2Nt
h
_
, R sin
_
2Nt
h
_
, t
_
, 0 t h
23. Consider the two springs in Figure 5. One has radius 5 cm, height 4 cm, and makes three complete turns. The other
has height 3 cm, radius 4 cm, and makes ve complete turns.
(a) Take a guess as to which spring uses more wire.
(b) Compute the lengths of the two springs (use Exercise 22) and compare.
4 cm
3 turns, radius 5 cm 5 turns, radius 4 cm
3 cm
FIGURE 5 Which spring uses more wire?
SOLUTION
(a) The second wire seems to use more wire than the rst one.
(b) Setting R = 5, h = 4 and N = 3 in the parametrization in Exercise 22 gives:
r
1
(t ) =
_
5 cos
2 3t
4
, 5 sin
2 3t
4
, t
_
=
_
5 cos
3t
2
, 5 sin
3t
2
, t
_
, 0 t 4
Setting R = 4, h = 3 and N = 5 in this parametrization we get:
r
2
(t ) =
_
4 cos
2 5t
3
, 4 sin
2 5t
3
, t
_
=
_
4 cos
10t
3
, 4 sin
10t
3
, t
_
, 0 t 3
We nd the derivatives of the two vectors and their lengths:
r

1
(t ) =
_

15
2
sin
3t
2
,
15
2
cos
3t
2
, 1
_
r

1
(t ) =
_
225
2
4
+1 =
1
2
_
225
2
+4
r

2
(t ) =
_

40
3
sin
10t
3
,
40
3
cos
10t
3
, 1
_
r

2
(t ) =
_
1,600
2
9
+1 =
1
3
_
1,600
2
+9
Using the Arc Length Formula we obtain the following lengths:
L
1
=
_
4
0
1
2
_
225
2
+4 dt = 2
_
225
2
+4 94.3
L
2
=
_
3
0
1
3
_
1,600
2
+9 dt =
_
1,600
2
+9 125.7
We see that the second spring uses more wire than the rst one.
Use Exercise 22 to nd a general formula for the length of a helix of radius R and height h that makes N complete
turns.
25. Evaluate s(t ) =
_
t

(u) du for the Bernoulli spiral r(t ) =


_
e
t
cos 4t, e
t
sin 4t
_
(Figure 6). It is convenient to
take as the lower limit since s() = 0. Then:
S E C T I O N 14.3 Arc Length and Speed (ET Section 13.3) 265
(a) Use s to obtain an arc length parametrization of r(t ).
(b) Prove that the angle between the position vector and the tangent vector is constant.
t = 0
t = 2
10 20
x
y
FIGURE 6 Bernoulli spiral.
SOLUTION
(a) We differentiate r(t ) and compute the norm of the derivative vector. This gives:
r

(t ) =
_
e
t
cos 4t 4e
t
sin 4t, e
t
sin 4t +4e
t
cos 4t
_
= e
t
cos 4t 4 sin 4t, sin 4t +4 cos 4t
r

(t ) = e
t
_
(cos 4t 4 sin 4t )
2
+(sin 4t +4 cos 4t )
2
= e
t
_
cos
2
4t 8 cos 4t sin 4t +16 sin
2
4t +sin
2
4t +8 sin 4t cos 4t +16 cos
2
4t
_
1/2
= e
t
_
cos
2
4t +sin
2
4t +16
_
sin
2
4t +cos
2
4t
_
= e
t

1 +16 1 =

17e
t
We now evaluate the improper integral:
s(t ) =
_
t

(u) du = lim
R
_
t
R

17e
u
du = lim
R

17e
u

t
R
= lim
R

17(e
t
e
R
)
=

17(e
t
0) =

17e
t
An arc length parametrization of r(t ) is r
1
(s) = r
_
(s)
_
where t = (s) is the inverse function of s(t ). We nd t = (s)
by solving s =

17e
t
for t :
s =

17e
t
e
t
=
s

17
t = (s) = ln
s

17
An arc length parametrization of r(t ) is:
r
1
(s) = r
_
(s)
_
=
_
e
ln(s/(

17))
cos
_
4 ln
s

17
_
, e
ln(s/(

17))
sin
_
4 ln
s

17
__
=
s

17
_
cos
_
4 ln
s

17
_
, sin
_
4 ln
s

17
__
(1)
(b) The cosine of the angle between the position vector r
1
(s) and the tangent vector r

1
(s) is:
cos =
r
1
(s) r

1
(s)
r
1
(s)r

1
(s)
Since for the arc length parametrization r

1
(s) = 1, we obtain:
cos =
r
1
(s) r

1
(s)
r
1
(s)
(2)
We compute the dot product in (2). We rst compute r

1
(s) from (1):
r

1
(s) =
1

17
_
cos
_
4 ln
s

17
_
, sin
_
4 ln
s

17
__
+
s

17
_
sin
_
4 ln
s

17
_

17
s

1

17
, cos
_
4 ln
s

17
_

17
s

1

17
_
=
1

17
_
cos
_
4 ln
s

17
_
, sin
_
4 ln
s

17
__
+
4

17
_
sin
_
4 ln
s

17
_
, cos
_
4 ln
s

17
__
=
1

17
_
cos
_
4 ln
s

17
_
4 sin
_
4 ln
s

17
_
, sin
_
4 ln
s

17
_
+4 cos
_
4 ln
s

17
__
266 C HA P T E R 14 CALCULUS OF VECTOR- VALUED FUNCTI ONS (ET CHAPTER 13)
Thus,
r
1
(s) r

1
(s) =
s

17

1

17
_
cos
_
4 ln
s

17
__
cos
_
4 ln
s

17
_
4 sin
_
4 ln
s

17
__
+sin
_
4 ln
s

17
__
sin
_
4 ln
s

17
_
+4 cos
_
4 ln
s

17
___
=
s
17
_
cos
2
_
4 ln
s

17
_
4 cos
_
4 ln
s

17
_
sin
_
4 ln
s

17
_
+sin
2
_
4 ln
s

17
_
+4 sin
_
4 ln
s

17
_
cos
_
4 ln
s

17
__
=
s
17
_
cos
2
_
4 ln
s

17
_
+sin
2
_
4 ln
s

17
__
=
s
17
1 =
s
17
(3)
We now compute r
1
(s) from (1) (Notice that s(t ) > 0 for all t ):
r
1
(s) =
|s|

17
_
cos
2
_
4 ln
s

17
_
+sin
2
_
4 ln
s

17
_
=
s

17
1 =
s

17
(4)
Combining (2), (3) and (4) yields:
cos =
s
17
s

17
=
1

17
The solution for 0 is = 1.326 rad. Thus, the angle between r
1
(s) and r

1
(s) is constant.
Further Insights and Challenges
Prove that the length of a curve as computed using the arc length integral does not depend on its parametriza-
tion. More precisely, let C be the curve traced by r(t ) for a t b. Let (s) be a differentiable function such that

(s) > 0 and that (c) = a and (d) = b. Then r


1
(s) = r((s)) parametrizes C for c s d. Verify that
_
b
a
r

(t ) dt =
_
d
c
r

1
(s) ds
27. Show that path r(t ) =
_
1 t
2
1 +t
2
,
2t
1 +t
2
_
parametrizes the unit circle with the point (1, 0) excluded for < t <
. Use this parametrization to compute the length of the unit circle as an improper integral. Hint: The expression for
r

(t ) simplies.
SOLUTION We have x(t ) =
1t
2
1+t
2
, y(t ) =
2t
1+t
2
. Hence,
x
2
(t ) + y
2
(t ) =
_
1 t
2
1 +t
2
_
2
+
_
2t
1 +t
2
_
2
=
1 2t
2
+t
4
+4t
2
_
1 +t
2
_
2
=
1 +2t
2
+t
4
_
1 +t
2
_
2
=
_
1 +t
2
_
2
_
1 +t
2
_
2
= 1
It follows that the path r(t ) lies on the unit circle. We now show that the entire circle is indeed parametrized by r(t )
as t moves from to . First, note that x

(t ) can be written as
_
2t (1 + t
2
) 2t (1 t
2
)
_
/(1 + t
2
)
2
which is
4t /(1 + t
2
)
2
. So, for t negative, x(t ) is an increasing function, y(t ) is negative, and since lim
t
x(t ) = 1 and
lim
t 0
x(t ) = 1, we conclude that r(t ) does indeed parametrize the lower half of the circle for negative t . A similar
argument proves that we get the upper half of the circle for positive t . We now compute r

(t ) and its length:


r

(t ) =
_
2t (1 +t
2
) 2t (1 t
2
)
(1 +t
2
)
2
,
2(1 +t
2
) 2t 2t
(1 +t
2
)
2
_
=
_

4t
(1 +t
2
)
2
,
2 2t
2
(1 +t
2
)
2
_
=
1
(1 +t
2
)
2
_
4t, 2(1 t
2
)
_
r

(t ) =
1
(1 +t
2
)
2
_
16t
2
+4(1 t
2
)
2
=
2
(1 +t
2
)
2
_
t
4
+2t
2
+1
=
2
(1 +t
2
)
2
_
(t
2
+1)
2
=
2(t
2
+1)
(1 +t
2
)
2
=
2
1 +t
2
That is,
r

(t ) =
2
1 +t
2
We now use the Arc Length Formula to compute the length of the circle:
L =
_

(t ) dt = 2
_

dt
1 +t
2
= 2
_
lim
R
tan
1
R lim
R
tan
1
R
_
= 2
_

2

_

2
__
= 2
S E C T I O N 14.3 Arc Length and Speed (ET Section 13.3) 267
The involute of a circle is the curve traced by a point at the end of a thread unwinding from a circular spool of
radius R. Parametrize the involute by nding the position vector r(t ) of the point P in Figure 7 as a function of t .
Determine the arc length function and the arc length parametrization.
29. The curve r(t ) = t tanh t, sech t is called a tractrix.
(a) Show that the arc length function s(t ) =
_
t
0
r

(u) du is equal to s(t ) = ln(cosh t ).


(b) Show that t = (s) = ln(e
s
+
_
e
2s
1) is an inverse of s(t ) and verify that
r
1
(s) =
_
tanh
1
__
1 e
2s
_

_
1 e
2s
, e
s
_
is an arc length parametrization of the tractrix.
(c) Plot the tractrix if you have a computer algebra system.
SOLUTION
(a) We compute the derivative vector and its length:
r

(t ) =
_
1 sech
2
t, sech t tanh t
_
r

(t ) =
_
(1 sech
2
t ) +sech
2
t tanh
2
t =
_
1 2 sech
2
t +sech
4
t +sech
2
t tanh
2
t
=
_
sech
2
t (2 tanh
2
t ) +1 +sech
4
t
We use the identity 1 tanh
2
t = sech
2
t to write:
r

(t ) =
_
sech
2
t (1 +sech
2
t ) +1 +sech
4
t =
_
sech
2
t sech
4
t +1 +sech
4
t
=
_
1 sech
2
t =
_
tanh
2
t = | tanh t |
For t 0, tanh t 0 hence, r

(t ) = tanh t . We now apply the Arc Length Formula to obtain:


s(t ) =
_
t
0
r

(u) du =
_
t
0
(tanh u) du = ln(cosh u)

t
0
= ln(cosh t ) ln(cosh 0) = ln(cosh t ) ln 1 = ln(cosh t )
That is:
s(t ) = ln(cosh t )
(b) We show that the function t = (s) = ln
_
e
s
+
_
e
2s
1
_
is an inverse of s(t ). First we note that s

(t ) = tanh t ,
hence s

(t ) > 0 for t > 0, which implies that s(t ) has an inverse function for t 0. Therefore, it sufces to verify that
(s(t )) = t . We have:
(s(t )) = ln
_
e
ln(cosh t )
+
_
e
2 ln(cosh t )
1
_
= ln
_
cosh t +
_
cosh
2
t 1
_
Since cosh
2
t 1 = sinh
2
t we obtain (for t 0):
(s(t )) = ln
_
cosh t +
_
sinh
2
t
_
= ln (cosh t +sinh t ) = ln
_
e
t
+e
t
2
+
e
t
e
t
2
_
= ln
_
e
t
_
= t
We thus proved that t = (s) is an inverse of s(t ). Therefore, the arc length parametrization is obtained by substituting
t = (s) in r(t ) = t tanh t, sech t . We compute t , tanh t and sech t in terms of s. We have:
s = ln (cosh t ) e
s
= cosh t sech t = e
s
Also:
tanh
2
t = 1 sech
2
t = 1 e
2s
tanh t =
_
1 e
2s
t = tanh
1
_
1 e
2s
Substituting in r(t ) gives:
r
1
(s) = t tanh t, sech t =
_
tanh
1
_
1 e
2s

_
1 e
2s
, e
s
_
(c) The tractrix is shown in the following gure:
2 4
y
x
1
268 C HA P T E R 14 CALCULUS OF VECTOR- VALUED FUNCTI ONS (ET CHAPTER 13)
14.4 Curvature (ET Section 13.4)
Preliminary Questions
1. What is the unit tangent vector of a line with direction vector v = 2, 1, 2?
SOLUTION A line with direction vector v has the parametrization:
r(t ) =

OP
0
+t v
hence, since

OP
0
and v are constant vectors, we have:
r

(t ) = v
Therefore, since v = 3, the unit tangent vector is:
T(t ) =
r

(t )
r

(t )
=
v
v
= 2/3, 1/3, 2/3
2. What is the curvature of a circle of radius 4?
SOLUTION The curvature of a circle of radius R is
1
R
, hence the curvature of a circle of radius 4 is
1
4
.
3. Which has larger curvature, a circle of radius 2 or a circle of radius 4?
SOLUTION The curvature of a circle of radius 2 is
1
2
, and it is larger than the curvature of a circle of radius 4, which is
1
4
.
4. What is the curvature of r(t ) = 2 +3t, 7t, 5 t ?
SOLUTION r(t ) parametrizes the line 2, 0, 5 +t 3, 7, 1, and a line has zero curvature.
5. What is the curvature at a point where T

(s) = 1, 2, 3 in an arc length parametrization r(s)?


SOLUTION The curvature is given by the formula:
(t ) =
T

(t )
r

(t )
In an arc length parametrization, r

(t ) = 1 for all t , hence the curvature is (t ) = T

(t ). Using the given information


we obtain the following curvature:
= 1, 2, 3 =
_
1
2
+2
2
+3
2
=

14
6. What is the radius of curvature of a circle of radius 4?
SOLUTION The denition of the osculating circle implies that the osculating circles at the points of a circle, is the
circle itself. Therefore, the radius of curvature is the radius of the circle, that is, 4.
7. What is the radius of curvature at P if
P
= 9?
SOLUTION The radius of curvature is the reciprocal of the curvature, hence the radius of curvature at P is:
R =
1

P
=
1
9
Exercises
In Exercises 16, calculate r

(t ) and T(t ), and evaluate T(1).


1. r(t ) =
_
12t
3
, 18t
2
, 9t
4
_
SOLUTION The derivative vector is:
r

(t ) =
_
36t
2
, 36t, 36t
3
_
= 36
_
t
2
, t, t
3
_
r

(t ) = 36
_
t
4
+t
2
+t
6
The unit vector is thus:
T(t ) =
r

(t )
r

(t )
=
36t
2
, t, t
3

36
_
t
4
+t
2
+t
6
=
t t, 1, t
2

|t |
_
1 +t
2
+t
4
=
t
|t |
_
1 +t
2
+t
4
t, 1, t
2

At t = 1 we have:
T(1) =
1
_
1 +1
2
+1
4
1, 1, 1
2
=
1

3
1, 1, 1 =
_
1

3
,
1

3
,
1

3
_
.
S E C T I O N 14.4 Curvature (ET Section 13.4) 269
r(t ) =
_
cos t, sin t, t
_ 3. r(t ) =
_
3 +4t, 3 5t, 9t
_
SOLUTION We rst nd the vector r

(t ) and its length:


r

(t ) = 4, 5, 9 r

(t ) =
_
4
2
+(5)
2
+9
2
=

122
The unit tangent vector is therefore:
T(t ) =
r

(t )
r

(t )
=
1

122
4, 5, 9 =
_
4

122
,
5

122
,
9

122
_
We see that the unit tangent vector is constant, since the curve is a straight line.
r(t ) =
_
1 +2t, t
2
, 3 t
2
_ 5. r(t ) =
_
4t
2
, 9t
_
SOLUTION We differentiate r(t ) to obtain:
r

(t ) = 8t, 9 r

(t ) =
_
(8t )
2
+9
2
=
_
64t
2
+81
We now nd the unit tangent vector:
T(t ) =
r

(t )
r

(t )
=
1
_
64t
2
+81
8t, 9
For t = 1 we obtain the vector:
T(t ) =
1

64 +81
8, 9 =
_
8

145
,
9

145
_
.
r(t ) =
_
e
t
, t
2
_
In Exercises 712, use Eq. (3) to calculate (t ).
7. r(t ) =
_
1, e
t
, t
_
SOLUTION We compute the rst and the second derivatives of r(t ):
r

(t ) =
_
0, e
t
, 1
_
, r

(t ) =
_
0, e
t
, 0
_
.
Next, we nd the cross product r

(t ) r

(t ):
r

(t ) r

(t ) =

i j k
0 e
t
1
0 e
t
0

e
t
1
e
t
0

0 1
0 0

j +

0 e
t
0 e
t

k = e
t
i =
_
e
t
, 0, 0
_
We need to nd the lengths of the following vectors:
r

(t ) r

(t ) =

_
e
t
, 0, 0
_

= e
t
r

(t ) =
_
0
2
+(e
t
)
2
+1
2
=
_
1 +e
2t
We now use the formula for curvature to calculate (t ):
(t ) =
r

(t ) r

(t )
r

(t )
3
=
e
t
__
1 +e
2t
_
3
=
e
t
_
1 +e
2t
_
3/2
r(t ) =
_
cos t, t sin t, t
_ 9. r(t ) =
_
4 cos t, t, 4 sin t
_
SOLUTION By the formula for curvature we have:
(t ) =
r

(t ) r

(t )
r

(t )
3
(1)
First we nd r

(t ) and r

(t ):
r

(t ) = 4 sin t, 1, 4 cos t
r

(t ) = 4 cos t, 0, 4 sin t
We compute the cross product:
r

(t ) r

(t ) =

i j k
4 sin t 1 4 cos t
4 cos t 0 4 sin t

270 C HA P T E R 14 CALCULUS OF VECTOR- VALUED FUNCTI ONS (ET CHAPTER 13)


=

1 4 cos t
0 4 sin t

4 sin t 4 cos t
4 cos t 4 sin t

j +

4 sin t 1
4 cos t 0

k
= 4 sin t i
_
16 sin
2
t +16 cos
2
t
_
j +4 cos t k
= 4 sin t i 16j +4 cos t k = 4 sin t, 4, cos t
We compute the lengths of the following vectors:
r

(t ) r

(t ) = 4
_
(sin t )
2
+(4)
2
+cos
2
t = 4
_
sin
2
t +16 +cos
2
t = 4

17r

(t )
=
_
(4 sin t )
2
+1
2
+(4 cos t )
2
=
_
16 sin
2
t +1 +16 cos
2
t =

17
Substituting in (1) gives the following curvature:
(t ) =
4

17
_
17
_
3
=
4

17
17

17
=
4
17
We see that this curve has constant curvature.
r(t ) =
_
4t +1, 4t 3, 2t
_
11. r(t ) =
_
t
1
, 1, t
_
SOLUTION By the formula for curvature we have:
(t ) =
r

(t ) r

(t )
r

(t )
3
(1)
We now nd r

(t ), r

(t ) and their cross product. This gives:


r

(t ) =
_
t
2
, 0, 1
_
, r

(t ) =
_
2t
3
, 0, 0
_
r

(t ) r

(t ) =
_
t
2
i +k
_
2t
3
i = 2t
3
k i = 2t
3
j
We compute the lengths of the vector in (1):
r

(t ) r

(t ) = 2t
3
j = 2|t
3
|
r

(t ) =
_
_
(t )
2
_
2
+0
2
+1
2
=
_
t
4
+1
Substituting in (1) we obtain the following curvature:
(t ) =
2|t |
3
__
t
4
+1
_
3
=
2|t |
3
_
t
4
+1
_
3/2
We multiply through by |t |
43/2
= |t |
6
to obtain:
(t ) =
2|t |
3
_
1 +|t |
4
_
3/2
r(t ) =
_
cosh t, sinh t, t
_
In Exercises 1316, nd the curvature of the plane curve at the point indicated.
13. y = e
t
, t = 3
SOLUTION We use the curvature of a graph in the plane:
(t ) =
| f

(t )|
_
1 + f

(t )
2
_
3/2
In our case f (t ) = e
t
, hence f

(t ) = f

(t ) = e
t
and we obtain:
(t ) =
e
t
_
1 +e
2t
_
3/2
(3) =
e
3
_
1 +e
6
_
3/2
0.0025
y = cos x, x = 0
15. y = t
4
, t = 2
S E C T I O N 14.4 Curvature (ET Section 13.4) 271
SOLUTION By the curvature of a graph in the plane, we have:
(t ) =
| f

(t )|
_
1 + f

(t )
2
_
3/2
In this case f (t ) = t
4
, f

(t ) = 4t
3
, f

(t ) = 12t
2
. Hence,
(t ) =
12t
2
_
1 +
_
4t
3
_
2
_
3/2
=
12t
2
_
1 +16t
6
_
3/2
At t = 2 we obtain the following curvature:
(2) =
12 2
2
(1 +16 2
6
)
3/2
=
48
(1,025)
3/2
0.0015.
y = t
n
, t = 1
17. Find the curvature of r(t ) = 2 sin t, cos 3t, t at t =

3
and t =

2
(Figure 15).
y
x
z

3
t =
FIGURE 15 The curve r(t ) = 2 sin t, cos 3t, t .
SOLUTION By the formula for curvature we have:
(t ) =
r

(t ) r

(t )
r

(t )
3
(1)
We compute the rst and second derivatives:
r

(t ) = 2 cos t, 3 sin 3t, 1 , r

(t ) = 2 sin t, 9 cos 3t, 0


At the points t =

3
and t =

2
we have:
r

3
_
=
_
2 cos

3
, 3 sin
3
3
, 1
_
=
_
2 cos

3
, 3 sin , 1
_
= 1, 0, 1
r

3
_
=
_
2 sin

3
, 9 cos
3
3
, 0
_
=
_

3, 9, 0
_
r

2
_
=
_
2 cos

2
, 3 sin
3
2
, 1
_
= 0, 3, 1
r

2
_
=
_
2 sin

2
, 9 cos
3
2
, 0
_
= 2, 0, 0
We compute the cross products required to use (1):
r

3
_
r

3
_
=

i j k
1 0 1

3 9 0

0 1
9 0

1 1

3 0

j +

1 0

3 9

k = 9i

3j +9k
r

2
_
r

2
_
=

i j k
0 3 1
2 0 0

3 1
0 0

0 1
2 0

j +

0 3
2 0

k = 2j +6k
Hence,
_
_
_r

3
_
r

3
__
_
_ =
_
(9)
2
+
_

3
_
2
+9
2
=

165
_
_
_r

3
__
_
_ =
_
1
2
+0
2
+1
2
=

2
272 C HA P T E R 14 CALCULUS OF VECTOR- VALUED FUNCTI ONS (ET CHAPTER 13)
At t =

2
we have:
_
_
_r

2
_
r

2
__
_
_ =
_
(2)
2
+6
2
=

40 = 2

10
_
_
_r

2
__
_
_ =
_
0
2
+3
2
+1
2
=

10
Substituting the values for t =

3
and t =

2
in (1) we obtain the following curvatures:

3
_
=

165
_
2
_
3
=

165
2

2
4.54

2
_
=
2

10
_
10
_
3
=
2

10
10

10
= 0.2
Find the curvature function (x) for y = sin x. Use a computer algebra system to plot (x) for 0 x 2.
Prove that the curvature takes its maximum at x =

2
and
3
2
. Hint: To simplify the calculation, nd the maximum
of (x)
2
.
19. Show that curvature at an inection point of a plane curve y = f (x) is zero.
SOLUTION The curvature of the graph y = f (x) in the plane is the following function:
(x) =
| f

(x)|
_
1 + f

(x)
2
_
3/2
(1)
At an inection point the second derivative changes its sign. Therefore, if f

is continuous at the inection point, it is


zero at this point, hence by (1) the curvature at this point is zero.
Show that the tractrix r(t ) = t tanh t, sech t has the curvature function (t ) = sech t .
21. Find the value of such that the curvature of y = e
x
at x = 0 is as large as possible.
SOLUTION Using the curvature of a graph in the plane we have:
(x) =
|y

(x)|
_
1 + y

(x)
2
_
3/2
(1)
In our case y

(x) = e
x
, y

(x) =
2
e
x
. Substituting in (1) we obtain
(x) =

2
e
x
_
1 +
2
e
2x
_
3/2
The curvature at the origin is thus
(0) =

2
e
0
_
1 +
2
e
20
_
3/2
=

2
_
1 +
2
_
3/2
Since (0) and
2
(0) have their maximum values at the same values of , we may maximize the function:
g() =
2
(0) =

4
(1 +
2
)
3
We nd the stationary points:
g

() =
4
3
(1 +
2
)
3

4
(3)(1 +
2
)
2
2
(1 +
2
)
6
=
2
3
(1 +
2
)
2
(2
2
)
(1 +
2
)
6
= 0
The stationary points are the solutions of the following equation:
2
3
(1 +
2
)
2
(2
2
) = 0

3
= 0 or 2
2
= 0
= 0 =

2
Since g() 0 and g(0) = 0, = 0 is a minimum point. Also, g

() is positive immediately to the left of



2 and
negative to the right. Hence, =

2 is a maximum point. Since g() is an even function, =

2 is a maximum
point as well. Conclusion: (x) takes its maximum value at the origin when =

2.
Find the point of maximum curvature on y = e
x
. 23. Show that the curvature function of the parametrization r(t ) = a cos t, b sin t of the ellipse
_
x
a
_
2
+
_
y
b
_
2
= 1 is
(t ) =
ab
(b
2
cos
2
t +a
2
sin
2
t )
3/2
8
S E C T I O N 14.4 Curvature (ET Section 13.4) 273
SOLUTION The curvature is the following function:
(t ) =
r

(t ) r

(t )
r

(t )
3
(1)
We compute the derivatives and their cross product:
r

(t ) = a sin t, b cos t , r

(t ) = a cos t, b sin t
r

(t ) r

(t ) = (a sin t i +b cos t j) (a cos t i b sin t j)


= ab sin
2
t k +ab cos
2
t k = ab
_
sin
2
t +cos
2
t
_
k = abk
Thus,
r

(t ) r

(t ) = abk = ab
r

(t ) =
_
(a sin t )
2
+(b cos t )
2
=
_
a
2
sin
2
t +b
2
cos
2
t
Substituting in (1) we obtain the following curvature:
(t ) =
ab
__
a
2
sin
2
t +b
2
cos
2
t
_
3
=
ab
_
a
2
sin
2
t +b
2
cos
2
t
_
3/2
Use a sketch to predict where the points of minimal and maximal curvature occur on an ellipse. Then use Eq. (8)
to conrm or refute your prediction.
25. In the notation of Exercise 23, assume that a b. Show that b/a
2
(t ) a/b
2
for all t .
SOLUTION In Exercise 23 we showed that the curvature of the ellipse r(t ) = a cos t, b sin t is the following function:
(t ) =
ab
_
b
2
cos
2
t +a
2
sin
2
t
_
3/2
Since a b > 0 the quotient becomes greater if we replace a by b in the denominator, and it becomes smaller if we
replace b by a in the denominator. We use the identity cos
2
t +sin
2
t = 1 to obtain:
ab
_
a
2
cos
2
t +a
2
sin
2
t
_
3/2
(t )
ab
_
b
2
cos
2
t +b
2
sin
2
t
_
3/2
ab
_
a
2
_
cos
2
t +sin
2
t
__
3/2
(t )
ab
_
b
2
_
cos
2
t +sin
2
t
__
3/2
ab
a
3
=
ab
(a
2
)
3/2
(t )
ab
(b
2
)
3/2
=
ab
b
3
b
a
2
(t )
a
b
2
Use Eq. (3) to prove that for a plane curve r(t ) = x(t ), y(t ),
(t ) =
|x

(t )y

(t ) x

(t )y

(t )|
(x

(t )
2
+ y

(t )
2
)
3/2
In Exercises 2730, use Eq. (9) to compute the curvature at the given point.
27.
_
t
2
, t
3
_
, t = 2
SOLUTION For the given parametrization, x(t ) = t
2
, y(t ) = t
3
, hence
x

(t ) = 2t
x

(t ) = 2
y

(t ) = 3t
2
y

(t ) = 6t
At the point t = 2 we have
x

(2) = 4, x

(2) = 2, y

(2) = 3 2
2
= 12, y

(2) = 12
Substituting in Eq. (9) we get
(2) =
|x

(2)y

(2) x

(2)y

(2)|
_
x

(2)
2
+ y

(2)
2
_
3/2
=
|4 12 2 12|
_
4
2
+12
2
_
3/2
=
24
160
3/2
0.012
_
cosh s, s
_
, s = 0
274 C HA P T E R 14 CALCULUS OF VECTOR- VALUED FUNCTI ONS (ET CHAPTER 13)
29.
_
t cos t, sin t
_
, t =
SOLUTION We have x(t ) = t cos t and y(t ) = sin t , hence:
x

(t ) = cos t t sin t x

() = cos sin = 1
x

(t ) = sin t (sin t +t cos t ) = 2 sin t t cos t x

() = 2 sin cos =
y

(t ) = cos t y

() = cos = 1
y

(t ) = sin t y

() = sin = 0
Substituting in Eq. (9) gives the following curvature:
() =
|x

()y

() x

()y

()|
_
x

()
2
+ y

()
2
_
3/2
=
| 1 0 (1)|
_
(1)
2
+(1)
2
_
3/2
=

2

2
1.11
_
sin 3s, 2 sin 4s
_
, s =

2
31. Let s(t ) =
_
t

(u) du for the Bernoulli spiral r(t ) =


_
e
t
cos 4t, e
t
sin 4t
_
(see Exercise 25 in Section 14.3).
Show that the radius of curvature is proportional to s(t ).
SOLUTION The radius of curvature is the reciprocal of the curvature:
R(t ) =
1
(t )
We compute the curvature using the equality given in Exercise 25 in Section 3:
(t ) =
|x

(t )y

(t ) x

(t )y

(t )|
_
x

(t )
2
+ y

(t )
2
_
3/2
(1)
In our case, x(t ) = e
t
cos 4t and y(t ) = e
t
sin 4t . Hence:
x

(t ) = e
t
cos 4t 4e
t
sin 4t = e
t
(cos 4t 4 sin 4t )
x

(t ) = e
t
(cos 4t 4 sin 4t ) +e
t
(4 sin 4t 16 cos 4t ) = e
t
(15 cos 4t +8 sin 4t )
y

(t ) = e
t
sin 4t +4e
t
cos 4t = e
t
(sin 4t +4 cos 4t )
y

(t ) = e
t
(sin 4t +4 cos 4t ) +e
t
(4 cos 4t 16 sin 4t ) = e
t
(8 cos 4t 15 sin 4t )
We compute the numerator in (1):
x

(t )y

(t ) x

(t )y

(t ) = e
2t
(cos 4t 4 sin 4t ) (8 cos 4t 15 sin 4t ) +e
2t
(15 cos 4t +8 sin 4t ) (sin 4t +4 cos 4t )
= e
2t
_
68 cos
2
4t +68 sin
2
4t
_
= 68e
2t
We compute the denominator in (1):
x

(t )
2
+ y

(t )
2
= e
2t
(cos 4t 4 sin 4t )
2
+e
2t
(sin 4t +4 cos 4t )
2
= e
2t
_
cos
2
4t 8 cos 4t sin 4t +16 sin
2
4t +sin
2
4t +8 sin 4t cos 4t +16 cos
2
4t
_
= e
2t
_
cos
2
4t +sin
2
4t +16
_
sin
2
4t +cos
2
4t
__
= e
2t
(1 +16 1) = 17e
2t
(2)
Hence
_
x

(t )
2
+ y

(t )
2
_
3/2
= 17
3/2
e
3t
Substituting in (2) we have
(t ) =
68e
2t
17
3/2
e
3t
=
4

17
e
t
R =

17
4
e
t
(3)
On the other hand, by the Fundamental Theorem and (2) we have
s

(t ) = r

(t ) =
_
x

(t )
2
+ y

(t )
2
=
_
17e
2t
=

17e
t
We integrate to obtain
s(t ) =
_

17 e
t
dt =

17 e
t
+C (4)
S E C T I O N 14.4 Curvature (ET Section 13.4) 275
Since s(t ) =
_
t

(u) du, we have lim


t
s(t ) = 0, hence by (4):
0 = lim
t
_

17e
t
+C
_
= 0 +C = C.
Substituting C = 0 in (4) we get:
s(t ) =

17e
t
(5)
Combining (3) and (5) gives:
R(t ) =
1
4
s(t )
which means that the radius of curvature is proportional to s(t ).
The Cornu spiral is the plane curve r(t ) = x(t ), y(t ), where
x(t ) =
_
t
0
sin
u
2
2
du, y(t ) =
_
t
0
cos
u
2
2
du
Verify that (t ) = |t |. Since the curvature increases linearly, the Cornu spiral is used in highway design to create
transitions between straight and curved road segments (Figure 16).
33. Plot and compute the curvature (t ) of the clothoid r(t ) = x(t ), y(t ), where
x(t ) =
_
t
0
sin
u
3
3
du, y(t ) =
_
t
0
cos
u
3
3
du
SOLUTION We use the following formula for the curvature (given earlier):
(t ) =
|x

(t )y

(t ) x

(t )y

(t )|
_
x

(t )
2
+ y

(t )
2
_
3/2
(1)
We compute the rst and second derivatives of x(t ) and y(t ). Using the Fundamental Theorem and the Chain Rule we
get:
x

(t ) = sin
t
3
3
x

(t ) =
3t
2
3
cos
t
3
3
= t
2
cos
t
3
3
y

(t ) = cos
t
3
3
y

(t ) =
3t
2
3
_
sin
t
3
3
_
= t
2
sin
t
3
3
Substituting in (1) gives the following curvature function:
(t ) =

sin
t
3
3
_
t
2
sin
t
3
3
_
t
2
cos
t
3
3
cos
t
3
3

_
_
sin
t
3
3
_
2
+
_
cos
t
3
3
_
2
_
3/2
=
t
2
_
sin
2 t
3
3
+cos
2 t
3
3
_
1
3/2
= t
2
That is, (t ) = t
2
. Here is a plot of the curvature as a function of t :

t
(t ) = t
2
Find the unit normal vector N() to r() = R cos , sin , the circle of radius R. Does N() point inside or
outside the circle? Draw N() at =

4
with R = 4.
35. Find the unit normal vector N(t ) to r(t ) = 4, sin 2t, cos 2t .
SOLUTION We rst nd the unit tangent vector:
T(t ) =
r

(t )
r

(t )
(1)
276 C HA P T E R 14 CALCULUS OF VECTOR- VALUED FUNCTI ONS (ET CHAPTER 13)
We have
r

(t ) =
d
dt
4, sin 2t, cos 2t = 0, 2 cos 2t, 2 sin 2t = 2 0, cos 2t, sin 2t
r

(t ) = 2
_
0
2
+cos
2
2t +(sin 2t )
2
= 2

0 +1 = 2
Substituting in (1) gives:
T(t ) =
2 0, cos 2t, sin 2t
2
= 0, cos 2t, sin 2t
The normal vector is the following vector:
N(t ) =
T

(t )
T

(t )
(2)
We compute the derivative of the unit tangent vector and its length:
T

(t ) =
d
dt
0, cos 2t, sin 2t = 0, 2 sin 2t, 2 cos 2t = 2 0, sin 2t, cos 2t
T

(t ) = 2
_
0
2
+sin
2
2t +cos
2
2t = 2

0 +1 = 2
Substituting in (2) we obtain:
N(t ) =
2 0, sin 2t, cos 2t
2
= 0, sin 2t, cos 2t
Sketch the graph of r(t ) =
_
t, t
3
_
. Since r

(t ) =
_
1, 3t
2
_
, the unit normal N(t ) points in one of the two directions

_
3t
2
, 1
_
. Which sign is correct at t = 1? Which is correct at t = 1?
37. Find the normal vectors to r(t ) = t, cos t at t =

4
and t =
3
4
.
SOLUTION The normal vector to r(t ) = t, cos t is T

(t ), where T(t ) =
r

(t )
r

(t )
is the unit tangent vector. We have
r

(t ) = 1, sin t r

(t ) =
_
1
2
+(sin t )
2
=
_
1 +sin
2
t
Hence,
T(t ) =
1
_
1 +sin
2
t
1, sin t
We compute the derivative of T(t ) to nd the normal vector.We use the Product Rule and the Chain Rule to obtain:
T

(t ) =
1
_
1 +sin
2
t
d
dt
1, sin t +
_
1
_
1 +sin
2
t
_

1, sin t
=
1
_
1 +sin
2
t
0, cos t
1
1 +sin
2
t

2 sin t cos t
2
_
1 +sin
2
t
1, sin t
=
1
_
1 +sin
2
t
0, cos t
sin 2t
2
_
1 +sin
2
t
_
3/2
1, sin t
At t =

4
we obtain the normal vector:
T

4
_
=
1
_
1 +
1
2
_
0,
1

2
_

1
2
_
1 +
1
2
_
3/2
_
1,
1

2
_
=
_
0,
1

3
_

_

2
3

3
,
1
3

3
_
=
_

2
3

3
,
2
3

3
_
At t =
3
4
we obtain:
T

_
3
4
_
=
1
_
1 +
1
2
_
0,
1

2
_

1
2
_
1 +
1
2
_
3/2
_
1,
1

2
_
=
_
0,
1

3
_
+
_

2
3

3
,
1
3

3
_
=
_

2
3

3
,
2
3

3
_
Find the unit normal to the Cornu spiral (Exercise 32) at t =

.
39. Find the unit normal to the clothoid (Exercise 33) at t =
1/3
.
S E C T I O N 14.4 Curvature (ET Section 13.4) 277
SOLUTION The Clothoid is the plane curve r(t ) = x(t ), y(t ) with
x(t ) =
_
t
0
sin
u
3
3
du, y(t ) =
_
t
0
cos
u
3
3
du
The unit normal is the following vector:
N(t ) =
T

(t )
T

(t )
(1)
We rst nd the unit tangent vector T(t ) =
r

(t )
r

(t )
. By the Fundamental Theorem we have
r

(t ) =
_
sin
t
3
3
, cos
t
3
3
_
r

(t ) =
_
sin
2
t
3
3
+cos
2
t
3
3
=

1 = 1
Hence,
T(t ) =
_
sin
t
3
3
, cos
t
3
3
_
We now differentiate T(t ) using the Chain Rule to obtain:
T

(t ) =
_
3t
2
3
cos
t
3
3
,
3t
2
3
sin
t
3
3
_
= t
2
_
cos
t
3
3
, sin
t
3
3
_
Hence,
T

(t ) = t
2
_
cos
2
t
3
3
+
_
sin
t
3
3
_
2
= t
2
Substituting in (1) we obtain the following unit normal:
N(t ) =
_
cos
t
3
3
, sin
t
3
3
_
At the point T =
1/3
the unit normal is
N(
1/3
) =
_
cos
(
1/3
)
3
3
, sin
(
1/3
)
3
3
_
=
_
cos

3
, sin

3
_
=
_
1
2
,

3
2
_
Method for Computing N Let v(t ) = r

(t ). Show that
N(t ) =
r

(t ) v

(t )T(t )
r

(t ) v

(t )T(t )
Hint: Differentiate r

(t ) = v(t )T(t ) and note that N is the unit vector in the direction T

.
In Exercises 4146, use Eq. (10) to nd N at the point indicated.
41.
_
1 +t
2
, 2t, t
3
_
, t = 1
SOLUTION We compute the values in formula (10). In our case
r(t ) =
_
1 +t
2
, 2t, t
3
_
Hence,
r

(t ) =
_
2t, 2, 3t
2
_
r

(t ) = 2, 0, 6t
v(t ) = r

(t ) =
_
(2t )
2
+2
2
+(3t
2
)
2
=
_
4t
2
+4 +9t
4
=
_
4 +4t
2
+9t
4
v

(t ) =
8t +36t
3
2
_
4 +4t
2
+9t
4
=
4t +18t
3
_
4 +4t
2
+9t
4
At the point t = 1, we have
r

(1) = 2, 0, 6 , v

(1) =
22

17
, T(1) =
r

(1)
r

(1)
=
2, 2, 3

17
Hence,
r

(1) v

(1)T(1) = 2, 0, 6
22

17

1

17
2, 2, 3 = 2, 0, 6
_
44
17
,
44
17
,
66
17
_
278 C HA P T E R 14 CALCULUS OF VECTOR- VALUED FUNCTI ONS (ET CHAPTER 13)
=
_

10
17
,
44
17
,
36
17
_
=
1
17
10, 44, 36
r

(1) v

(1)T(1) =
1
17
_
(10)
2
+(44)
2
+36
2
=
1
17
_
3,332 =
14

17
17
=
14

17
Substituting in equation (10) we get
N(1) =
r

(1) v

(1)T(1)
r

(1) v

(1)T(1)
=
1
17
10, 44, 36
14

17
=
1
7

17
5, 22, 18
_
t, e
t
, t
_
, t = 2
43.
_
t sin t, 1 cos t
_
, t =
SOLUTION We use the following equality:
N(t ) =
r

(t ) v

(t )T(t )
r

(t ) v

(t )T(t )
(1)
We compute the vectors in the above equality. For r(t ) = t sin t, 1 cos t we have
r

(t ) = 1 cos t, sin t
r

(t ) = sin t, cos t
v(t ) = r

(t ) =
_
(1 cos t )
2
+sin
2
t =
_
1 2 cos t +cos
2
t +sin
2
t
=

1 2 cos t +1 =
_
2(1 cos t ) =
_
2 2 sin
2
t
2
= 2

sin
t
2

For 0 t 2, sin
t
2
0, hence v(t ) = 2 sin
t
2
. Therefore,
v

(t ) = 2
1
2
cos
t
2
= cos
t
2
, 0 t 2
At the point t = we have
r

() = sin , cos = 0, 1
v

() = cos

2
= 0
r

() = 1 cos , sin = 2, 0
T() =
r

()
r

()
=
2, 0
2
= 1, 0
We now substitute these values in (1) to obtain the following unit normal:
N() =
0, 1 0 1, 0
0, 1 0 1, 0
=
0, 1
1
= 0, 1
_
t
2
, t
3
_
, t = 1
45.
_
t
1
, t, t
2
_
, t = 1
SOLUTION We use the equality
N(t ) =
r

(t ) v

(t )T(t )
r

(t ) v

(t )T(t )
(1)
We compute the vectors in the above equality. For r(t ) =
_
t
1
, t, t
2
_
we have
r

(t ) =
_
t
2
, 1, 2t
_
r

(t ) =
_
2t
3
, 0, 2
_
v(t ) = r

(t ) =
_
t
4
+1 +4t
2
v

(t ) =
4t
5
+8t
2
_
t
4
+1 +4t
2
=
2t
5
+4t
_
t
4
+1 +4t
2
At the point t = 1 we get
r

(1) = 1, 1, 2 , r

(1) = 2, 0, 2 , v

(1) =
2 4

1 +1 +4
=
2

6
,
S E C T I O N 14.4 Curvature (ET Section 13.4) 279
T(1) =
r

(1)
r

(1)
=
1, 1, 2

6
Hence,
r

(1) v

(1)T(1) = 2, 0, 2 +
2

6

1

6
1, 1, 2 =
_

7
3
,
1
3
,
4
3
_
=
1
3
7, 1, 4
r

(1) v

(1)T(1) =
1
3
_
(7)
2
+1
2
+4
2
=
1
3

66
Substituting in (1) gives the following unit normal:
N(1) =
1
3
7, 1, 4
1
3

66
=
1

66
7, 1, 4
_
cosh t, sinh t, t
_
, t = 0
47. Let f (x) = x
2
. Show that the center of the osculating circle at (x
0
, x
2
0
) is given by
_
4x
3
0
,
1
2
+3x
2
0
_
.
SOLUTION We parametrize the curve by r(x) =
_
x, x
2
_
. The center Q of the osculating circle at x = x
0
has the
position vector

OQ = r(x
0
) +(x
0
)
1
N(x
0
) (1)
We rst nd the curvature, using the formula for the curvature of a graph in the plane. We have f

(x) = 2x and
f

(x) = 2, hence,
(x) =
| f

(x)|
(1 + f

(x)
2
)
3/2
=
2
(1 +4x
2
)
3/2
(x
0
)
1
=
1
2
(1 +4x
2
0
)
3/2
To nd the unit normal vector N(x
0
) we use the following considerations:
The tangent vector is r

(x
0
) = 1, 2x
0
, hence the vector 2x
0
, 1 is orthogonal to r

(x
0
) (since their dot product
is zero). Hence N(x
0
) is one of the two unit vectors
1
_
1+4x
2
0
2x
0
, 1.
The graph of f (x) = x
2
shows that the unit normal vector points in the positive y-direction, hence, the appropriate
choice is:
N(x
0
) =
1
_
1 +4x
2
0
2x
0
, 1 (2)
y
x
f (x) = x
2
We now substitute (2), (3), and r(x
0
) =
_
x
0
, x
2
0
_
in (1) to obtain

OQ =
_
x
0
, x
2
0
_
+
1
2
_
1 +4x
2
0
_
3/2

1
_
1 +4x
2
0
_
2x
0
, 1
_
=
_
x
0
, x
2
0
_
+
1
2
_
1 +4x
2
0
_
2x
0
, 1
=
_
x
0
, x
2
0
_
+
_
x
0
4x
3
0
,
1
2
_
1 +4x
2
0
_
_
=
_
4x
3
0
,
1
2
+3x
2
0
_
The center of the osculating circle is the terminal point of

OQ, that is,


Q =
_
4x
3
0
,
1
2
+3x
2
0
_
Find a parametrization of the osculating circle to y = x
2
at x = 1.
49. Find a parametrization of the osculating circle to y = sin x at x =

2
.
280 C HA P T E R 14 CALCULUS OF VECTOR- VALUED FUNCTI ONS (ET CHAPTER 13)
SOLUTION We use the parametrization r(x) = x, sin x. The radius of the osculating circle is the radius of curvature
R =
1

2
_
and the center is the terminal point of the following vector:

OQ = r
_

2
_
+ RN
_

2
_
We rst compute the curvature. Since y

(x) = cos x and y

(x) = sin x, we have:


(x) =
|y

(x)|
(1 + y

(x)
2
)
3/2
=
| sin x|
(1 +cos
2
x)
3/2

_

2
_
=
sin

2
_
1 +cos
2

2
_
3/2
=
1
1
= 1
We compute the unit normal vector N(x). N(x) is a unit vector orthogonal to the tangent vector r

(x) = 1, cos x. We
observe that cos x, 1 is orthogonal to r

(x), since their dot product is zero. Therefore, N(x) is the unit vector in the
direction of either cos x, 1 or cos x, 1, depending on the graph. Considering the accompanying gure, we see
that the unit normal vector at x = /2 points to the negative y-direction. Thus,
N(x) =
cos x, 1
cos x, 1
=
cos x, 1
_
cos
2
x +(1)
2
N
_

2
_
= 0, 1
y
x
P
(
, 1
)
Q
(
, 0
)
We now nd the center of the osculating circle. We substitute R =
1

2
_
= 1, N
_

2
_
= 0, 1, and r
_

2
_
=
_

2
, sin

2
_
=
_

2
, 1
_
into (1) to obtain

OQ =
_

2
, 1
_
+1 0, 1 =
_

2
, 0
_
The osculating circle is the circle with center at the point
_

2
, 0
_
and radius 1, so it has the following parametrization:
c(t ) =
_

2
, 0
_
+1 cos t, sin t =
_

2
, 0
_
+cos t, sin t
Use Eq. (7) to nd the center of curvature to r(t ) =
_
t
2
, t
3
_
at t = 1.
In Exercises 5155, nd a parametrization of the osculating circle at the point indicated.
51.
_
cos t, sin t
_
, t =

4
SOLUTION The curve r(t ) = cos t, sin t is the unit circle. By the denition of the osculating circle, it follows that the
osculating circle at each point of the circle is the circle itself. Therefore the osculating circle to the unit circle at t =

4
is
the unit circle itself.
_
sin t, cos t
_
, t = 0
53.
_
t sin t, 1 cos t
_
, t = (use Exercise 43)
SOLUTION
Step 1. Find and N. In Exercise 43 we found that:
N() = 0, 1 (1)
To nd we use the formula for curvature:
() =
r

() r

()
r

()
3
(2)
For r(t ) = t sin t, 1 cos t we have:
r

(t ) = 1 cos t, sin t r

() = 1 cos , sin = 2, 0
r

(t ) = sin t, cos t r

() = sin , cos = 0, 1
Hence,
r

() r

() = 2i (j) = 2k
S E C T I O N 14.4 Curvature (ET Section 13.4) 281
r

() r

() = 2k = 2 and r

() = 2, 0 = 2
Substituting in (2) we get:
() =
2
2
3
=
1
4
(3)
Step 2. Find the center of the osculating circle. The center Q of the osculating circle at r () = , 2 has position vector

OQ = r () +()
1
N ()
Substituting (1), (3) and r () = , 2 we get:

OQ = , 2 +
_
1
4
_
1
0, 1 = , 2 +0, 4 = , 2
Step 3. Parametrize the osculating circle. The osculating circle has radius R =
1
()
and it is centered at (, 2), hence
it has the following parametrization:
c(t ) = , 2 +4 cos t, sin t
_
1 +t
2
, 2t, t
3
_
, t = 1 (use Exercise 41)
55. r(t ) =
_
cosh t, sinh t, t
_
, t = 0 (use Exercise 12)
SOLUTION
Step 1. Find and N. In Exercise 12 we found that:
(t ) =
1
2 cosh
2
t
(0) =
1
2 cosh
2
0
=
1
2
(1)
We now must nd the unit normal N. We have:
r

(t ) = sinh t, cosh t, 1
r

(t ) =
_
sinh
2
t +cosh
2
t +1 =
_
cosh
2
t 1 +cosh
2
t +1 =
_
2 cosh
2
t =

2 cosh t
T(t ) =
r

(t )
r

(t )
=
1

2 cosh t
sinh t, cosh t, 1 =
1

2
tanh t, 1, sech t
T

(t ) =
1

2
_
sech
2
t, 0, sech t tanh t
_
We compute the length of T

(t ). Using the identity tanh


2
t +sech
2
t = 1 we get:
T

(t ) =
1

2
_
sech
4
t +0 +sech
2
t tanh
2
t =
1

2
_
sech
2
t
_
tanh
2
t +sech
2
t
_
=
1

2
_
sech
2
t 1 =
sech t

2
Hence,
N(t ) =
T

(t )
T

(t )
=

2
sech t
1

2

_
sech
2
t, 0, sech t tanh t
_
=
1
2
sech t, 0, tanh t
At the point t = 0 we have sech 0 = 1, tanh 0 = 0, hence
N(0) = 1, 0, 0 (2)
Step 2. Find the center of the osculating circle. The center Q of the osculating circle at r(0) = 1, 0, 0 has position
vector:

OQ = r(0) +(0)
1
N(0)
Substituting (1), (2) and r(0) = 1, 0, 0 we get:

OQ = 1, 0, 0 +2 1, 0, 0 = 3, 0, 0
Step 3. Parametrize the osculating circle. The osculating circle is centered at Q = (3, 0, 0) and has radius R =
1
(0)
= 2,
hence it has the following parametrization:
c(t ) = 3, 0, 0 +2Ncos t +2Tsin t = 3, 0, 0 +21, 0, 0 cos t +
2

2
0, 1, 1 sin t
Find the curvature and unit normal vector to the helix r(t ) =
_
cos t, sin t, t
_
at t = 0. Then use Eq. (7) to nd the
center of curvature and a parametrization of the osculating circle at t = 0.
282 C HA P T E R 14 CALCULUS OF VECTOR- VALUED FUNCTI ONS (ET CHAPTER 13)
57. Figure 17 shows the graph of the half-ellipse y =
_
2r x px
2
, where r and p are positive constants. Show that
the radius of curvature at the origin is equal to r. Hint: One way of proceeding is to write the ellipse in the form of
Exercise 23 and apply Eq. (8).
x
y
r
r
FIGURE 17 The curve y =
_
2r x px
2
and the osculating circle at the origin.
SOLUTION The radius of curvature is the reciprocal of the curvature. We thus must nd the curvature at the origin. We
use the following simple variant of the formula for the curvature of a graph in the plane:
(y) =
|x

(y)|
_
1 + x

(y)
2
_
3/2
(1)
(The traditional formula of (x) =
|y

(x)|
_
1+y

(x)
2
_
3/2
is inappropriate for this problem, as y

(x) is undened at x = 0.) We


nd x in terms of y:
y =
_
2r x px
2
y
2
= 2r x px
2
px
2
2r x + y
2
= 0
We solve for x and obtain:
x =
1
p
_
r
2
py
2
+
r
p
, y 0.
We nd x

and x

:
x

=
2py
2p
_
r
2
py
2
=
y
_
r
2
py
2
x

=
1
_
r
2
py
2
y
py

r
2
py
2
r
2
py
2
=
r
2
py
2
+ py
2
_
r
2
py
2
_
3/2
=
r
2
_
r
2
py
2
_
3/2
At the origin we get:
x

(0) = 0, x

(0) =
r
2
(r
2
)
3/2
=
1
r
Substituting in (1) gives the following curvature at the origin:
(0) =
|x

(0)|
(1 + x

(0)
2
)
3/2
=
|
1
r
|
(1 +0)
3/2
=
1
|r|
=
1
r
We conclude that the radius of curvature at the origin is
R =
1
(0)
= r
In a recent study of laser eye surgery by Gatinel, Hoang-Xuan, and Azar, a vertical cross section of the cornea is
modeled by the half-ellipse of Exercise 57. Show that the half-ellipse can be written in the form x = f (y), where
f (y) = p
1
_
r
_
r
2
py
2
_
. During surgery, tissue is removed to a depth t (y) at height y for S y S, where
t (y) is given by Munnerlyns equation (for some R > r):
t (y) =
_
R
2
S
2

_
R
2
y
2

_
r
2
S
2
+
_
r
2
y
2
After surgery, the cross section of the cornea has the shape x = f (y) + t (y) (Figure 18). Show that after surgery,
the radius of curvature at the point P (where y = 0) is R.
59. The angle of inclination of a plane curve with parametrization r(t ) is dened as the angle (t ) between the unit
tangent vector T(t ) and the x-axis (Figure 19). Show that T

(t ) = |

(t )| and conclude that if r(s) is a parametrization


by arc length, then
(s) =

d
ds

11
Hint: Observe that T(t ) = cos (t ), sin (t ).
S E C T I O N 14.4 Curvature (ET Section 13.4) 283
y
x
r(t)
(t)
T(t) = cos (t), sin (t)
FIGURE 19 The curvature is the rate of change of (t ).
SOLUTION Since T(t ) is a unit vector that makes an angle (t ) with the positive x-axis, we have
T(t ) = cos (t ), sin (t ) .
Differentiating this vector using the Chain Rule gives:
T

(t ) =
_

(t ) sin (t ),

(t ) cos (t )
_
=

(t ) sin (t ), cos (t )
We compute the norm of the vector T

(t ):
T

(t ) =

(t ) sin (t ), cos (t ) = |

(t )|
_
(sin (t ))
2
+(cos (t ))
2
= |

(t )| 1 = |

(t )|
When r(s) is a parametrization by arc length we have:
(s) =
_
_
_
_
dT
ds
_
_
_
_
=
_
_
_
_
dT
dt
_
_
_
_

dt
d
d
ds

(t )

1
|

(t )|

d
ds

d
ds

as desired.
A particle moves along the path y = x
3
with unit speed. How fast is the tangent turning (i.e., how fast is the
angle of inclination changing) when the particle passes through the point (2, 8)?
61. Verify Eq. (11) for a circle of radius R. Suppose that a particle traverses the circle at unit speed. Show that the change
in the angle during an interval of length t is = t /R and conclude that

(s) = 1/R.
SOLUTION The particle traverses the circle at unit speed hence the parametrization is the arc length parametrization of
the circle. That is,
r(s) = R
_
cos
s
R
, sin
s
R
_
The angle is (s) =
s
R
, hence the change in the angle during an interval of length s is:
= (s +s) (s) =
s +s
R

s
R
=
s
R
Therefore,

s
=
1
R
and we obtain the following derivative:

(s) = lim
s0

s
= lim
s0
1
R
=
1
R
The curvature of a circle of radius R is
1
R
, therefore, we have:
d
ds
=
1
R
.
This equality veries Eq. (11), for this case.
Let (x) be the angle of inclination at a point on the graph y = f (x) (see Exercise 59).
(a) Use the relation f

(x) = tan to prove that


d
dx
=
f

(x)
(1 + f

(x)
2
)
.
(b) Use the arc length integral to show that
ds
dx
=
_
1 + f

(x)
2
.
(c) Now give a proof of Eq. (5) using Eq. (11).
63. Use the parametrization r() = f () cos , f () sin to show that a curve r = f () in polar coordinates has
curvature
() =
| f ()
2
+2 f

()
2
2 f () f

()|
( f ()
2
+ f

()
2
)
3/2
12
SOLUTION By the formula for curvature we have
() =
r

() r

()
r

()
3
(1)
We differentiate r() and r

():
r

() =
_
f

() cos f () sin , f

() sin + f () cos
_
r

() =
_
f

() cos f

() sin f

() sin f () cos ,
f

() sin + f

() cos + f

() cos f () sin
_
=
__
f

() f ()
_
cos 2 f

() sin ,
_
f

() f ()
_
sin +2 f

() cos
_
284 C HA P T E R 14 CALCULUS OF VECTOR- VALUED FUNCTI ONS (ET CHAPTER 13)
Hence,
r

() r

() =
_
f

() cos f () sin
_

__
f

() f ()
_
sin +2 f

() cos
_
k

_
f

() sin + f () cos
_

__
f

() f ()
_
cos 2 f

() sin
_
k
=
_
f

()
_
f

() f ()
_
cos sin f ()
_
f

() f ()
_
sin
2
+2 f
2
() cos
2

2 f () f

() sin cos
_
f

()
_
f

() f ()
_
sin cos f ()
_
f

() f ()
_
cos
2

+2 f

()
2
sin
2
+2 f () f

() cos sin
__
k
=
_
f ()
_
f

() f ()
_ _
sin
2
+cos
2

_
+2 f
2
()
_
cos
2
+sin
2

_
_
k
=
_
f ()
_
f

() f ()
_
+2 f
2
()
_
k
=
_
f () f

() + f
2
() +2 f
2
()
_
k
The length of the cross product is:
r

() r

() = | f
2
() +2 f

2
() f () f

()| (2)
We compute the length of r

():
r

()
2
=
_
f

() cos f () sin
_
2
+
_
f

() sin + f () cos
_
2
= f

2
() cos
2
2 f

() f () cos sin + f
2
() sin
2
+ f

2
() sin
2

+2 f

() f () sin cos + f
2
() cos
2

= f

2
()
_
cos
2
+sin
2

_
+ f
2
()
_
sin
2
+cos
2

_
= f

2
() + f
2
()
Hence,
r

() =
_
f

2
() + f
2
() (3)
Substituting (2) and (3) in (1) gives:
() =
| f
2
() +2 f

2
() f () f

()|
_
f

2
() + f
2
()
_
3/2
In Exercises 6466, use Eq. (12) to nd the curvature of the curve given in polar form.
f () = 2 cos
65. f () =
SOLUTION We have f

() = 1, f

() = 0. The numerator and denominator in Eq. (12) are thus:


f ()
2
+2 f

() f () f

() =
2
+2 1 0 =
2
+2
_
f ()
2
+ f

()
2
_
3/2
=
_

2
+1
_
3/2
Hence,
() =

2
+2
_

2
+1
_
3/2
f () = e

67. Use Eq. (12) to nd the curvature of the general Bernoulli spiral r = ae
b
in polar form (a and b are constants).
SOLUTION By Eq. (12):
() =
| f ()
2
+2 f

()
2
f () f

()|
_
f ()
2
+ f

2
()
_
3/2
In our case f () = ae
b
hence f

() = abe
b
and f

() = ab
2
e
b
. We compute the numerator of ():
f ()
2
+2 f

()
2
f () f

() = a
2
e
2b
+2a
2
b
2
e
2b
ae
b
ab
2
e
b
= a
2
e
2b
+2a
2
b
2
e
2b
a
2
b
2
e
2b
S E C T I O N 14.4 Curvature (ET Section 13.4) 285
= a
2
e
2b
+a
2
b
2
e
2b
= a
2
(1 +b
2
)e
2b
We compute the denominator of ():
_
f ()
2
+ f

()
2
_
3/2
=
_
a
2
e
2b
+a
2
b
2
e
2b
_
3/2
=
_
a
2
e
2b
_
1 +b
2
__
3/2
= a
3
e
3b
_
1 +b
2
_
3/2
Therefore:
() =
a
2
(1 +b
2
)e
2b
a
3
(1 +b
2
)
3/2
e
3b
=
1
a
_
1 +b
2
e
b
Show that both r

(t ) and r

(t ) lie in the osculating plane for a vector function r(t ). Hint: Differentiate r

(t ) =
v(t )T(t ).
69. Show that
(s) =
1

N +
1

_
(sin s)T (cos s)N
_
is an arc length parametrization of the osculating circle.
SOLUTION Let P be a xed point on the curve C, T and N are the unit tangent and the unit normal to the curve at
P. We place the xy-coordinate system so that the origin is at P and the x and y axes are in the directions of T and N,
respectively. We next show that (s) is an arc length parametrization of the osculating circle at P.
P
T
N
We compute the following expression:
_
_
_
_
(s)
1

N
_
_
_
_
2
=
1

2
(sin s) T (cos s) N
2
=
1

2
((sin s) T (cos s) N) ((sin s) T (cos s) N)
=
1

2
_
sin
2
sT T (sin s cos s) T N (cos s sin s) N T +
_
cos
2
s
_
N N
_
The vectors T and N are orthogonal unit vectors, hence T N = N T = 0 and T T = T
2
= 1, N N = N
2
= 1.
We use the identity sin
2
(s) +cos
2
(s) = 1 to obtain
_
_
_
_
(s)
1

N
_
_
_
_
2
=
1

2
_
sin
2
s +cos
2
s
_
=
1

2
That is,
_
_
_
_
(s)
1

N
_
_
_
_
=
1

(1)
Notice that , N, and T are xed and only s is changing in (s). It follows by (1) that (s) is a circle of radius
1

centered
at
1

N. The curvature of the circle is the reciprocal of the radius, which is (the curvature of C at the point P). We thus
showed that the circle (s) satises the second condition in the denition of the osculating circle. We now show that the
rst condition is satised as well.
The center of the circle is the terminal point of the vector
1

N, which is in the direction of N and orthogonal to T. This


shows that Tand N are the unit tangent and unit normal to the circle at P. Finally, we verify that the given parametrization
is the arc length parametrization, by showing that

(s) = 1. Differentiating (s) with respect to s gives (notice that


, T, and N are xed):

(s) =
1

_
( cos s) T +( sin s) N
_
= (cos s)T +(sin s)N
Hence, since T T = N N = 1 and T N = N T = 0 we get:

(s)
2
=
_
(cos s)T +(sin s)N
_

_
(cos s)T +(sin s)N
_
=
_
cos
2
s
_
T T +(cos s)(sin s)T N +(sin s cos s)N T +
_
sin
2
s
_
N N
= cos
2
s +sin
2
s = 1
Hence

(s) = 1
286 C HA P T E R 14 CALCULUS OF VECTOR- VALUED FUNCTI ONS (ET CHAPTER 13)
Two vector-valued functions r
1
(s) and r
2
(s) are said to agree to order 2 at s
0
if
r
1
(s
0
) = r
2
(s
0
), r

1
(s
0
) = r

2
(s
0
), r

1
(s
0
) = r

2
(s
0
)
Let r(s) be an arc length parametrization of a path C and let P be the terminal point of r(0). Let (s) be the arc
length parametrization of the osculating circle given in Exercise 69. Show that r(s) and (s) agree to order 2 at s = 0
(in fact, the osculating circle is the unique circle that approximates C to order 2 at P).
71. Let r(t ) = x(t ), y(t ), z(t ) be a path with curvature (t ) and dene the scaled path r
1
(t ) = x(t ), y(t ), z(t ),
where = 0 is a constant. Prove that curvature varies inversely with the scale factor, that is, the curvature
1
(t ) of r
1
(t )
is
1
(t ) =
1
(t ). This explains why the curvature of a circle of radius R is proportional to 1/R (in fact, it is equal to
1/R). Hint: Use Eq. (3).
SOLUTION The resulting curvature k
1
and the original curvature are:

1
(t ) =
r

1
(t ) r

1
(t )
r

1
(t )
3
, (t ) =
r

(t ) r

(t )
r

(t )
3
We have
r

1
(t ) =
d
dt
(r(t )) = r

(t )
r

1
(t ) =
d
dt
_
r

1
(t )
_
=
d
dt
_
r

(t )
_
= r

(t )
Hence,
r

1
(t ) r

1
(t ) = r

(t ) r

(t ) =
2
r

(t ) r

(t )
r

1
(t ) = r

(t ) = ||r

(t )
Substituting in (1) we get:

1
(t ) =

2
r

(t ) r

(t )
||
3
r

(t )
3
=
1
||
r

(t ) r

(t )
r

(t )
3
=
1
||
(t )
We conclude that the resulting curvature is:

1
(t ) =
1
||
(t )
Multiplying the vector by causes the curvature to be divided by ||.
Further Insights and Challenges
Show that the curvature of Vivianis curve, given by r(t ) = 1 +cos t, sin t, 2 sin(t /2), is
(t ) =

13 +3 cos t
(3 +cos t )
3/2
73. Let r(s) be an arc length parametrization of a closed curve C of length L. We call C an oval if d/ds > 0 (see
Exercise 59). Observe that N points to the outside of C. For k > 0, the curve C
1
dened by r
1
(s) = r(s) kN is called
the expansion of c(s) in the normal direction.
(a) Show that r

1
(s) = r

(s) +k(s).
(b) As P moves around the oval counterclockwise, increases by 2 [Figure 20(A)]. Use this and a change of variables
to prove that
_
L
0
(s) ds = 2.
(c) Show that C
1
has length L +2k.
(A) An oval
T
N
C
1
is the expansion of C
in normal direction.
(B)
C
1
C
x
y
P P
FIGURE 20 As P moves around the oval, increases by 2.
SOLUTION
(a) Since r
1
(s) = r(s) kN we have
r

1
(s) = r

(s) k
dN
ds
(1)
We compute
dN
ds
using the Chain Rule:
dN
ds
=
dN
d

d
ds
(2)
By Exercise 59 and since C is oval we have:
(s) =

d
ds

=
d
ds
(3)
S E C T I O N 14.4 Curvature (ET Section 13.4) 287
T
N
Also, as illustrated in the gure, the following holds:
N =
_
cos
_

2
+
_
, sin
_

2
+
__
= sin , cos
Hence:
dN
d
= cos , sin = cos , sin = T (4)
Substituting (3) and (4) in (2) yields:
dN
ds
= (s)T(s)
Substituting in (1) we obtain:
r

1
(s) = r

(s) +k(s)T(s)
In the arc length parametrization, T(s) = r

(s), therefore:
r

1
(s) = r

(s) +k(s)r

(s) = r

(s) (1 +k(s))
Computing the length and using r

(s) = 1 we obtain:
r

1
(s) = r

(s) (1 +k(s)) = r

(s) +r

(s) k(s) = r

(s) +k(s)
(b) In Exercise 59 we showed that:
(s) =

d
ds

Since
d
ds
> 0 we have (s) =
d
ds
. As P moves around the oval, increases by 2, hence (s = L) (s = 0) = 2.
Using these considerations we get:
_
L
0
(s) ds =
_
(L)
(0)
d
ds
ds =
_
(L)
(0)
d = (L) (0) = 2.
(c) We use the Arc Length Formula and the equality in part (a) to write the length L
1
of C
1
as the following integral:
L
1
=
_
L
0
r

1
(s) ds =
_
L
0
r

(s) ds +k
_
L
0
(s) ds
By the Arc Length Formula, the rst integral is the length L of C. The second integral was computed in part (b). Therefore
we get:
L
1
= L +k 2 = L +2k.
In Exercises 7481, let B denote the binormal vector at a point on a space curve C, dened by B = T N.
Show that B is a unit vector.
75. Follow steps (a)(c) to prove that there is a number (lowercase Greek tau) called the torsion such that
dB
ds
= N 13
(a) Show that
dB
ds
= T
dN
ds
and conclude that dB/ds is orthogonal to T.
(b) Differentiate B B = 1 with respect to s to show that dB/ds is orthogonal to B.
(c) Conclude that dB/ds is a multiple of N.
SOLUTION
288 C HA P T E R 14 CALCULUS OF VECTOR- VALUED FUNCTI ONS (ET CHAPTER 13)
(a) Using the Product Rule for cross product we have:
dB
ds
=
d
ds
(T N) =
dT
ds
N +T
dN
ds
N is a unit vector in the direction of
dT
ds
, hence
dT
ds
N = 0, so we obtain:
dB
ds
= T
dN
ds
By properties of cross products we conclude that
dB
ds
is orthogonal to T.
(b) We differentiate B B = 1 using the Product Rule for dot products:
B
dB
ds
+
dB
ds
B = 0
2B
dB
ds
= 0 B
dB
ds
= 0
Since the dot product of B and
dB
ds
is zero, the two vectors are orthogonal.
(c) In parts (a) and (b) we showed that
dB
ds
is orthogonal to B and T. It follows that
dB
ds
is parallel to any other vector
that is orthogonal to B and T. We show that N is such a vector.
Since B = T N, the vectors N and B are orthogonal. The unit normal N is also orthogonal to the unit tangent T. We
conclude that
dB
ds
and N are parallel, hence there exists a number () such that:
dB
ds
= N.
Show that if C is contained in a plane P, then B is a unit vector normal to P. Conclude that = 0 for a plane
curve.
77. Torsion means twisting. Is this an appropriate name for ? Explain by interpreting geometrically.
SOLUTION B is the unit normal to the osculating plane at a point P on the curve. As P moves along the curve, the
unit normal B is changing by
dB
ds
= N. Geometrically the osculating plane is twisted and is a measure for this
twisting.
Use the identity
a (b c) = (a c)b (a b)c
to prove
N B = T, B T = N
79. Follow steps (a)(b) to prove
dN
ds
= T +B 15
(a) Show that dN/ds is orthogonal to N. Conclude that dN/ds lies in the plane spanned by T and B, and hence,
dN/ds = aT +bB for some scalars a, b.
(b) Use N T = 0 to show that T
dN
ds
= N
dT
ds
and compute a. Compute b similarly.
Equations (13) and (15) together with dT/dt = N are called the Frenet formulas and were discovered by the French
geometer Jean Frenet (18161900).
SOLUTION
(a) We rst show that
dN
ds
is orthogonal to N. Earlier we showed that
dB
ds
= T
dN
ds
and
dB
ds
= N, hence:
N = T
dN
ds
By properties of the cross product, this equality implies that
dN
ds
is orthogonal to N, hence it is orthogonal to N. Now,
N is orthogonal to T and B, hence N is normal to the plane spanned by T and B. Therefore, since N is orthogonal to
dN
ds
,
this last vector lies in the plane spanned by T and B, that is, there exist scalars a and b such that:
dN
ds
= aT +bB
(b) By the orthogonality of N and T we have:
N T = 0
Differentiating this equality, using the product rule for dot product we get:
N
dT
ds
+
dN
ds
T = 0 T
dN
ds
= N
dT
ds
To compute a, we substitute
dN
ds
= aT +bB and use T T = T
2
= 1 and T B = 0. This gives:
T (aT +bB) = N
dT
ds
S E C T I O N 14.4 Curvature (ET Section 13.4) 289
aT T +bT B = N
dT
ds
a 1 +b 0 = N
dT
ds
a = N
dT
ds
(1)
To nd b we differentiate the equality N B = 0 (notice that by B = T N follows the orthogonality of N and B). We
get:
N
dB
ds
+
dN
ds
B = 0
dN
ds
B = N
dB
ds
We now substitute
dN
ds
= aT +bB and we use B B = B
2
= 1 and T B = 0 to obtain:
(aT +bB) B = N
dB
ds
aT B +bB B = N
dB
ds
a 0 +b 1 = N
dB
ds
b = N
dB
ds
Since
dB
ds
= N we may write:
b = N (N) = N N = N
2
= (2)
Also for the arc length parametrization
dT
ds
= (s)N, hence by (1):
a = N (s)N = (s)N N = (s)N
2
= (s) (3)
We combine (2), (3), and part (a) to conclude:
dN
ds
= T +B.
Show that r

is a multiple of B. Conclude that


B =
r

81. The vector N can be computed using N = B T [Eq. (14)] with B, as in Eq. (16). Use this method to nd N in the
following cases:
(a) r(t ) =
_
cos t, t, t
2
_
at t = 0
(b) r(t ) =
_
t
2
, t
1
, t
_
at t = 1
SOLUTION
(a) We rst compute the vector B using Eq. (16):
B =
r

(1)
Differentiating r(t ) =
_
cos t, t, t
2
_
gives
r

(t ) = sin t, 1, 2t
r

(t ) = cos t, 0, 2

r

(0) = 0, 1, 0
r

(0) = 1, 0, 2
We compute the cross product:
r

(0) r

(0) = j (i +2k) = j i +2j k = k +2i = 2, 0, 1


r

(0) r

(0) =
_
2
2
+0
2
+1
2
=

5
Substituting in (1) we obtain:
B(0) =
2, 0, 1

5
=
1

5
2, 0, 1
We now compute T(0):
T(0) =
r

(0)
r

(0)
=
0, 1, 0
0, 1, 0
= 0, 1, 0
Finally we nd N = B T:
N(0) =
1

5
2, 0, 1 0, 1, 0 =
1

5
(2i +k) j =
1

5
(2i j +k j) =
1

5
(2k i) =
1

5
1, 0, 2
290 C HA P T E R 14 CALCULUS OF VECTOR- VALUED FUNCTI ONS (ET CHAPTER 13)
(b) Differentiating r(t ) =
_
t
2
, t
1
, t
_
gives
r

(t ) =
_
2t, t
2
, 1
_
r

(t ) =
_
2, 2t
3
, 0
_
r

(1) = 2, 1, 1
r

(1) = 2, 2, 0
We compute the cross product:
r

(1) r

(1) =

i j k
2 1 1
2 2 0

1 1
2 0

2 1
2 0

j +

2 1
2 2

k = 2i +2j +6k = 2, 2, 6
r

(1) r

(1) =
_
(2)
2
+2
2
+6
2
=

44 = 2

11
Substituting in (1) gives:
B(1) =
2, 2, 6
2

11
=
1

11
1, 1, 3
We now nd T(1):
T(1) =
r

(1)
r

(1)
=
2, 1, 1

4 +1 +1
=
1

6
2, 1, 1
Finally we nd N(1) by computing the following cross product:
N(1) = B(1) T(1) =
1

11
1, 1, 3
1

6
2, 1, 1 =
1

66

i j k
1 1 3
2 1 1

=
1

66
_

1 3
1 1

1 3
2 1

j +

1 1
2 1

k
_
=
1

66
(4i +7j k) =
1

66
4, 7, 1
14.5 Motion in Three-Space (ET Section 13.5)
Preliminary Questions
1. If a particle travels with constant speed, must its acceleration vector be zero? Explain.
SOLUTION If the speed of the particle is constant, the tangential component, a
T
(t ) = v

(t ), of the acceleration is zero.


However, the normal component, a
N
(t ) = (t )v(t )
2
is not necessarily zero, since the particle may change its direction.
2. For a particle in uniform circular motion around a circle, which of the vectors v(t ) or a(t ) always points toward the
center of the circle?
SOLUTION For a particle in uniform circular motion around a circle, the acceleration vector a(t ) points towards the
center of the circle, whereas v(t ) is tangent to the circle.
3. Two objects travel to the right along the parabola y = x
2
with nonzero speed. Which of the following must be true?
(a) Their velocity vectors point in the same direction.
(b) Their velocity vectors have the same length.
(c) Their acceleration vectors point in the same direction.
SOLUTION
(a) The velocity vector points in the direction of motion, hence the velocities of the two objects point in the same
direction.
(b) The length of the velocity vector is the speed. Since the speeds are not necessarily equal, the velocity vectors may
have different lengths.
(c) The acceleration is determined by the tangential component v

(t ) and the normal component (t )v(t )


2
. Since v and
v

may be different for the two objects, the acceleration vectors may have different directions.
4. Use the decomposition of acceleration into tangential and normal components to explain the following statement: If
the speed is constant, then the acceleration and velocity vectors are orthogonal.
SOLUTION If the speed is constant, v

(t ) = 0. Therefore, the acceleration vector has only the normal component:


a(t ) = a
N
(t )N(t )
The velocity vector always points in the direction of motion. Since the vector N(t ) is orthogonal to the direction of
motion, the vectors a(t ) and v(t ) are orthogonal.
S E C T I O N 14.5 Motion in Three-Space (ET Section 13.5) 291
5. If a particle travels along a straight line, then the acceleration and velocity vectors are (choose the correct statement):
(a) Orthogonal (b) Parallel
SOLUTION Since a line has zero curvature, the normal component of the acceleration is zero, hence a(t ) has only the
tangential component. The velocity vector is always in the direction of motion, hence the acceleration and the velocity
vectors are parallel to the line. We conclude that (b) is the correct statement.
6. What is the length of the acceleration vector of a particle traveling around a circle of radius 2 cm with constant
velocity 4 cm/s?
SOLUTION The acceleration vector is given by the following decomposition:
a(t ) = v

(t )T(t ) +(t )v(t )


2
N(t ) (1)
In our case v(t ) = 4 is constant hence v

(t ) = 0. In addition, the curvature of a circle of radius 2 is (t ) =


1
2
. Substituting
v(t ) = 4, v

(t ) = 0 and (t ) =
1
2
in (1) gives:
a(t ) =
1
2
4
2
N(t ) = 8N(t )
The length of the acceleration vector is, thus,
a(t ) = 8 cm/s
2
7. Two cars are racing around a circular track. If, at a certain moment, both of their speedometers read 110 mph. then
the two cars have the same (choose one):
(a) a
T
(b) a
N
SOLUTION The tangential acceleration a
T
and the normal acceleration a
N
are the following values:
a
T
(t ) = v

(t ); a
N
(t ) = (t )v(t )
2
At the moment where both speedometers read 110 mph, the speeds of the two cars are v = 110 mph. Since the track is
circular, the curvature (t ) is constant, hence the normal accelerations of the two cars are equal at this moment. Statement
(b) is correct.
Exercises
1. Use the table here to calculate the difference quotients
r(1 +h) r(1)
h
for h = 0.2, 0.1, 0.1, 0.2. Then estimate
the velocity and speed at t = 1.
r(0.8) 1.557, 2.459, 1.970
r(0.9) 1.559, 2.634, 1.740
r(1) 1.540, 2.841, 1.443
r(1.1) 1.499, 3.078, 1.035
r(1.2) 1.435, 3.342, 0.428
SOLUTION
(h = 0.2)
r(1 0.2) r(1)
0.2
=
r(0.8) r(1)
0.2
=
1.557, 2.459, 1.970 1.540, 2.841, 1.443
0.2
=
0.017, 0.382, 0.527
0.2
= 0.085, 1.91, 2.635
(h = 0.1)
r(1 0.1) r(1)
0.1
=
r(0.9) r(1)
0.1
=
1.559, 2.634, 1.740 1.540, 2.841, 1.443
0.1
=
0.019, 0.207, 0.297
0.1
= 0.19, 2.07, 2.97
(h = 0.1)
r(1 +0.1) r(1)
0.1
=
r(1.1) r(1)
0.1
=
1.499, 3.078, 1.035 1.540, 2.841, 1.443
0.1
=
0.041, 0.237, 0.408
0.1
= 0.41, 2.37, 4.08
292 C HA P T E R 14 CALCULUS OF VECTOR- VALUED FUNCTI ONS (ET CHAPTER 13)
(h = 0.2)
r(1 +0.2) r(1)
0.2
=
r(1.2) r(1)
0.2
=
1.435, 3.342, 0.428 1.540, 2.841, 1.443
0.2
=
0.105, 0.501, 1.015
0.2
= 0.525, 2.505, 5.075
The velocity vector is dened by:
v(t ) = r

(t ) = lim
h0
r(t +h) r(t )
h
We may estimate the velocity at t = 1 by:
v(1) 0.3, 2.2, 3.5
and the speed by:
v(1) = v(1)
_
0.3
2
+2.2
2
+3.5
2
= 4.1
Draw the vectors r(2 + h) r(2) and
r(2 +h) r(2)
h
for h = 0.5 for the path in Figure 9. Draw v(2) (using a
rough estimate for its length).
In Exercises 36, calculate the velocity and acceleration vectors and the speed at the time indicated.
3. r(t ) =
_
t
3
, 1 t, 4t
2
_
, t = 1
SOLUTION In this case r(t ) =
_
t
3
, 1 t, 4t
2
_
hence:
v(t ) = r

(t ) =
_
3t
2
, 1, 8t
_
v(1) = 3, 1, 8
a(t ) = r

(t ) = 6t, 0, 8 a(1) = 6, 0, 8
The speed is the magnitude of the velocity vector, that is,
v(1) = v(1) =
_
3
2
+(1)
2
+8
2
=

74
r(t ) = e
t
j cos(2t )k, t = 0
5. r() = sin , cos , cos 3, =

3
SOLUTION Differentiating r() = sin , cos , cos 3 gives:
v() = r

() = cos , sin , 3 sin 3


v
_

3
_
=
_
cos

3
, sin

3
, 3 sin
_
=
_
1
2
,

3
2
, 0
_
a() = r

() = sin , cos , 9 cos 3


a
_

3
_
=
_
sin

3
, cos

3
, 9 cos
_
=
_

3
2
,
1
2
, 9
_
The speed is the magnitude of the velocity vector, that is:
v
_

3
_
=
_
_
_v
_

3
__
_
_ =

_
_
1
2
_
2
+
_

3
2
_
2
+0
2
= 1
r(s) =
_
1
1 +s
2
,
s
1 +s
2
_
, s = 2
7. Find a(t ) for a particle moving around a circle of radius 8 cm at a constant speed of v = 4 cm/s (see Example 2).
Draw the path and acceleration vector at t =

4
.
SOLUTION The position vector is:
r(t ) = 8 cos t, sin t
Hence,
v(t ) = r

(t ) = 8 sin t, cos t = 8 sin t, cos t (1)


We are given that the speed of the particle is v = 4cm/s. The speed is the magnitude of the velocity vector, hence:
v = 8
_
(sin t)
2
+cos
2
t = 8 = 4 =
1
2
rad/s
S E C T I O N 14.5 Motion in Three-Space (ET Section 13.5) 293
Substituting in (2) we get:
v(t ) = 4
_
sin
t
2
, cos
t
2
_
We now nd a(t ) by differentiating the velocity vector. This gives
a(t ) = v

(t ) = 4
_

1
2
cos
t
2
,
1
2
sin
t
2
_
= 2
_
cos
t
2
, sin
t
2
_
The path of the particle is r(t ) = 8
_
cos
t
2
, sin
t
2
_
and the acceleration vector at t =

4
is:
a
_

4
_
= 2
_
cos

8
, sin

8
_
1.85, 0.77
The path r(t ) and the acceleration vector at t =

4
are shown in the following gure:
8
r(t ) = 8
_
cos
t
2
, sin
t
2
_
Sketch the path r(t ) =
_
1 t
2
, 1 t
_
for 2 t 2, indicating the direction of motion. Draw the velocity and
acceleration vectors at t = 0 and t = 1.
9. Sketch the path r(t ) =
_
t
2
, t
3
_
together with the velocity and acceleration vectors at t = 1.
SOLUTION We compute the velocity and acceleration vectors at t = 1:
v(t ) = r

(t ) =
_
2t, 3t
2
_
v(1) = 2, 3
a(t ) = v

(t ) = 2, 6t a(1) = 2, 6
The following gure shows the path r(t ) =
_
t
2
, t
3
_
and the vectors v(1) and a(1):
x
y
a(1)
r(t) = (t
2
, t
3
)
v(1)
t = 1
The paths r(t ) =
_
t
2
, t
3
_
and r
1
(t ) =
_
t
4
, t
6
_
trace the same curve and r
1
(1) = r(1). Do you expect either the
velocity or acceleration vectors of these paths at t = 1 to point in the same direction? Compute these vectors and
draw them on a single plot of the path.
In Exercises 1114, nd v(t ) given a(t ) and the initial velocity.
11. a(t ) =
_
t, 4
_
, v(0) =
_
1
3
, 2
_
SOLUTION We nd v(t ) by integrating a(t ):
v(t ) =
_
t
0
a(u)du =
_
t
0
u, 4 du =
_
1
2
u
2
, 4u
_

t
0
+v
0
=
_
t
2
2
, 4t
_
+v
0
The initial condition gives:
v(0) = 0, 0 +v
0
=
_
1
3
, 2
_
v
0
=
_
1
3
, 2
_
Hence,
v(t ) =
_
t
2
2
, 4t
_
+
_
1
3
, 2
_
=
_
3t
2
+2
6
, 4t 2
_
294 C HA P T E R 14 CALCULUS OF VECTOR- VALUED FUNCTI ONS (ET CHAPTER 13)
a(t ) =
_
e
t
, 0, t +1
_
, v(0) =
_
1, 3,

2
_
13. a(t ) = k, v(0) = i
SOLUTION We compute v(t ) by integrating the acceleration vector:
v(t ) =
_
t
0
a(u) du =
_
t
0
k du = ku

t
0
+v
0
= t k +v
0
(1)
Substituting the initial condition gives:
v(0) = 0k +v
0
= i v
0
= i
Combining with (1) we obtain:
v(t ) = i +t k
a(t ) = t
2
k, v(0) = i j
In Exercises 1518, nd r(t ) and v(t ) given a(t ) and the initial velocity and position.
15. a(t ) = t, 4, v(0) = 3, 2, r(0) = 0, 0
SOLUTION We rst integrate a(t ) to nd the velocity vector:
v(t ) =
_
t
0
u, 4 du =
_
u
2
2
, 4u
_

t
0
+v
0
=
_
t
2
2
, 4t
_
+v
0
(1)
The initial condition v(0) = 3, 2 gives:
v(0) = 0, 0 +v
0
= 3, 2 v
0
= 3, 2
Substituting in (1) we get:
v(t ) =
_
t
2
2
, 4t
_
+3, 2 =
_
t
2
2
+3, 4t 2
_
We now integrate the velocity vector to nd r(t ):
r(t ) =
_
t
0
_
u
2
2
+3, 4u 2
_
du =
_
u
3
6
+3u, 2u
2
2u
_

t
0
+r
0
=
_
t
3
6
+3t, 2t
2
2t
_
+r
0
The initial condition r(0) = 0, 0 gives:
r(0) = 0, 0 +r
0
= 0, 0 r
0
= 0, 0
Hence,
r(t ) =
_
t
3
6
+3t, 2t
2
2t
_
a(t ) =
_
e
t
, 2t, t +1
_
, v(0) = 1, 0, 1, r(0) = 2, 1, 1
17. a(t ) = t k, v(0) = i, r(0) = j
SOLUTION Integrating the acceleration vector gives:
v(t ) =
_
t
0
uk du =
u
2
2
k

t
0
+v
0
=
t
2
2
k +v
0
(1)
The initial condition for v(t ) gives:
v(0) =
0
2
2
k +v
0
= i v
0
= i
We substitute in (1):
v(t ) =
t
2
2
k +i = i +
t
2
2
k
We now integrate v(t ) to nd r(t ):
r(t ) =
_
t
0
_
i +
u
2
2
k
_
du = ui +
u
3
6
k

t
0
+r
0
= t i +
t
3
6
k +r
0
(2)
S E C T I O N 14.5 Motion in Three-Space (ET Section 13.5) 295
The initial condition for r(t ) gives:
r(0) = 0i +0k +r
0
= j r
0
= j
Combining with (2) gives the position vector:
r(t ) = t i +j +
t
3
6
k
a(t ) = cos t k, v(0) = i j, r(0) = i
19. A bullet is red from the ground at an angle of 45

. What initial speed must the bullet have in order to hit the top of
a 400-ft tower located 600 ft away?
SOLUTION We place the gun at the origin and let r(t ) be the bullets position vector.
Step 1. Use Newtons Law. The net force vector acting on the bullet is the force of gravity F = 0, gm = m 0, g.
By Newtons Second Law, F = mr

(t ), hence:
m 0, g = mr

(t ) r

(t ) = 0, g
We compute the position vector by integrating twice:
r

(t ) =
_
t
0
r

(u) du =
_
t
0
0, g du = 0, gt +v
0
r(t ) =
_
t
0
r

(u) du =
_
t
0
(0, gu +v
0
) du =
_
0, g
t
2
2
_
+v
0
t +r
0
That is,
r(t ) =
_
0,
g
2
t
2
_
+v
0
t +r
0
(1)
Since the gun is at the origin, r
0
= 0. The bullet is red at an angle of 45

, hence the initial velocity v


0
points in the
direction of the unit vector
_
cos 45

, sin 45

_
=
_
2
2
,

2
2
_
therefore, v
0
= v
0
_
2
2
,

2
2
_
. Substituting these initial values
in (1) gives:
r(t ) =
_
0,
g
2
t
2
_
+t v
0
_

2
2
,

2
2
_
Step 2. Solve for v
0
. The position vector of the top of the tower is 600, 400, hence at the moment of hitting the tower
we have,
r(t ) =
_
0,
g
2
t
2
_
+t v
0
_

2
2
,

2
2
_
= 600, 400
_
t v
0

2
2
,
g
2
t
2
+

2
2
t v
0
_
= 600, 400
Equating components, we get the equations:

t v
0

2
2
= 600

g
2
t
2
+

2
2
t v
0
= 400
The rst equation implies that t =
1,200

2v
0
. We substitute in the second equation and solve for v
0
(we use g = 32 ft/s
2
):
16
_
1,200

2v
0
_
2
+

2
2
_
1,200

2v
0
_
v
0
= 400
8
_
1,200
v
0
_
2
+600 = 400
_
1,200
v
0
_
2
= 25
1,200
v
0
= 5 v
0
= 240 ft/s
The initial speed of the bullet must be v
0
= 240 ft/s.
A bullet is red from the ground at an angle of 60

with initial speed v


0
= 30 ft/s. How far does the bullet travel?
What is the bullets velocity vector and speed when it hits the ground?
296 C HA P T E R 14 CALCULUS OF VECTOR- VALUED FUNCTI ONS (ET CHAPTER 13)
21. A projectile red at an angle of 60

lands 1,200 ft away. What was its initial speed?


SOLUTION We place the gun at the origin and let r(t ) be the projectiles position vector. The net force acting on the
projectile is:
F = 0, mg = m 0, g = m 0, 32
By Newtons Second Law, F = mr

(t ), hence:
m 0, 32 = mr

(t ) r

(t ) = 0, 32
We integrate twice to obtain:
v(t ) =
_
t
0
r

(u) du =
_
t
0
0, 32 du = 0, 32t +v
0
r(t ) =
_
t
0
v(u) du =
_
t
0
(0, 32u +v
0
) du =
_
0, 16t
2
_
+v
0
t +r
0
(1)
Since the gun is at the origin, r
0
= 0. The ring is at an angle of 60

, hence the initial velocity points in the direction of


the unit vector
_
cos 60

, sin 60

_
=
_
1
2
,

3
2
_
. Hence,
v
0
= v
0
_
1
2
,

3
2
_
=
_
v
0
2
,

3v
0
2
_
Substituting the initial vectors in (1) we get:
r(t ) =
_
0, 16t
2
_
+
_
v
0
2
,

3v
0
2
_
t =
_
v
0
t
2
, 16t
2
+

3v
0
2
t
_
At the moment of landing, the x-component of r(t ) is 1200, and the y-component is zero. The corresponding equations
are:

v
0
2
t = 1,200
16t
2
+

3v
0
2
t = 0
The rst equation implies that v
0
t = 2,400. We substitute in the second equation and solve for t :
16t
2
+

3
2
2,400 = 0
16t
2
= 1,200

3
t
2
129.9 t 11.4 s
By v
0
t = 2,400 we get v
0
=
2,400
11.4
210.53 ft/s.
Show that a projectile red at an angle with initial speed v
0
travels a total distance (v
2
0
/g) sin 2 before hitting
the ground. Conclude that the maximum distance (for a given v
0
) is attained for = 45

.
23. A baseball is thrown to another player standing 80 ft away with initial speed 60 ft/s. Use the result of Exercise 22 to
nd two angles at which the ball can be released. Which angle gets the ball there faster?
SOLUTION We suppose that the baseball is thrown from the origin, and that r(t ) is the baseballs position vector. By
Exercise 22 the total distance travelled by the ball is
v
2
0
g
sin 2. Using the given information we obtain the following
equation:
60
2
32
sin 2 = 80
sin 2 =
32 80
60
2
0.711
The solutions for 0 90

are:
2 45.32

22.66

or
2 134.68

67.34

By Newtons Second Law we have:


F = m 0, g = mr

(t ) r

(t ) = 0, g = 0, 32
S E C T I O N 14.5 Motion in Three-Space (ET Section 13.5) 297
Integrating gives:
v(t ) =
_
t
0
r

(u) du =
_
t
0
0, 32 du = 0, 32t +v
0
(1)
The initial velocity points in the direction of the unit vector cos , sin and its magnitude is the initial speed v
0
= 60.
Hence, v
0
= 60 cos , sin . Substituting in (1) we get:
v(t ) = 0, 32t +60 cos , sin (2)
Integrating this vector with respect to t and using r
0
= 0 we obtain:
r(t ) =
_
t
0
v(u) du =
_
t
0
(0, 32u +60 cos , sin ) du =
_
0, 16t
2
_
+60t cos , sin
At the nal time x(t ) = 80. This gives:
x(t ) = 60t cos = 80 t =
4
3 cos
Since we want to minimize t we need to maximize cos , hence, to minimize . Therefore, = 22.66

will get the ball


faster to the other player.
Show that a bullet red at an angle will hit the top of an h-ft tower located d ft away if its initial speed is
v
0
=
4d sec

d tan h
25. At a certain moment, a moving particle has velocity v = 2, 2, 1 and a = 0, 4, 3. Find T, N, and the decompo-
sition of a into tangential and normal components.
SOLUTION We go through the following steps:
Step 1. Compute T and a
T
. The unit tangent is the following vector:
T =
v
v
=
2, 2, 1
_
2
2
+2
2
+(1)
2
=
1
3
2, 2, 1 (1)
The tangential component of a = 0, 4, 3 is:
a
T
= a T = 0, 4, 3
1
3
2, 2, 1 =
1
3
(0 +8 3) =
5
3
Step 2. Compute a
N
and N. Since a
N
N = a a
T
T, we have:
a
N
N = 0, 4, 3
5
3

1
3
2, 2, 1 = 0, 4, 3
_
10
9
,
10
9
,
5
9
_
=
1
9
10, 26, 32 (2)
The unit normal N is a unit vector, therefore:
a
N
= a
N
N =
1
9
_
(10)
2
+26
2
+32
2
=
1
9
30

2 =
10

2
3
(3)
We compute N, using (3) and (4):
N =
a
N
N
a
N
=
1
9
10, 26, 32
10

2
3
=
1
15

2
5, 13, 16
Step 3. Write the decomposition. Using (1)(4) we obtain the following decomposition:
a = a
T
T +a
N
N
0, 4, 3 =
5
3
T +
10

2
3
N,
where T =
_
2
3
,
2
3
,
1
3
_
and N =
1
15

2
5, 13, 16.
At a certain moment, a moving particle has velocity v = 12, 20, 20 and acceleration a = 2, 1, 3. Is the
particle speeding up or slowing down?
27. A particle follows a path r(t ) for 0 t T, beginning at the origin O. The vector v =
1
T
_
T
0
r

(t ) dt is called the
average velocity vector. Suppose that v = 0. Answer and explain the following:
(a) Where is the particle located at time T if v = 0?
(b) Is the particles average speed necessarily equal to zero?
SOLUTION
(a) If the average velocity is 0, then the particle must be back at its original position at time t = T. This is perhaps best
seen by noting that v =
1
T
_
T
0
r

(t ) dt = r(t )

T
0
.
298 C HA P T E R 14 CALCULUS OF VECTOR- VALUED FUNCTI ONS (ET CHAPTER 13)
(b) The average speed need not be zero! Consider a particle moving at constant speed around a circle, with position
vector r(t ) = cos t, sin t . From 0 to 2, this has average velocity of 0, but constant average speed of 1.
One consequence of Keplers Laws of Planetary Motion is that a planet moves faster when it is closer to the
sun. Which of Figure 10(A) or 10(B) represents a planetary orbit (with the velocity vectors as shown)?
29. A space shuttle orbits the earth at an altitude 200 miles above the earths surface, with constant speed v =
17,000 mph. Find the magnitude of the shuttles acceleration (in ft/s
2
), assuming that the radius of the earth is 4,000
miles (Figure 11).
FIGURE 11 Space shuttle orbit.
SOLUTION The shuttle is in a uniform circular motion, therefore the tangential component of its acceleration is zero,
and the acceleration can be written as:
a = v
2
N (1)
The radius of motion is 4000 + 200 = 4200 miles hence the curvature is =
1
4200
. Also by the given information the
constant speed is v = 17,000 mph. Substituting these values in (1) we get:
a =
1
4200
17,000
2
N = 6.88 10
4
miles/h
2
The magnitude of the shuttles acceleration is thus:
a = 6.88 10
4
miles/h
2
In units of ft/s
2
we obtain
a =
6.88 10
4
5.28 10
3
3600
2
= 28.03 ft/s
2
In Exercises 3033, use (4) and (5) to nd a
T
and a
N
as a function of t or at the point indicated.
r(t ) =
_
t
2
, t
3
_
31. r(t ) =
_
t, cos t, sin t
_
SOLUTION We nd a
T
and a
N
using the following equalities:
a
T
= a T, a
N
=
a v
v
.
We compute v and a by differentiating r twice:
v(t ) = r

(t ) = 1, sin t, cos t v(t ) =


_
1 +(sin t )
2
+cos
2
t =

2
a(t ) = r

(t ) = 0, cos t, sin t
The unit tangent vector T is, thus:
T(t ) =
v(t )
v(t )
=
1

2
1, sin t, cos t
Since the speed is constant (v = v(t ) =

2), the tangential component of the acceleration is zero, that is:
a
T
= 0
To nd a
N
we rst compute the following cross product:
a v =

i j k
0 cos t sin t
1 sin t cos t

cos t sin t
sin t cos t

0 sin t
1 cos t

j +

0 cos t
1 sin t

k
=
_
cos
2
t +sin
2
t
_
i sin t j +cos t k = i sin t j +cos t k = 1, sin t, cos t
S E C T I O N 14.5 Motion in Three-Space (ET Section 13.5) 299
Hence,
a
N
=
a v
v
=
_
(1)
2
+(sin t )
2
+cos
2
t

2
=

2
= 1.
r(t ) =
_
t
1
, ln t, t
2
_
, t = 1
33. r(t ) =
_
e
t
, t, e
t
_
, t = 0
SOLUTION We use the following equalities:
a
T
= a T, a
N
=
a v
v
We rst nd v and a by twice differentiating r. This gives:
v(t ) = r

(t ) =
_
e
t
, 1, e
t
_
v(0) = 1, 1, 1 , v(0) =

3
a(t ) = r

(t ) =
_
e
t
, 0, e
t
_
a(0) = 1, 0, 1
The unit tangent at t = 0 is, thus:
T(0) =
v(0)
v(0)
=
1

3
1, 1, 1
We now nd a
T
at the point t = 0:
a
T
= a T = 1, 0, 1
1

3
1, 1, 1 =
1

3
(1 +0 1) = 0
To nd a
N
we rst compute the cross product:
a v = 1, 0, 1 1, 1, 1 =

i j k
1 0 1
1 1 1

0 1
1 1

1 1
1 1

j +

1 0
1 1

k
= 1i +2j +1k = 1, 2, 1
Hence,
a
N
=
a v
v
=
1, 2, 1

3
=

3
=

2
In Exercise 3441, use (4) and (6) to nd the decomposition of a(t ) into tangential and normal components at the point
indicated, as in Example 6.
r(t ) =
_
4 t, t +1, t
2
_
, t = 1
35. r(t ) =
_
t, e
t
, t e
t
_
, t = 0
SOLUTION We have r(t ) =
_
t, e
t
, t e
t
_
. We nd the decomposition of a(t ) into tangential and normal components,
using the following steps.
Step 1. Compute T and a
T
. Differentiating r(t ) twice we obtain:
v(t ) = r

(t ) =
_
1, e
t
, e
t
+t e
t
_
=
_
1, e
t
, (1 +t )e
t
_
v(0) = 1, 1, 1 , v(0) =
_
1
2
+1
2
+1
2
=

3
a(t ) = r

(t ) =
_
0, e
t
, e
t
+(1 +t )e
t
_
=
_
0, e
t
, (2 +t )e
t
_
a(0) = 0, 1, 2
We compute the unit normal vector at t = 0:
T =
v
v
=
1, 1, 1

3
=
1

3
1, 1, 1
Using Eq. (4) we obtain:
a
T
= a T = 0, 1, 2
1

3
1, 1, 1 =
1

3
(0 +1 +2) =
3

3
=

3
Step 2. Compute a
N
and N. By Eq. (6) we have:
a
N
N = a a
T
T = 0, 1, 2

3
1

3
1, 1, 1 = 1, 0, 1
300 C HA P T E R 14 CALCULUS OF VECTOR- VALUED FUNCTI ONS (ET CHAPTER 13)
Since N is a unit vector, the following holds:
a
N
= a
N
N =
_
(1)
2
+0
2
+1
2
=

2
N =
a
N
N
a
N
=
1, 0, 1

2
=
_

2
, 0,
1

2
_
Step 3. Write the decomposition. Using the results obtained in the previous steps, we obtain the following decomposition
of a(0):
a(0) = a
T
(0)T(0) +a
N
(0)N(0) =

3T +

2N
where T = T(0) =
1

3
1, 1, 1 and N = N(0) =
_

2
, 0,
1

2
_
.
r(t ) =
_
t,
1
2
t
2
,
1
6
t
3
_
, t = 1
37. r(t ) =
_
t,
1
2
t
2
,
1
6
t
3
_
, t = 4
SOLUTION
(a) By Eq. (4) we have:
a
T
= a T =
a v
v
We compute v and a by differentiating r(t ) =
_
t,
1
2
t
2
,
1
6
t
3
_
twice. We obtain:
v(t ) = r

(t ) =
_
1, t,
1
2
t
2
_
v(4) = 1, 4, 8 ,
v(4) =
_
1
2
+4
2
+8
2
= 9
a(t ) = r

(t ) = 0, 1, t a(4) = 0, 1, 4
Hence,
a
T
(4) =
a(4) v(4)
v(4)
=
0, 1, 4 1, 4, 8
9
=
36
9
= 4
(b) By Eq. (6) we have:
a
N
N = a a
T
T
Since T(4) =
v(4)
v(4)
=
1,4,8
9
=
_
1
9
,
4
9
,
8
9
_
and by part (a) a
T
(4) = 4, and a(4) = 0, 1, 4, we have:
a
N
N = 0, 1, 4 4
_
1
9
,
4
9
,
8
9
_
=
_

4
9
,
7
9
,
4
9
_
We see that a
N
N is a unit vector and conclude that:
N(4) = a
N
N =
_

4
9
,
7
9
,
4
9
_
and a
N
(4) = 1.
(c) The acceleration vector a(4) can be written as the following decomposition:
a(4) = a
T
(4)T(4) +a
N
(4)N(4)
In parts (a) and (b) we found that a
T
(4) = 4, T(4) =
_
1
9
,
4
9
,
8
9
_
, a
N
(4) = 1, and N(4) =
_

4
9
,
7
9
,
4
9
_
. This gives the
following decomposition:
a(4) = 4T(4) +1N(4) = 4T(4) +N(4) =
_
4
9
,
16
9
,
32
9
_
+
_

4
9
,
7
9
,
4
9
_
= 0, 1, 4
r(t ) =
_
e
t
, 1 t
_
, t = 0
39. r() = cos , sin , , = 0
SOLUTION We have r(t ) = cos t, sin t, t . We nd the decomposition of a(0), using the following steps:
Step 1. Compute T and a
T
. We differentiate r(t ) twice to obtain:
v(t ) = r

(t ) = sin t, cos t, 1
a(t ) = r

(t ) = cos t, sin t, 0 a(0) = 1, 0, 0


S E C T I O N 14.5 Motion in Three-Space (ET Section 13.5) 301
Computing the magnitude of v(t ) (the speed) we obtain:
v(t ) = v(t ) =
_
(sin t )
2
+(cos t )
2
+1
2
=

1 +1 =

2
Since the speed is constant, the tangential component of the acceleration a(t ) is zero. That is:
a
T
(t ) = 0
Step 2. Compute a
N
and N. Since a
T
(t ) = 0, we obtain the decomposition:
a(t ) = a
N
(t )N(t ) a(0) = a
N
(0)N(0) (1)
Notice that the vector a(0) = 1, 0, 0 is already a unit vector, hence (1) implies that N(0) = a(0) and a
N
(0) = 1.
Hence the required decomposition reduces to:
a(0) = 1 N(0) = 1 N where N = a(0) = 1, 0, 0
r(t ) =
_
1
3
t
3
, 1 3t
_
, t = 1
41. r(t ) = t, cos t, t sin t , t =

2
SOLUTION In this case, r(t ) = t, cos t, t sin t . We nd the decomposition of a
_

2
_
, using the following steps:
Step 1. Compute T and a
T
. We differentiate r(t ) twice to obtain:
v(t ) = r

(t ) = 1, sin t, sin t +t cos t


v
_

2
_
= 1, 1, 1 ,
v
_

2
_
=

3
a(t ) = r

(t ) = 0, cos t, cos t +cos t t sin t = 0, cos t, 2 cos t t sin t


a
_

2
_
=
_
0, 0,

2
_
The unit tangent at t =

2
is, thus:
T
_

2
_
=
v
_

2
_
_
_
v
_

2
__
_
=
1, 1, 1

3
=
1

3
1, 1, 1
Using Eq. (4) we get at t =

2
:
a
T
= a T =
_
0, 0,

2
_

3
1, 1, 1 =
1

3

_

2
_
=

2

3
Step 2. Compute a
N
and N. By Eq. (6) we have at t =

2
:
a
N
N = a a
T
T =
_
0, 0,

2
_

3
_

3
1, 1, 1 =
_
0, 0,

2
_
+

6
1, 1, 1
=
_

6
,

6
,

3
_
=

6
1, 1, 2
Since N is a unit vector we get:
a
N
= a
N
N =

6
_
1
2
+(1)
2
+(2)
2
=

6
N =
a
N
N
a
N
=

6
1, 1, 2

6
=
1

6
1, 1, 2
Step 3. Write the decomposition. Using the results obtained in the previous steps we obtain the following decomposition
of a
_

2
_
:
a = a
T
T +a
N
N =

2

3
T +

6
N
where T =
1

3
1, 1, 1 and N =
1

6
1, 1, 2.
Let r(t ) =
_
t
2
, 4t 3
_
. Find T(t ) and N(t ), and show that the decomposition of a(t ) into tangential and normal
components is
a(t ) =
_
2t
_
2
_
T +
_
4
2
+4
_
N
43. Find the components a
T
and a
N
of the acceleration vector of a particle moving along a circular path of radius
R = 100 cm with constant velocity v
0
= 5 cm/s.
302 C HA P T E R 14 CALCULUS OF VECTOR- VALUED FUNCTI ONS (ET CHAPTER 13)
SOLUTION Since the particle moves with constant speed, we have v

(t ) = 0, hence:
a
T
= v

(t ) = 0
The normal component of the acceleration is a
N
= (t )v(t )
2
. The curvature of a circular path of radius R = 100 is
(t ) =
1
R
=
1
100
, and the velocity is the constant value v(t ) = v
0
= 5. Hence,
a
N
=
1
R
v
2
0
=
25
100
= 0.25 cm/s
2
At time t
0
, a moving particle has velocity vector v = 2i and acceleration vector a = 3i + 18k. Determine the
curvature (t
0
) of the particles path at time t
0
.
45. A car proceeds along a circular path of radius R = 1,000 ft centered at the origin. Starting at rest, its speed increases
at a rate of t ft/s
2
. Find the acceleration vector a at time t = 3 s and determine its decomposition into normal and
tangential components.
SOLUTION The acceleration vector can be decomposed into tangential and normal directions as follows:
a(t ) = a
T
(t )T(t ) +a
N
(t )N(t ) (1)
where
a
T
(t ) = v

(t ) and a
N
(t ) = (t )v(t )
2
(2)
Since the speed v(t ) is increasing at a rate of t ft/s
2
, we have v

(t ) = t . The car starts at rest hence the initial speed is


v
0
= 0. We now integrate to nd v(t ):
v(t ) =
_
t
0
v

(u) du =
_
t
0
u du =
1
2
t
2
+v
0
=
1
2
t
2
+0 =
1
2
t
2
The curvature of the circular path is (t ) =
1
R
=
1
1000
. Substituting v

(t ) = t , =
1
1000
, and v(t ) =
1
2
t
2
in (2) gives:
a
T
(t ) = t, a
N
(t ) =
1
1000
_
1
2
t
2
_
2
=
1
4000
t
4
Combining with (1) gives the following decomposition:
a(t ) = t T(t ) +
1
4000
t
4
N(t ) (3)
We now nd the unit tangent T(t ) and the unit normal N(t ).
N
T
Q (starting
point)
P
O
We have (see gure):
T =
_
cos
_

2
+
_
, sin
_

2
+
__
= sin , cos (4)
N = cos ( +) , sin ( +) = cos , sin (5)
We use the arc length formula to nd :

PQ=
_
t
0
r

(u) du =
_
t
0
v(u) du =
_
t
0
1
2
u
2
du =
t
3
6
In addition,

PQ= R = 1000. Hence,


1,000 =
t
3
6
=
t
3
6,000
Substituting in (4) and (5) yields:
T =
_
sin
t
3
6,000
, cos
t
3
6,000
_
; N =
_
cos
t
3
6,000
, sin
t
3
6,000
_
(6)
S E C T I O N 14.5 Motion in Three-Space (ET Section 13.5) 303
We now combine (3) and (6) to obtain the following decomposition:
a(t ) = t
_
sin
t
3
6,000
, cos
t
3
6,000
_
+
1
4,000
t
4
_
cos
t
3
6,000
, sin
t
3
6,000
_
At t = 3 we get:
a
T
= 3a
N
=
3
4
4,000
0.02025
T =
_
sin
3
3
6,000
, cos
3
3
6,000
_
0.0045, 0.9999
N =
_
cos
3
3
6,000
, sin
3
3
6,000
_
0.9999, 0.0045
In the notation of Example 5, nd the acceleration vector for a person seated in a car at (a) the highest point of
the ferris wheel and (b) the two points level with the center of the wheel.
47. Suppose that r = r(t ) lies on a sphere of radius R for all t . Let J = r r

. Show that r

= (J r)/r
2
. Hint:
Observe that r and r

are perpendicular.
SOLUTION
(a) Solution 1. Since r = r(t ) lies on the sphere, the vectors r = r(t ) and r

= r

(t ) are orthogonal, therefore:


r r

= 0 (1)
We use the following well-known equality:
a (b c) = (a c) b (a b) c
Using this equality and (1) we obtain:
J r =
_
r r

_
r = r
_
r r

_
=
__
r r

_
r (r r) r

_
=
_
r r

_
r +r
2
r

= 0r +r
2
r

= r
2
r

Divided by the scalar r


2
we obtain:
r

=
J r
r
2
(b) Solution 2. The cross product J = r r

is orthogonal to r and r

. Also, r and r

are orthogonal, hence the vectors r,


r

and J are mutually orthogonal. Now, since r

is orthogonal to r and J, the right-hand rule implies that r

points in the
direction of J r. Therefore, for some > 0 we have:
r

= J r = r


J r
J r
(2)
By properties of the cross product and since J, r, and r

are mutually orthogonal we have:


J r = Jr = r r

r = rr

r = r
2
r

Substituting in (2) we get:


r

= r

J r
r
2
r

=
J r
r
2
A particle moves counterclockwise around a circle. Which of the vectors in Figure 12 is not a possible acceler-
ation vector? Explain. For the two possible acceleration vectors, state whether the particle is speeding up or slowing
down.
Further Insights and Challenges
49. The orbit of a planet is an ellipse with the sun at one focus. The suns gravitational force acts along the
radial line from the planet to the sun (the dashed lines in Figure 13), and by Newtons Second Law, the acceleration
vector points in the same direction. Explain in words why the planet must slow down in the upper half of the orbit (as
it moves away from the sun) and speed up in the lower half. Keplers Second Law, discussed in the next section, gives
a more precise version of this qualitative conclusion. Hint: Consider the decomposition of a into normal and tangential
components.
304 C HA P T E R 14 CALCULUS OF VECTOR- VALUED FUNCTI ONS (ET CHAPTER 13)
Planetary motion
Sun
N
N
N
a
a
FIGURE 13 Elliptical orbit of a planet around the sun.
SOLUTION In the upper half of the orbit, as the planet moves away from the sun the acceleration vector has a negative
component in the tangential direction T, so the particles velocity is decreasing (since a
T
(t ) = v

(t ) < 0).
T
T
a
a
Sun
However, in the lower half of the orbit, as the planet gets closer to the sun, the acceleration has a positive component in
the tangential direction, that is, a
T
(t ) = v

(t ) > 0. Therefore the velocity v(t ) is increasing.


In Exercises 5054, consider a car of mass m traveling along a curved but level road. To avoid skidding, the road must
supply a frictional force F = ma, where a is the cars acceleration vector. The maximum magnitude of the frictional
force is mg, where is the coefcient of friction and g = 32 ft/s
2
. Let v be the cars speed in feet per second.
Show that the car will not skid if the curvature of the road is such that (with R = 1/)
(v

)
2
+
_
v
2
R
_
2
< (g)
2
Note that braking (v

< 0) and speeding up (v

> 0) contribute equally to skidding.


51. Suppose that the maximum radius of curvature along a curved highway is R = 600 ft. How fast can a car travel (at
constant speed) along the highway without skidding if the coefcient of friction is = 0.5?
SOLUTION In Exercise 50 we showed that the car will not skid if the following inequality is satised:
_
v

_
2
+
v
4
R
2
<
2
g
2
We compute the constant speed v for which the car can travel without skidding. In case of constant speed, v

= 0. We
substitute R = 600, = 0.5 and g = 32 and solve for v. This gives:
v
4
600
2
< 0.5
2
32
2
v
4
< 9,600
2
v < 98 ft/s
The maximum speed (in case of constant speed) is about 98 ft/s.
Beginning at rest, a car drives around a circular track of radius R = 1,000 ft, accelerating at a rate of 1 ft/s
2
.
After how many seconds will the car begin to skid if the coefcient of friction is = 0.6?
53. You want to reverse your direction in the shortest possible time by driving around a semicircular bend (Figure 14).
If you travel at the maximum possible constant speed v that will not cause skidding, is it faster to hug the inside curve
(radius r) or the outside curb (radius R)? Hint: Use Eq. (7) to show that at maximum speed, the time required to drive
around the semicircle is proportional to the square root of the radius.
r
R
FIGURE 14 Car going around the bend.
SOLUTION In Exercise 50 we showed that the car will not skid if the following inequality is satised:
_
v

_
2
+
v
4
R
2
<
2
g
2
In case of constant speed, v

= 0, so the inequality becomes:


v
4
R
2
<
2
g
2
S E C T I O N 14.6 Planetary Motion According to Kepler and Newton (ET Section 13.6) 305
We solve for v:
v
4
<
_
gR
_
2
v <
_
gR
The maximum speed in which skidding does not occur is, thus,
v
_
gR (1)
If T is the time required to drive around the semicircle of radius R at the constant speed v, then the length of the semicircle
can be written as:
R =
_
T
0
r

(t ) dt =
_
T
0
v dt = vT
Hence,
T =
R
v
(2)
Combining (1) and (2) gives:
T
R
_
gR

R
We conclude that it is faster to hug the inside curve of radius r (r < R), rather than the outside curve of radius R.
What is the smallest radius R about which a car can turn without skidding at 60 mph if = 0.75 (a typical
value)?
14.6 Planetary Motion According to Kepler and Newton (ET Section 13.6)
Preliminary Questions
1. Describe the relation between the vector J = r r

and the rate at which the radial vector sweeps out area.
SOLUTION The rate at which the radial vector sweeps out area equals half the magnitude of the vector J. This relation
is expressed in the formula:
d A
dt
=
1
2
J.
2. Equation (1) shows that r

is proportional to r. Explain how this fact is used to prove Keplers Second Law.
SOLUTION In the proof of Keplers Second Law it is shown that the rate at which area is swept out is
d A
dt
=
1
2
J, where J = r(t ) r

(t )
To show that J is constant, show that J is constant. This is done using the proportionality of r

and r which implies


that r(t ) r

(t ) = 0. Using this we get:


dJ
dt
=
d
dt
_
r r

_
= r r

+r

= 0 +0 = 0 J = const
3. How is the period T affected if the semimajor axis a is increased four-fold?
SOLUTION Keplers Third Law states that the period T of the orbit is given by:
T
2
=
_
4
2
GM
_
a
3
or
T =
2

GM
a
3/2
If a is increased four-fold the period becomes:
2

GM
(4a)
3/2
= 8
2

GM
a
3/2
That is, the period is increased eight-fold.
306 C HA P T E R 14 CALCULUS OF VECTOR- VALUED FUNCTI ONS (ET CHAPTER 13)
Exercises
1. Keplers Third Law states that T
2
/a
3
has the same value for each planetary orbit. Do the data in the following table
support this conclusion? Estimate the length of Jupiters period, assuming that a = 77.8 10
10
m.
Planet Mercury Venus Earth Mars
a (10
10
m) 5.79 10.8 15.0 22.8
T (years) 0.241 0.615 1.00 1.88
SOLUTION Using the given data we obtain the following values of T
2
/a
3
, where a, as always, is measured not in
meters but in 10
10
m:
Planet Mercury Venus Earth Mars
T
2
/a
3
2.99 10
4
3 10
4
2.96 10
4
2.98 10
4
The data on the planets supports Keplers prediction. We estimate Jupiters period (using the given a) as T

a
3
3 10
4
11.9 years.
A satellite has initial position r = 1,000, 2,000, 0 and initial velocity r

= 1, 2, 2 (units of kilometers and


seconds). Find the equation of the plane containing the satellites orbit. Hint: This plane is orthogonal to J.
3. The earths orbit is nearly circular with radius R = 93 10
6
miles (the eccentricity is e = 0.017). Find the rate at
which the earths radial vector sweeps out area in units of ft
2
/s. What is the magnitude of the vector J = r r

for the
earth (in units of squared feet per second)?
SOLUTION The rate at which the earths radial vector sweeps out area is
d A
dt
=
1
2
J; J = r(t ) r

(t ) (1)
Since J is a constant vector, its length is constant. Moreover, if we assume that the orbit is circular then r(t ) lies on a
circle, and therefore r(t ) and r

(t ) are orthogonal. Using properties of the cross product we get:


J = r(t ) r

(t ) = r(t )r

(t ) = Rr

(t ) = const
We conclude that the speed v = r

(t ) is constant. We nd the speed using the following equality:


2R = vT v =
2R
T
.
Therefore,
J = R
2R
T
=
2R
2
T
.
Substituting in (1) we get:
d A
dt
=
1
2

2R
2
T
=
R
2
T
.
For R = 93 10
6
miles = 4.9 10
11
ft and T = 365 24 3,600 = 31,536,000 s we obtain:
J =
2 (4.9 10
11
)
2
31,536,000
= 4.78 10
16
ft
2
/s
d A
dt
= 2.39 10
16
ft
2
/s
Finding the Mass of a Star Using Keplers Third Law, show that if a planet revolves around a star with period
T and semimajor axis a, then the mass of the star is M =
_
4
2
G
__
a
3
T
2
_
.
5. Ganymede, one of Jupiters moons discovered by Galileo, has an orbital period of 7.154 days and a semimajor axis
of 1.07 10
9
m. Use Exercise 4 to estimate the mass of Jupiter.
SOLUTION By Exercise 4, the mass of Jupiter can be computed using the following equality:
M =
4
2
G
a
3
T
2
We substitute the given data T = 7.154 24 60
2
= 618,105.6 a = 1.07 10
9
m and G = 6.67300
10
11
m
3
kg
1
s
1
, to obtain:
M =
4
2

_
1.07 10
9
_
3
6.67300 10
11
(618,105.6)
2
1.897 10
27
kg.
An astronomer observes a planet orbiting a star with a period of 9.5 years and a semimajor axis of 3 10
8
km.
Find the mass of the star.
S E C T I O N 14.6 Planetary Motion According to Kepler and Newton (ET Section 13.6) 307
7. Use the fact that J is constant to show that a planet in a circular orbit travels at constant speed.
SOLUTION It is shown in the proof of Keplers Second Law that the vector J = r(t ) r

(t ) is constant, hence its


length is constant:
J = r(t ) r

(t ) = const (1)
We consider the orbit as a circle of radius R, therefore, r(t ) and r

(t ) are orthogonal and r(t ) = R. By (1) and using


properties of the cross product we obtain:
r(t ) r

(t ) = r(t )r

(t ) sin

2
= R r

(t ) = const
We conclude that r

(t ) is constant, that is the speed v = r

(t ) of the planet is constant.


Prove that if a planetary orbit is circular, then vT = 2R, where v is the planets speed (constant by Exercise 7)
and T is the period. Then use Keplers Third Law to prove that v =
_
GM
R
.
9. Show directly that the circular orbit
r(t ) = R cos t, R sin t
satises the differential equation, Eq. (2), provided that
2
= k R
3
. Then deduce Keplers Third Law T
2
=
_
4
2
k
_
R
3
for this orbit.
SOLUTION Note that r = R, and note that
r

= R sin t, R cos t and r

=
_
R
2
cos t, R
2
sin t
_
We rewrite this as:
r

=
2
R cos t, R sin t =
2
r
Since
2
= k/R
3
and R = r, we get r

=
k
r
3
r, as desired. Since T =
2

then T
2
=
4
2

2
=
4
2
R
3
k
, as desired.
The orbit of a satellite orbiting above the equator of the earth is called geosynchronous if the period is T = 24
hours (in this case, the satellite stays over a xed point on the equator). Use Keplers Third Law to nd the altitude
h above the earths surface of a geosynchronous orbit. The earth has mass M 5.974 10
24
kg and radius
R 6,371 km.
11. Use the results of Exercises 8 and 10 to nd the velocity of a satellite in geosynchronous orbit.
SOLUTION In Exercise 8 we showed that the velocity of a planet in a circular orbit of radius a is:
v =
2a
T
(1)
A geosynchronous orbit has period T = 24 hours and in Exercise 10 we found that a = 42,246 km. Substituting in (1)
we get:
v =
2 42,246
24
= 11,060 km/h
Show that if a planet revolves around a star of mass M in a circular orbit of radius R with speed v, then
M =
Rv
2
G
.
13. Mass of the Milky Way The sun revolves around the center of mass of the Milky Way galaxy in an orbit that
is approximately circular, of radius a 2.8 10
17
km and velocity v 250 km/s. Use the result of Exercise 12 to
estimate the mass of the portion of the Milky Way inside the suns orbit (place all of this mass at the center of the orbit).
SOLUTION Let M be the mass of the portion of the Milky Way inside the suns orbit, assuming that all this mass is at
the center of the suns orbit. By Exercise 12, the following equality holds:
M =
av
2
G
.
We substitute the values a = 2.8 10
20
m, v = 250 10
3
m/s and G = 6.673 10
11
m
3
kg
1
s
1
and compute the
mass M. This gives:
M =
2.8 10
20
(250 10
3
)
2
6.673 10
11
= 2.6225 10
41
kg.
The mass of the sun is 1.989 10
30
kg, hence M is 1.32 10
11
times the mass of the sun (132 billions times the mass
of the sun).
Conservation of Energy The total mechanical energy (kinetic plus potential) of a planet of mass m orbiting a
sun of mass M with position r and speed v = r

is
E =
1
2
mv
2

GMm
r
Use (2) and (9) to show that E is conserved, that is,
dE
dt
= 0.
15. Show that the total energy (11) of a planet in a circular orbit of radius R is E = GMm/(2R). Hint: Use Exercise
8.
308 C HA P T E R 14 CALCULUS OF VECTOR- VALUED FUNCTI ONS (ET CHAPTER 13)
SOLUTION The total energy of a planet in a circular orbit of radius R is
E =
1
2
mv
2

GMm
r
=
1
2
mv
2

GMm
R
(1)
In Exercise 8 we showed that
v
2
=
GM
R
(2)
Substituting (2) in (1) we obtain:
E =
1
2
m
GM
R

GMm
R
=
1
2
GMm
R
=
GMm
2R
.
In Exercises 1620, we consider a planetary orbit with orbital parameters p and e. The perihelion and aphelion of the
orbits are the points on the orbit closest to and farthest from the sun (Figure 7). Denote the distances from the sun at the
perihelion and aphelion by r
per
and r
ap
and the speeds of the planet at the perihelion and aphelion by v
per
and v
ap
.
F
2
F
1
y
x
O
Semimajor axis
Aphelion
Perihelion
v
per
r
v
ap
a
FIGURE 7 r and r

are perpendicular at the perihelion and aphelion.


Use the polar equation
r =
p
1 +e cos
to show that r
per
= a(1 e) and r
ap
= a(1 +e).
17. Compute r
per
and r
ap
for the orbit of Mercury, which has eccentricity e = 0.244 (see the table in Exercise 1 for the
semimajor axis).
SOLUTION The length of the semi-major axis of the orbit of mercury is a = 5.79 10
7
km. We substitute a and e =
0.244 in the formulas for r
per
and r
ap
obtained in Exercise 16, to obtain the shortest and longest distances respectively.
This gives:
r
per
= a(1 e) = 5.79 10
7
(1 0.244) = 4.377 10
7
km
r
ap
= a(1 +e) = 5.79 10
7
(1 +0.244) = 7.203 10
7
km.
Prove the formulas
e =
r
ap
r
per
r
ap
+r
per
, p =
2r
ap
r
per
r
ap
+r
per
19. Prove that v
per
(1 e) = v
ap
(1 +e). Hint: r r

is constant by Eq. (5). Compute this cross product at the perihelion


and aphelion, noting that r is perpendicular to r

at these two points.


SOLUTION Since the vector J(t ) = r(t ) r

(t ) is constant, it is the same vector at the perigee and at the apogee, hence
we may equate the length of J(t ) at these two points. Since at the perigee and at the apogee r(t ) and r

(t ) are orthogonal
we have by properties of the cross product:
r
ap
r

ap
= r
ap
r

ap
= r
ap
v
ap
r
per
r

per
= r
per
r

per
= r
per
v
per
Equating the two values gives:
r
ap
v
ap
= r
per
v
per
(1)
In Exercise 16 we showed that r
per
= a(1 e) and r
ap
= a(1 +e). Substituting in (1) we obtain:
a(1 +e)v
ap
= a(1 e)v
per
(1 +e)v
ap
= (1 e)v
per
Prove that v
per
=
_
_
GM
a
_
1 +e
1 e
as follows:
(a) Use Conservation of Energy (see Exercise 14) to show that
v
2
per
v
2
ap
= 2GM(r
1
per
r
1
ap
)
(b) Show r
1
per
r
1
ap
=
2e
a(1 e
2
)
using Exercise 16.
(c) Show that v
2
per
v
2
ap
= 4
e
(1 +e)
2
v
2
per
using Exercise 19. Then solve for v
per
using (a) and (b).
21. Show that the total mechanical energy E of a planet in an elliptical orbit with semimajor axis a is E =
GMm
2a
.
Hint: Use Exercise 20 to compute the total energy at the perihelion.
SOLUTION The total energy of a planet of mass m orbiting a sun of mass M with position r and speed v = r

is
(given in Exercise 14):
E =
1
2
mv
2

GMm
r
(1)
S E C T I O N 14.6 Planetary Motion According to Kepler and Newton (ET Section 13.6) 309
The energy E is conserved, so we can compute it using any point on the elliptical orbit, for instance the perihelion. By
Exercise 16 and Exercise 20 we have:
r
per
= a(1 e)
v
per
=
_
GM
a
1 +e
1 e
(2)
Substituting (2) into (1) gives:
E =
1
2
m
GM
a
1 +e
1 e

GMm
a(1 e)
=
GMm
a(1 e)
_
1 +e
2
1
_
=
GMm
a(1 e)
1 +e 2
2
=
GMm
a(1 e)
e 1
2
=
GMm
2a
Prove that v
2
= GM
_
2
r

1
a
_
at any point on an elliptical orbit with semimajor axis a, where r = r.
23. Two space shuttles A and B orbit the earth along the solid trajectory in Figure 8. Hoping to catch up to B,
the pilot of A applies a forward thrust to increase her shuttles kinetic energy. Use Exercise 21 to show that shuttle A will
move off into a larger orbit as shown in the gure. Then use Keplers Third Law to show that As orbital period T will
increase (and she will fall farther and farther behind B)!
B
Earth A
FIGURE 8
SOLUTION In Exercise 21 we showed that the total mechanical energy E of a planet in an elliptical orbit with semi-
major axis a is
E =
GMm
2a
(1)
Since E is increased, a is increased, resulting in moving to an elliptic orbit as the dashed orbit in the gure. Now, by
Keplers Third Law,
T
2
=
_
4
2
GM
_
a
3
We conclude that the orbital period T of shuttle A is also increasing, which means that A will get further and further
behind B.
Further Insights and Challenges
In this exercise, we prove Eq. (3) in a rigorous fashion. Let r(t ) = r(t ) and let (t ) be the angle between r(t )
and the x-axis. Then r = r(t ) cos (t ), sin (t ).
(a) Prove that
d A
dt
=
1
2
r(t )
2

(t ) by applying the Fundamental Theorem of Calculus to the formula for area in


polar coordinates:
A() =
1
2
_

0
r(u)
2
du
(b) Show that J = r r

= r(t )
2

(t ).
(c) Conclude that
d A
dt
=
1
2
J.
In Exercises 2526, we prove Keplers Third Law. Figure 9 shows an elliptical orbit with polar equation
r =
p
1 +e cos
where p = J
2
/k. Let a and b be the semimajor and semiminor axes, respectively. The origin is located at F
1
.
F
2
F
1
B
A
Semimajor axis
Semiminor axis
a
a a
b
FIGURE 9
25. The goal of this exercise is to show that b =

pa.
(a) Show that F
1
A + F
2
A = 2a. Conclude that F
1
B + F
2
B = 2a and hence F
1
B = F
2
B = a.
(b) Show that F
1
A =
p
1 +e
and F
2
A =
p
1 e
, and conclude that a =
p
1 e
2
.
(c) Use the Pythagorean Theorem to prove that
b =
p
_
1 e
2
=

pa
310 C HA P T E R 14 CALCULUS OF VECTOR- VALUED FUNCTI ONS (ET CHAPTER 13)
SOLUTION
(a) Since CF
2
= AF
1
, we have:
F
2
A = CA CF
2
= 2a F
1
A
Therefore,
F
1
A + F
2
A = 2a (1)
0
C A
B
F
2
F
1
The ellipse is the set of all points such that the sum of the distances to the two foci F
1
and F
2
is constant. Therefore,
F
1
A + F
2
A = F
1
B + F
2
B (2)
Combining (1) and (2), we obtain:
F
1
B + F
2
B = 2a (3)
The triangle F
2
BF
1
is isosceles, hence F
2
B = F
1
B and so we conclude that
F
1
B = F
2
B = a
(b) The polar equation of the ellipse, where the focus F
1
is at the origin is
r =
p
1 +e cos
0
C A
r
B
F
2
F
1
The point A corresponds to = 0, hence,
F
1
A =
p
1 +e cos 0
=
p
1 +e
(4)
The point C corresponds to = hence,
F
1
C =
p
1 +e cos
=
p
1 e
We now nd F
2
A. Using the equality CF
2
= AF
1
we get:
F
2
A = F
2
F
1
+ F
1
A = F
2
F
1
+ F
2
C = F
1
C =
p
1 e
That is,
F
2
A =
p
1 e
(5)
Combining (1), (4), and (5) we obtain:
p
1 +e
+
p
1 e
= 2a
Hence,
a =
1
2
_
p
1 +e
+
p
1 e
_
=
p(1 e) + p(1 +e)
2(1 +e)(1 e)
=
2p
2
_
1 e
2
_ =
p
1 e
2
S E C T I O N 14.6 Planetary Motion According to Kepler and Newton (ET Section 13.6) 311
(c) We use Pythagoras Theorem for the triangle OBF
1
:
OB
2
+ OF
2
1
= BF
2
1
(6)
0
A
B
b
F
1
a
Using (4) we have
OF
1
= a F
1
A = a
p
1 +e
Also OB = b and BF
1
= a, hence (6) gives:
b
2
+
_
a
p
1 +e
_
2
= a
2
We solve for b:
b
2
+a
2

2ap
1 +e
+
p
2
(1 +e)
2
= a
2
b
2

2ap
1 +e
+
p
2
(1 +e)
2
= 0
In part (b) we showed that a =
p
1e
2
. We substitute to obtain:
b
2

2p
1 +e

p
1 e
2
+
p
2
(1 +e)
2
= 0
b
2
=
2p
2
(1 +e)
2
(1 e)

p
2
(1 +e)
2
=
2p
2
p
2
(1 e)
(1 +e)
2
(1 e)
=
p
2
(1 +e)
(1 +e)
2
(1 e)
=
p
2
1 e
2
Hence,
b =
p
_
1 e
2
Since 1 e
2
=
p
a
we also have
b =
p
_
p
a
=

ap
We prove Keplers Third Law by computing the area A of the ellipse in two ways.
(a) Use Exercise 25 to show that A = (

p)a
3/2
.
(b) Use Keplers First Law to show that A =
1
2
JT.
(c) Deduce that T
2
=
4
2
GM
a
3
.
27. Let e
r
= cos , sin and e

= sin , cos . Write the position vector of a planet as r = re


r
, where r = r.
(a) Show that
de

d
= e
r
.
(b) Write Eq. (2) in the form
dv
dt
=
k
r
2
e
r
and use the Chain Rule to show that
dv
d
d
dt
=
k
r
2
de

d
(c) Show that
dv
d
=
k
J
de

d
. Hint: Use Exercise 24 to show that
d
dt
= J/r
2
.
(d) Conclude that there is a constant vector w such that
v() =
k
J
e

+w
This shows that as varies from 0 to 2, the velocity vector v traces out a circle of radius k/J with center at the
terminal point of w (Figure 10).
312 C HA P T E R 14 CALCULUS OF VECTOR- VALUED FUNCTI ONS (ET CHAPTER 13)
A D
w
C
B
v( ) v( )
D
Planetary orbit
A
C
Velocity circle
B
FIGURE 10 The terminal point of the velocity vector traces out the circle as the planet travels along its orbit.
SOLUTION
(a) Differentiating the vector e

= sin , cos with respect to gives:


de

d
=
d
d
sin , cos = cos , sin = cos , sin = e
r
(b) Eq. (2) is the following equality:
r

(t ) =
k
r
3
r(t ) =
GM
r
3
r(t )
Writing r = re
r
and r

(t ) =
d
dt
r

(t ) =
dv
dt
we get:
dv
dt
=
GM
r
3
re
r
=
GM
r
2
e
r
=
GM
r
2
e
r
That is,
dv
dt
=
GM
r
2
e
r
(1)
By the Chain Rule,
dv
dt
=
dv
d

d
dt
and by part (a)
de

d
= e
r
. Substituting in (1), we get:
dv
d
d
dt
=
GM
r
2
de

d
(c) In Exercise 24 we showed that
2
d A
dt
= r r

= J
d A
dt
=
1
2
J
and
d A
dt
=
1
2
r
2
d
dt
Combining the two equalities we get:
1
2
r
2
d
dt
=
1
2
J
d
dt
=
J
r
2
Substituting in the equality obtained in part (b) we obtain:
dv
d
J
r
2
=
GM
r
2
de

d
Denoting C =
GM
J
we obtain:
dv
d
=
GM
J
de

d
= C
de

d
(d) Integrating the two sides of
dv
d
= C
de

d
we have
v() =
_
dv
d
d = C
_
de

d
d = Ce

+u
where u is a constant vector. Notice that v() u = Ce

= |C|, which is the equation of a circle of radius C


(recall, C = GM/J = k/J) centered at the terminal point of u.
Chapter Review Exercises 313
CHAPTER REVIEW EXERCISES
1. Determine the domains of the vector-valued functions.
(a) r
1
(t ) =
_
t
1
, (t +1)
1
, sin
1
t
_
(b) r
2
(t ) =
_
_
8 t
3
, ln t, e

t
_
SOLUTION
(a) We nd the domain of r
1
(t ) =
_
t
1
, (t +1)
1
, sin
1
t
_
. The function t
1
is dened for t = 0. (t +1)
1
is dened
for t = 1 and sin
1
t is dened for 1 t 1. Hence, the domain of r
1
(t ) is dened by the following inequalities:
t = 0
t = 1 1 < t < 0
1 t 1
or 0 < t 1
(b) We nd the domain of r
2
(t ) =
_
_
8 t
3
, ln t, e

t
_
. The domain of
_
8 t
3
is 8 t
3
0. The domain of ln t is
t > 0 and e

t
is dened for t 0. Hence, the domain of r
2
(t ) is dened by the following inequalities:
8 t
3
0
t > 0
t 0

t
3
8
t > 0
0 < t 2
Sketch the paths r
1
() = , cos and r
2
() = cos , in the xy-plane.
3. Find a vector parametrization of the intersection of the surfaces x
2
+ y
4
+2z
3
= 6 and x = y
2
in R
3
.
SOLUTION We need to nd a vector parametrization r(t ) = x(t ), y(t ), z(t ) for the intersection curve. Using t = y
as a parameter, we have x = t
2
and y = t . We substitute in the equation of the surface x
2
+ y
4
+2z
3
= 6 and solve for
z in terms of t . This gives:
t
4
+t
4
+2z
3
= 6
2t
4
+2z
3
= 6
z
3
= 3 t
4
z =
3
_
3 t
4
We obtain the following parametrization of the intersection curve:
r(t ) =
_
t
2
, t,
3
_
3 t
4
_
.
Find a vector parametrization using trigonometric functions of the intersection of the plane x + y + z = 1 and
the elliptical cylinder
_
y
3
_
2
+
_
z
8
_
2
= 1 in R
3
.
In Exercises 510, calculate the derivative indicated.
5. r

(t ), where r(t ) =
_
1 t, t
2
, ln t
_
SOLUTION We use the Theorem on Componentwise Differentiation to compute the derivative r

(t ). We get
r

(t ) =
_
(1 t )

, (t
2
)

, (ln t )

_
=
_
1, 2t
3
,
1
t
_
r

(1), where r(t ) =


_
t
3
, 4t
2
, 7t
_
7. r

(0), where r(t ) =


_
e
2t
, e
4t
2
, e
6t
_
SOLUTION We differentiate r(t ) componentwise to nd r

(t ):
r

(t ) =
_
(e
2t
)

, (e
4t
2
)

, (e
6t
)

_
=
_
2e
2t
, 8t e
4t
2
, 6e
6t
_
The derivative r

(0) is obtained by setting t = 0 in r

(t ). This gives
r

(0) =
_
2e
20
, 8 0e
40
2
, 6e
60
_
= 2, 0, 6
r

(2), where r(t ) =


_
e
2t
, e
4t
2
, e
6t
_
9.
d
dt
e
t
_
1, t, t
2
_
SOLUTION Using the Product Rule for differentiation gives
d
dt
e
t
_
1, t, t
2
_
= e
t
d
dt
_
1, t, t
2
_
+
_
e
t
_

_
1, t, t
2
_
= e
t
0, 1, 2t +e
t
_
1, t, t
2
_
= e
t
_
0, 1, 2t +
_
1, t, t
2
_
_
= e
t
_
1, 1 +t, 2t +t
2
_
314 C HA P T E R 14 CALCULUS OF VECTOR- VALUED FUNCTI ONS (ET CHAPTER 13)
d
d
r(cos ), where r(s) =
_
s, 2s, s
2
_
In Exercises 1114, calculate the derivative at t = 3 assuming that
r
1
(3) = 1, 1, 0 , r
2
(3) = 1, 1, 0
r

1
(3) = 0, 0, 1 , r

2
(3) = 0, 2, 4
11.
d
dt
(6r
1
(t ) 4 r
2
(t ))
SOLUTION Using Differentiation Rules we obtain:
d
dt
(6r
1
(t ) 4r
2
(t ))

t =3
= 6r

1
(3) 4r

2
(3) = 6 0, 0, 1 4 0, 2, 4
= 0, 0, 6 0, 8, 16 = 0, 8, 10
d
dt
_
e
t
r
2
(t )
_ 13.
d
dt
_
r
1
(t ) r
2
(t )
_
SOLUTION Using Product Rule for Dot Products we obtain:
d
dt
r
1
(t ) r
2
(t ) = r
1
(t ) r

2
(t ) +r

1
(t ) r
2
(t )
Setting t = 3 gives:
d
dt
r
1
(t ) r
2
(t )

t =3
= r
1
(3) r

2
(3) +r

1
(3) r
2
(3) = 1, 1, 0 0, 2, 4 +0, 0, 1 1, 1, 0 = 2 +0 = 2
d
dt
_
r
1
(t ) r
2
(t )
_
15. Calculate
_
3
0
_
4t +3, t
2
, 4t
3
_
dt .
SOLUTION By the denition of vector-valued integration, we have
_
3
0
_
4t +3, t
2
, 4t
3
_
dt =
_
_
3
0
(4t +3) dt,
_
3
0
t
2
dt,
_
3
0
4t
3
dt
_
(1)
We compute the integrals on the right-hand side:
_
3
0
(4t +3) dt = 2t
2
+3t

3
0
= 2 9 +3 3 0 = 27
_
3
0
t
2
dt =
t
3
3

3
0
=
3
3
3
= 9
_
3
0
4t
3
dt = t
4

3
0
= 3
4
= 81
Substituting in (1) gives the following integral:
_
3
0
_
4t +3, t
2
, 4t
3
_
dt = 27, 9, 81
Calculate
_

0
_
sin , , cos 2
_
d.
17. Find the unit tangent vector to r(t ) =
_
sin t, t, cos t
_
at t = .
SOLUTION The unit tangent vector at t = is
T() =
r

()
r

()
(1)
We differentiate r(t ) componentwise to obtain:
r

(t ) = cos t, 1, sin t
Therefore,
r

() = cos , 1, sin = 1, 1, 0
We compute the length of r

():
r

() =
_
(1)
2
+1
2
+0
2
=

2
Chapter Review Exercises 315
Substituting in (1) gives:
T() =
_
1

2
,
1

2
, 0
_
Find the unit tangent vector to r(t ) =
_
t
2
, tan
1
t, t
_
at t = 1.
19. A particle located at (1, 1, 0) at time t = 0 follows a path whose velocity vector is v(t ) =
_
1, t, 2t
2
_
. Find the
particles location at t = 2.
SOLUTION We rst nd the path r(t ) by integrating the velocity vector v(t ):
r(t ) =
_
_
1, t, 2t
2
_
dt =
__
1 dt,
_
t dt,
_
2t
2
dt
_
=
_
t +c
1
,
1
2
t
2
+c
2
,
2
3
t
3
+c
3
_
Denoting by c = c
1
, c
2
, c
3
the constant vector, we obtain:
r(t ) =
_
t,
1
2
t
2
,
2
3
t
3
_
+c (1)
To nd the constant vector c, we use the given information on the initial position of the particle. At time t = 0 it is at
the point (1, 1, 0). That is, by (1):
r(0) = 0, 0, 0 +c = 1, 1, 0
or,
c = 1, 1, 0
We substitute in (1) to obtain:
r(t ) =
_
t,
1
2
t
2
,
2
3
t
3
_
+1, 1, 0 =
_
t +1,
1
2
t
2
+1,
2
3
t
3
_
Finally, we substitute t = 2 to obtain the particles location at t = 2:
r(2) =
_
2 +1,
1
2
2
2
+1,
2
3
2
3
_
=
_
3, 3,
16
3
_
At time t = 2 the particle is located at the point
_
3, 3,
16
3
_
Find the vector-valued function r(t ) =
_
x(t ), y(t )
_
in R
2
satisfying r

(t ) = r(t ) with initial conditions r(0) =


1, 2.
21. Compute the length of the path r(t ) =
_
sin 2t, cos 2t, 3t 1
_
for 1 t 3.
SOLUTION We use the formula for the arc length:
L =
_
3
1
r

(t ) dt (1)
We compute the derivative vector r

(t ) and its length:


r

(t ) = 2 cos 2t, 2 sin 2t, 3


r

(t ) =
_
(2 cos 2t )
2
+(2 sin 2t )
2
+3
2
=
_
4 cos
2
2t +4 sin
2
2t +9
=
_
4
_
cos
2
2t +sin
2
2t
_
+9 =

4 1 +9 =

13
We substitute in (1) and compute the integral to obtain the following length:
L =
_
3
1

13 dt =

13t

3
1
= 2

13.
Express the length of the path r(t ) =
_
ln t, t, e
t
_
for 1 t 2 as a denite integral and use a computer algebra
system to nd its value to two decimal places.
23. A string in the shape of a helix has a height of 20 cm and makes four full rotations over a circle of radius 5 cm. Find
a parametrization r(t ) of the string and compute its length.
316 C HA P T E R 14 CALCULUS OF VECTOR- VALUED FUNCTI ONS (ET CHAPTER 13)
SOLUTION Since the radius is 5 cm and the height is 20 cm, the helix is traced by a parametrization of the form:
r(t ) = 5 cos at, 5 sin at, t , 0 t 20
Since the helix makes exactly 4 full rotations, we have:
a 20 = 4 2 a =
2
5
The parametrization of the helix is, thus:
r(t ) =
_
5 cos
2t
5
, 5 sin
2t
5
, t
_
, 0 t 20
The helix is shown in the following gure:
0
5
5
0
20
15
10
5
0
5
5
To nd the length of the helix, we use the arc length formula:
L =
_
20
0
r

(t ) dt (1)
We nd r

(t ) and its length:


r

(t ) =
_
5
2
5
sin
2t
5
, 5
2
5
cos
2t
5
, 1
_
=
_
2 sin
2t
5
, 2 cos
2t
5
, 1
_
r

(t ) =
_
4
2
sin
2
2t
5
+4
2
cos
2
2t
5
+1 =
_
4
2
_
sin
2
2t
5
+cos
2
2t
5
_
+1 =
_
1 +4
2
Substituting in (1) we get:
L =
_
20
0
_
1 +4
2
dt = 20
_
1 +4
2
127.2
Find the minimum speed of a particle with trajectory r(t ) =
_
t, e
t 3
, e
4t
_
.
25. Calculate the curvature (t ) for r(t ) =
_
t
1
, ln t, t
_
and nd the unit tangent and normal vectors at t = 1.
SOLUTION The unit normal vector is dened by:
N(t ) =
T

(t )
T

(t )
(1)
Since T(t ) =
r

(t )
r

(t )
, we must nd r

(t ) and its length:


r

(t ) =
d
dt
_
t
1
, ln t, t
_
=
_
t
2
,
1
t
, 1
_
=
1
t
2
_
1, t, t
2
_
r

(t ) =
1
t
2
_
1 +t
2
+t
4
(2)
Hence:
T(t ) =
r

(t )
r

(t )
=
_
1
_
1 +t
2
+t
4
,
t
_
1 +t
2
+t
4
,
t
2
_
1 +t
2
+t
4
_
Chapter Review Exercises 317
Setting t = 1 gives:
T(1) =
_

3
,
1

3
,
1

3
_
We now compute T

(t ) and its length:


T

(t ) =
d
dt
_
1
_
1 +t
2
+t
4
,
t
_
1 +t
2
+t
4
,
t
2
_
1 +t
2
+t
4
_
=
_ 2t +4t
3
2

1+t
2
+t
4
1 +t
2
+t
4
,
1
_
1 +t
2
+t
4
t
2t +4t
3
2

1+t
2
+t
4
1 +t
2
+t
4
,
2t
_
1 +t
2
+t
4
t
2

2t +4t
3
2

1+t
2
+t
4
1 +t
2
+t
4
_
=
_
t +2t
3
_
1 +t
2
+t
4
_
3/2
,
1 t
4
_
1 +t
2
+t
4
_
3/2
,
2t +t
3
_
1 +t
2
+t
4
_
3/2
_
T

(t ) =
1
_
1 +t
2
+t
4
_
3/2
_
_
t +2t
3
_
2
+
_
1 t
4
_
2
+
_
2t +t
3
_
2
=
_
t
8
+5t
6
+6t
4
+5t
2
+1
_
1 +t
2
+t
4
_
3/2
Substituting in (1) we get:
N(t ) =
1
_
t
8
+5t
6
+6t
4
+5t
2
+1
_
t +2t
3
, 1 t
4
, 2t +t
3
_
N(1) =
3

18
1, 0, 1
To nd the curvature we use the following formula:
(t ) =
r

(t ) r

(t )
r

(t )
3
(3)
We rst nd r

(t ):
r

(t ) =
d
dt
_
t
2
, t
1
, 1
_
=
_
2t
3
, t
2
, 0
_
=
_
2
t
3
,
1
t
2
, 0
_
We compute the cross product:
r

(t ) r

(t ) =

i j k

1
t
2
1
t
1
2
t
3

1
t
2
0

=
1
t
2
i +
2
t
3
j +
_
1
t
4

2
t
4
_
k =
1
t
2
i +
2
t
3
j
1
t
4
k
Hence:
_
_
r

(t ) r

(t )
_
_
=
_
_
1
t
2
_
2
+
_
2
t
3
_
2
+
_

1
t
4
_
2
=
_
1
t
4
+
4
t
6
+
1
t
8
=
1
t
4
_
t
4
+4t
2
+1 (4)
We now substitute (2) and (4) in (3) to obtain the following curvature:
(t ) =
t
2
_
t
4
+4t
2
+1
__
t
4
+t
2
+1
_
3
Setting t = 1 we get:
(1) =

2
3
A specially trained mouse runs counterclockwise in a circle of radius 2 ft on the oor of an elevator with speed
1 ft/s while the elevator ascends from ground level (along the z-axis) at a speed of 40 ft/s. Find the mouses accel-
eration vector as a function of time. Assume that the circle is centered at the origin of the xy-plane and the mouse is
at (2, 0, 0) at t = 0.
In Exercises 2730, let r(t ) =
_
t, e
t
2 _
.
27. Compute the curvature function (t ).
SOLUTION The curvature is the following function:
(t ) =
r

(t ) r

(t )
r

(t )
3
(1)
318 C HA P T E R 14 CALCULUS OF VECTOR- VALUED FUNCTI ONS (ET CHAPTER 13)
We nd the derivatives of r(t ):
r

(t ) =
_
1, 2t e
t
2 _
r

(t ) =
_
0, 2e
t
2
2t (2t )e
t
2 _
=
_
0, (4t
2
2)e
t
2 _
Therefore:
r

(t ) r

(t ) =
_
i 2t e
t
2
j
_
(4t
2
2)e
t
2
j = (4t
2
2)e
t
2
k =
_
0, 0, (4t
2
2)e
t
2 _
We compute the lengths of the vectors in (1):
r

(t ) =
_
1
2
+
_
2t e
t
2
_
2
=
_
1 +4t
2
e
2t
2
r

(t ) r

(t ) = |4t
2
2|e
t
2
Substituting in (1) gives the following curvature:
(t ) =
|4t
2
2|e
t
2
_
1 +4t
2
e
2t
2
_
3/2
Plot r(t ) and (t ) on the same set of axes and estimate the values of t where the curvature has a maximum
value.
29. Find the unit tangent and normal vectors at t = 0 and t = 1.
SOLUTION The unit tangent vector is dened by:
T(t ) =
r

(t )
r

(t )
(1)
The derivative of r(t ) =
_
t, e
t
2
_
is:
r

(t ) =
_
1, 2t e
t
2 _
At t = 0 and t = 1 we have:
r

(0) =
_
1, 2 0 e
0
2 _
= 1, 0
r

(0) =
_
1
2
+0
2
= 1
r

(1) =
_
1, 2 1 e
1
2 _
=
_
1,
2
e
_
r

(1) =
_
1
2
+
_

2
e
_
2
=
_
1 +
4
e
2
=
1
e
_
e
2
+4
Substituting in (1) we obtain:
T(0) =
r

(0)
r

(0)
= 1, 0
T(1) =
r

(1)
r

(1)
=
_
1,
2
e
_
1
e
_
e
2
+4
=
_
e
_
e
2
+4
,
2
_
e
2
+4
_
The unit normal vector is a unit vector orthogonal to r

(t ) =
_
1, 2t e
t
2
_
. We observe that
_
2t e
t
2
, 1
_
is orthogonal to
r

(t ), since their dot product is zero, hence N(t ) is a unit vector in the direction of
_
2t e
t
2
, 1
_
or
_
2t e
t
2
, 1
_
.
Recall that N(t ) points in the direction of bending. A graph of r(t ) (which is also the graph of y = e
x
2
) helps us
nd the appropriate direction. It is easy to show (by taking y

and y

) that the inection points occur at x = 1/

2.
Thus, we get the following picture:
xx
NN
NN
y
1
0 1
2
1
2
Chapter Review Exercises 319
We conclude that:
N(t ) =

_
2t e
t
2
, 1
_
_
1 +4t
2
e
2t
2
, |t | <
1

2
_
2t e
t
2
, 1
_
_
1 +4t
2
e
2t
2
, |t | >
1

2
For t = 0 and t = 1 we get:
N(0) = 0, 1 , N(1) =
_
2e
1
, 1
_
_
1 +4e
2
=
_
2
_
e
2
+4
,
e
_
e
2
+4
_
Write the acceleration vector at t = 1 as a sum of tangential and normal components.
31. Find the curvature (t ) and unit normal vector N(t ) for r(t ) =
_
sin t, sin t, cos t
_
.
SOLUTION The curvature is the following function:
(t ) =
r

(t ) r

(t )
r

(t )
3
(1)
We compute the derivatives of r(t ) = sin t, sin t, cos t :
r

(t ) = cos t, cos t, sin t ,


r

(t ) = sin t, sin t, cos t


We calculate the cross product:
r

(t ) r

(t ) =

i j k
cos t cos t sin t
sin t sin t cos t

=
_
cos
2
t sin
2
t
_
i
_
cos
2
t sin
2
t
_
j +(cos t sin t +cos t sin t )k
= i +j = 1, 1, 0
We nd the lengths of the vectors in (1):
r

(t ) r

(t ) = 1, 1, 0 =

2
_
_
r

(t )
_
_
=
_
cos
2
t +cos
2
t +sin
2
t =
_
1 +cos
2
t
Substituting in (1) gives the following curvature:
(t ) =

2
_
1 +cos
2
t
_
3/2
The unit normal is the following vector:
N(t ) =
T

(t )
T

(t )
(2)
We nd the unit tangent vector:
T(t ) =
r

(t )
r

(t )
=
1
_
1 +cos
2
t
cos t, cos t, sin t
We differentiate T(t ) using the Product Rule:
T

(t ) =
1
_
1 +cos
2
t
sin t, sin t, cos t +
2 cos t sin t
2

1+cos
2
t
1 +cos
2
t
cos t, cos t, sin t
=
1
_
1 +cos
2
t
sin t, sin t, cos t +
cos t sin t
_
1 +cos
2
t
_
3/2
cos t, cos t, sin t
=
1
_
1 +cos
2
t
_
3/2
_
(1 +cos
2
t )sin t, sin t, cos t +sin t cos t cos t, cos t, sin t
_
=
1
_
1 +cos
2
t
_
3/2

_
sin t, sin t, cos t cos
3
t sin
2
t cos t
_
320 C HA P T E R 14 CALCULUS OF VECTOR- VALUED FUNCTI ONS (ET CHAPTER 13)
=
1
_
1 +cos
2
t
_
3/2
sin t, sin t, 2 cos t
Hence:
T

(t ) =
1
_
1 +cos
2
t
_
3/2
_
2 sin
2
+4 cos
2
t =

2
_
1 +cos
2
t
_
1 +cos
2
t
_
3/2
=

2
1 +cos
2
t
Thus,
N(t ) =
1

2

1
_
1 +cos
2
t
sin t, sin t, 2 cos t
Find the curvature (t ) and unit normal vector N(t ) for r(t ) =
_
ln t, t
1
, t
2
_
.
In Exercises 3334, write the acceleration vector a at the point indicated as a sum of tangential and normal components.
33. r() =
_
cos , sin 2
_
, =

4
SOLUTION
Step 1. Compute T and a
T
. Since r() = cos , sin 2, we have r

() = sin , 2 cos 2 and r

() =
cos , 4 sin 2. Thus, at =

4
,
v = r

4
_
=
_
sin

4
, 2 cos

2
_
=
_

2
, 0
_
T =
v
v
=
_

2
, 0
_
_
_
1

2
_
2
+0
=

2
_

2
, 0
_
= 1, 0
a = r

4
_
=
_
cos

4
, 4 sin

2
_
=
_

2
, 4
_
a
T
= a T =
_

2
, 4
_
1, 0 =
1

2
Step 2. Compute a
N
and N. We have
a
N
N = a a
T
T =
_

2
, 4
_

2
1, 0 = 0, 4
Since N is a unit vector, we can nd a
N
by:
a
N
= a
N
N = 0, 4 = 4
Therefore,
N =
a
N
N
a
N
=
0, 4
4
= 0, 1
Step 3. Write the decomposition. We found that a
T
=
1

2
and a
N
= 4, hence the decomposition of a is
a = a
T
T +a
N
N =
1

2
T +4N
where T = 1, 0 and N = 0, 1.
r(t ) =
_
t
2
, t
3
_
, t = 2
35. Find the osculating circle to the curve y = e
x
2
at x = 0.
SOLUTION We rst nd the curvature, using the formula for the curvature of a graph in the plane:
(x) =
|y

(x)|
_
1 + y

(x)
2
_
3/2
(1)
Since y = e
x
2
, we have:
y

(x) = 2xe
x
2
,
y

(x) = 2e
x
2
+4x
2
e
x
2
= 2
_
2x
2
1
_
e
x
2
Chapter Review Exercises 321
Substituting in (1) we obtain the following curvature at x = 0:
(x) =
|y

(0)|
_
1 + y

(0)
2
_
3/2
=
| 2|
(1 +0)
3/2
= 2 (2)
We now nd the osculating circle at x = 0. The radius of the osculating circle is R =
1
(0)
=
1
2
, and since the graph of
y = e
x
2
is symmetric about the y-axis, so also must be the osculating circle (at x = 0). The circle touches the graph at
(0, 1), is symmetric about the y-axis, and has radius
1
2
, so it must be centered at
_
0,
1
2
_
. Therefore it is parametrized by,
c(t ) =
_
0,
1
2
_
+
1
2
cos t, sin t , 0 t 2
Find the osculating circle to the curve y = ln x at x = 1.
37. If a planet is in orbit around a sun whose mass is zero, Newtons Laws imply that the position vector of the plane
satises r

(t ) = 0. Show that in this case, the orbit is the straight line with parametrization r(t ) = r +t v, where r = r(0)
and v = r

(0) (Figure 1).


Sun
Planet
r(0)
v
r(t) = r(0) + t v
FIGURE 1
SOLUTION Integrating r

(t ) = 0 gives:
r

(t ) = c
The constant c is r

(0) = v(0). That is,


r

(t ) = v
We integrate again:
r(t ) = vt +d
The constant d is r = r(0). Hence, r(t ) = r +t v, where r = r(0) and v = r

(0).
Continuing with Exercise 37, show that the area A swept out by the radial vector r(t ) over the time interval [0, t ]
is equal to A =
1
2
r(0) vt . Thus, Keplers Second Law continues to hold since area is swept out at the constant
rate
1
2
r(0) v.
39. Suppose that the planetary orbit in Figure 2 is an ellipse with eccentricity e (by denition, e = c/a). Use Keplers
Second Law to show that if the period of the orbit is T, the time it takes for a planet to travel from A

to B

is equal to
_
1
4
+
e
2
_
T
(c, 0)
Sun
y
x
O
B
b
B
a A A
FIGURE 2
SOLUTION By the Law of Equal Areas, the position vector pointing from the sun to the planet sweeps out equal areas
in equal times. We denote by S
1
the area swept by the position vector when the planet moves from A

to B

, and t is the
desired time. Since the position vector sweeps out the whole area of the ellipse (ab) in time T, the Law of Equal Areas
implies that:
S
1
ab
=
t
T
t =
T S
1
ab
(1)
We now nd the area S
1
as the sum of the area of a quarter of the ellipse and the area of the triangle ODB. That is,
S
1
=
ab
4
+
OD OB

2
=
ab
4
+
cb
2
=
b
4
(a +2c)
322 C HA P T E R 14 CALCULUS OF VECTOR- VALUED FUNCTI ONS (ET CHAPTER 13)
Substituting in (1) we get:
t =
Tb(a +2c)
4ab
=
T(a +2c)
4a
= T
_
1
4
+
1
2
c
a
_
= T
_
1
4
+
e
2
_
D(c, 0)
Sun
y
x
O
S
1
b
B
a A
The period of Mercury is approximately 88 days and its orbit has eccentricity 0.205. How much longer does it
take Mercury to travel from A

to B

than from B

to A (Figure 2)?

You might also like